Download as pdf or txt
Download as pdf or txt
You are on page 1of 96

CATapult Courseware

Module 1
Practice Exercise Solutions
CATapult Courseware Module 1
Practice Exercise Solutions

Published by IMS Learning Resources Pvt. Ltd. in the Year 2020

Registered Office: 6th Floor, NCL Building, ‘E’ Block, Near Bandra Family Court,
Bandra Kurla Complex (BKC), Bandra (E), Mumbai - 400051
Tel.: +91 22 66170000
Toll Free: 1800-1234-467
CIN : U80220MH1999PTC121823
E-mail : [email protected] Website: www.imsindia.com

Copyright © IMS Learning Resources Pvt. Ltd.

All copyrights to this material vests with IMS Learning Resources Pvt. Ltd. No part of this material
either in part or as a whole shall be copied, reprinted, reproduced, sold, distributed or transmitted
in any form or by any means, electronic, mechanical, photocopying, recording or otherwise, or
stored in any retrieval system of any nature without the permission of IMS Learning Resources
Pvt. Ltd., and any such violation would entail initiation of suitable legal proceedings.

The views of and opinions expressed in this book are not necessarily those of the publishers. While
every effort has been made to ensure that all facts are stated correctly, the publishers regret their
inability to accept responsibility for any inadvertent errors or inaccuracies. Readers are advised in
their own interest to reconfirm facts before acting upon them.

The publishers shall endeavour, wherever possible to remedy all errors of commission and omission
which are brought to their attention in subsequent editions.

This book is sold subject to the condition that it shall not, but way of trader or otherwise, be
lent, resold, hired out, or otherwise circulated without the publisher’s prior written consent in any
form of binding or cover other than that in which it is published and without a similar condition
including this condition being imposed on the subsequent purchaser and without limiting the rights
under copyright reserved above.
CATapult Courseware Module 1

Practice Exercise Solutions

INDEX

QUANTITATIVE ABILITY
Chapter Page No.

QA-Intro Introduction to QA-DI ........................................................................... 1

QA-1.1 Properties of Numbers ......................................................................... 2-3

QA-1.2 Divisibility and GCD-LCM ...................................................................... 4-7

QA-1.3 Indices and Surds ................................................................................. 8-10

QA.1.4 Linear and Quadratic Equations .......................................................... 11-12

QA-1.5 Percentages, Profit & Loss, Interest ................................................... 13-18

QA-1.6 Ratio, Proportion, and Variation .......................................................... 19-22

QA-1.7 Means and Weighted Averages............................................................ 23-30


DATA INTERPRETATION & LOGICAL REASONING
Chapter Page No.

DI-1.1 Basics of Data Interpretation .............................................................. 33-39

DI-1.2 Calculation Based DI ............................................................................ 40-45

LR-1.1 Types of Arrangements......................................................................... 46-53

LR-1.2 Conditionalities & Grouping.................................................................. 54-57

VERBAL ABILITY
Chapter Page No.

VA-1.2 Question Types ...................................................................................... 60-63

VA-1.3 Speed Reading ....................................................................................... 64-66

VA-1.4 Understanding Passages ....................................................................... 67-71

VA-1.5 Articles and Parts of Speech ............................................................... 72-77

VA-1.6 Jumbled Paragraphs and Odd Sentence ............................................ 78-90


Quantitative Ability
Chapter QA
INTRODUCTION TO QA-DI Intro

QA-Intro INTRODUCTION TO QA-DI

PRACTICE EXERCISE

1) Give 4 children an apple each, and give the basket (containing the last apple) to the 5th child.

2) A decimal point. (Since 5 < 5.7 < 7)

3) The number of bakers and the number of days have both doubled, hence the number of cakes will increase
2 × 2 = 4 times, hence 20 cakes will be made. Hence (4)

4) The fourth child is Mary herself, naturally! Hence (4)

5) If the original number is abc, the new 6-digit number will be abc x 1002 = abc × 2 × 3 × 167. Hence it will
be divisible by 6 (2 × 3) and by 501 (3 × 167) and by 167 itself. But it need not be divisible by 37. Hence
(2)

For questions 6-10:

The following table can be made:


Candidate Round 1 Round 2 Round 3
P 13 16 20
Q 10 - -
R 12 15 19
S 11 15 21
T 14 14 -

In the first round, 5 distinct numbers, the lowest being 10, add up to 60. The numbers must be Q – 10, S – 11, R
– 12, T – 13 and P – 14. In the second round, 4 numbers add up to 60, and the sole lowest is 14 – consequently
the numbers must be T – 14, R – 15, S – 15 and P – 16 (note that in round 2 it is not said that the scores are
all distinct!). In round 3, S increases by 6 and hence reaches 21. The other two can be shown to have scored as
follows: P – 20 and R – 19. The results can be tabulated as follows:

6) T got 14 votes in round 1. Hence (2)

7) P got 16 votes in round 2. Hence (2)

8) S got 15 votes in round 2. Hence (1)

9) R got 19 votes in round 3. Hence (3)

10) S got 47 votes in all across the three rounds. Hence (4)

1
CATapult
NUMBERS

QA-1.1 PROPERTIES OF NUMBERS


PRACTICE EXERCISE
= × 100 ÷ –
1. AVG b1 1
+ , 1 + 2,
1
+ , 2 + 3l
1
1 1 2 2
= AVG(2, 3, 1, 5) = × 100 ÷
2+3+1+5 11
= = . Hence, [2].
4 4
= × 100 ÷
Answers to questions 2 to 5:
= × (100 ÷ 0.1)
2
2. R(b, c) = R(2, 3) =
3
2 2 4 = × 1000 = × 1000 =
Q(2, )=2 × =
3 3 3
4 4 -1 Hence, [3].
P(1, )=1– = < 0. Hence, [2]. Alternatively,
3 3 3
3. R(c, a) = R(3, 1) = 3 + × 100 ÷ –
S(a, b) = S(1, 2) = 3 =( + ) × 100 ÷ –
Q(a, c) = Q(1, 3) = 3
P(c, a) = P(3, 1) = 2 = × 100 ÷ ( – )
Q(3, 3) = 9; S(3, 2) = 5
= × (100 ÷ 0.1)
Now, P(9, 5) = 4. Hence, [3].

4. S(x, y) = x + y; P(x, y) = x – y
Q(x + y, x – y) = (x + y) (x – y) = x2 – y2 = × 1000 = . Hence, [3].
Q(3, x + y) = 3(x + y)
P(3x + 3y, x + y) = 3x + 3y – x – y 8. If Anil has to give 1 rupee he needs a bag with Re.1.
= 2(x + y) For 2 rupees he had two bags with Re.1 each or Rs.2
2
x – y
2
x– y bag. To have minimum bags he has a bag with Rs.2.
R(x2 – y2, 2(x + y)) = = . Hence, [4]. Now with the two bags he can give Rs.3. So next he
+
2 (x y) 2
will require a bag with Rs.4. With these three he can
5. I. S(x – y, x – y) = 2(x – y) give Rs.5, Rs.6 and Rs.7 and next bag will be one con-
) = (x + y) b l =x+
2
x x x taining Rs.8 and so on. Thus he would have bags with
II. Q(x + y,
y y y Re.1, 2, 4, 8, 16, 32. Sum of which is 63 and remain-
ing 37 can be put in the last bag. So total number of
III. P(x + y, xy) = x + y – xy
bags is 7. Hence, [3].
But, which one is the greatest can be found out only if
x and y are known. Hence, [4].
9. A = 0.a1a1a1... 10A = a1.a1a1...
st
6. 1 digit being an even prime number, is 2. Hence op-
tion 1] is ruled out.  9A = a1  A = .
The next digit is square value of 1234. So it should
end in 6. Hence, [4].
Alternatively, Similarly, B = .
1st digit is 2 & next part = (1234)2 = 1522756
Hence the number is 21522756. Now, ‘a1’ and ‘a2’ are multiples of 3 and are distinct.
Also, these values are less than 8.
either a1 = 3 and a2 = 6 or a1 = 6 and a2 = 3
7. + × 100 ÷ – a1 + a2 = 9.

= + × 100 ÷ – (0.3 + ) A + B = = = = 1. Hence, [3].

= + × 100 ÷ –

2
Chapter QA
PROPERTIES OF NUMBERS 1.1

10. The number of digits written by Sachin can be divided 16. Statement I suggests that xyz is odd. This is only pos-
into one digit, two-digit and three-digit numbers. sible if all three of them are odd. Hence, z – x is even.
Statement II states that xy + yz + xz is even.
Numbers Number of digits
For this we consider 2 possible scenarios.
written written
Case I: exactly 2 of xy, yz and xz is odd and the other
One digit 1 to 9 9 is even.
two-digit 10 to 99 180 Let us suppose that xy and xz are both odd.
If xy is odd both x and y are odd.
three-digit 100 to 999 2700 If xz is odd both x and z are odd.
Thus, from 1 to 999 he has written 2889 digits. This means that each of x, y and z are odd.
Now, last three numbers till 999 are 997, 998, 999. So case I is not possible.
Thus, 2883rd digit will be 7. Hence, [3]. Case II: each of xy, yz and xz is even.
In this case at least 2 of x, y and z has to be even.
11. Using the given options: If exactly 2 of x, y and z are even, then each of xy, yz
(i) 72 × 3.23 = 232.56 and xz will be even. However, we can have 2 subcases
(ii) 72 × 5.11 = 367.92 in case II where either exactly 1 amongst x and z is
(iii) 72 × 5.51 = 396.72 even and the other is odd or both x and z are even.
(iv) 72 × 7.22 = 519.84 So then the difference between x and z can be either
Thus, it can be seen that only option 3 is possible. odd or even.
Hence, [3]. So statement II alone is not sufficient to answer the
Note: The best way to solve this question is to multiply question.
the alternatives by 72 and find which one gives you the Since the question can be answered by using state-
middle three digits 96.7. To cut time, you can choose ment I alone we mark option (1) as the correct an-
to multiply 72 by integer values only, e.g. 72 × 3 = 216, swer. Hence, [1].
72 × 5 = 360 and 72 × 7 = 504. It must be remembered
that the decimal part of the answer will be multiplied 17. This can be answered using both the statements.
by 72 and the actual answer will increase. Statement II suggests that both t and z are odd.
Let us now roughly multiply the decimal values of the Statement I suggests that if (x + y + t) is even. Since
options also by 72, e.g. 72 × 0.2 = 14.4, 72 × 0.1 = the difference between an even and an odd number is
7.2 and 72 × 0.5 = 36. So option (1) will yield (216 + always odd, (x + y + t) – z will be odd.
14) = 230 (approximately), (2) will yield (360 + 7) = Hence, [3].
367 (approximately), (3) will yield (360 + 36) = 396 18. Depending on the whether y is greater or equal to 0.5,
(approximately) and (4) will yield (504 + 14) = 528 either x or y would be the median. In either case, me-
(approximately). Of these only option (3) satisfies our dian lies between 0 and 1.
requirement of 2nd and 3rd digits being 96. Hence, [3]. Hence, [2].
12. Let the number to be multiplied be x. 19. As x, y and z are odd numbers, any power of these
Then we can say 53x – 35x = 540. numbers will be odd.
Solving, we get x = 30. Option (1): x2y2z2 = odd × odd × odd = odd  Always
Therefore the product obtained should have been = true
30 × 53 = 1590. Hence, [4]. Option (2): 3(x2 + y3)z2 = 3(odd + odd) × odd = 3 ×
even × odd = even  Always true
13. As 2 and 5 are prime numbers, there multiplication Option (3): 5x + y + z4 = odd + odd + odd = odd 
will give the only possible zero. Hence, [1]. Always true
Option (4): z2(x4 + y4)/2 = odd(odd + odd)/2
14. The possible squares >300 are 324, 361, 400, 441, = odd × odd = odd  Always true.
484, 529,576, 625, 676, 729, 784, 841, 900 and Hence, [4].
961.
Out of these values, only 441 matches the type ccb 20. Let us evaluate each option : 2) 0 < y < 1 and z > 1 so
and also the square of 21 = 441. Therefore, b = 1 yz > 0.
Hence, [1]. Option 3) Since both x & y are not equal to 0, xy will
never be 0.
15. Option 1: (x − 1)yz = xyz – yz Option 4) y is a positive number < 1 and z is a positive
Option 2: x(y − 1)z = xyz – xz number > 1, therefore (y2 – z2) is always negative.
Option 3: xy(z −1) = xyz – xy Option 1) If |x| > |z|; x2 – z2 > 0
Option 4: x(y + 1)z = xyz + xz If |x| = |z|; x2 – z2 = 0
x > y > z  yz < xz < xy If |x| < |z|; x2 – z2 < 0
xyz – yz is closest to xyz. Hence, [1]. So Option 1 may or may not be true.
Hence, [1].

3
CATapult
NUMBERS

QA-1.2 DIVISIBILITY AND GCD-LCM

PRACTICE EXERCISE 1 8. From 1 to 9 there are 3 numbers divisible by 3.


From 10 to 99, there are 30 numbers divisible by 3.
Thus, there are 30 × 2 + 3 = 63 digits till 99.
1. Let the two numbers be 65a and 65b. ( a and b are To get 100th digit, we need to get the 37th digit after
co-prime numbers) the 2nd 9 of 99.
Sum of the two numbers = 65a + 65b = 1560 Note: 37 = 12 × 3 + 1. Hence we need the 1st digit of
 65 (a + b) = 1560 the 13th number, divisible by 3, after 99. The number
 a + b = 24 is 138.
The possible pairs of a and b will be (1, 23), (5, 19), (7,  The 100th digit of given number is 1.
17) and (11, 13) Hence, [1].
4 pairs are possible.
9. Sum of digits of 198a is 18 + a, so for a = 3 or 9 the
2. HCF ( 259, 333, 481, 555) = 37 kg number will be divisible by 3. If a = 1, 1981 is divisible
Therefore, the required answer is 37 kg. by 7. Hence, [3].

3. The factors of a2 are 1, a and a2. The factors of ab are 10. As we have to use the minimum number of marble
1, a, b and ab. The factors of a3 are 1, a, a2 and a3. tiles, the size of the tiles should be the maximum pos-
Hence, [3]. sible size. It should be the highest common factor of
2 m 56 cm and 3m 36 cm.
4. Let S1 = 17, 21, 25, ... 417 2 m 56 cm = 256 cm and
and S2 = 16, 21, 26, ... 466 3 m 36 cm = 336 cm.
So, terms of S1 are in the form 4n + 1 (4 < n < 104) 256 = 16 × 16
and terms of S2 are in the form 5m + 1 (3 < m < 93) 336 = 16 × 21
In order to have same terms, we should get 4n = 5m. The HCF of 256 and 336 = 16
Hence, the side of the square is 16 cm.
This happens only 20 times.
The number of such square marble tiles required =
Thus, we get 21, 41, 61, .... 401 i.e., 20 common 256 × 336
terms. Hence, [3]. = 336 marble tiles. Hence, [2].
16 × 16

5. Consider n = 8 Answers to questions 11 and 12:


Number of factors of 8 = 23 are k = 4
n 11. H.C.F. of the individual number of diamonds is 13.
k> .
2
Similarly, the numbers 1, 2, 3, 4, 6, 8 and 12 satisfy
the given condition. Hence, [1].
Hence, [1].
6. We need to use coins of maximum denominations for
minimum number of coins. 12. H.C.F. of the individual number of diamonds is 17. So
 36 paise = 25 + 5 + 2 + 2 + 2  5 coins they have 17(3 + 5 + 6) = 17 × 14 = 238 diamonds.
77 paise = 50 + 25 + 2  3 coins 238 is not a multiple of 3. So it is not possible to dis-
Rs.2.03 rupees = 50 + 50 + 50 + 25 + 10 + 10 + 2 + 2 tribute them evenly. Hence, [4].
+ 2 + 2  10 coins.
Total number of coins = 5 + 3 + 10 = 18. 13. To divide according to the condition, the share each
Hence, [3]. one will get

= HCF b 4 , 3 , 6 l = HCF b , l
1 1 1 9 13 19
,
7. The LCM of 135, 72, and 120 is 1080 seconds = 18 2 4 3 2 4 3
minutes. 1
This means all lights will flash simultaneously after =
12 1
18 minutes. But in this 18 minutes the clock will lose i.e., each guest will get litres.
2 × 18 = 36 seconds. Therefore clock will show 36 12
Total number of guests
seconds less than the correct time. 9 13 19
Hence 8 : 17 : 24. Hence, [4].
2 + 4 + 3
= = 54 + 39 + 76 = 169.
1 1 1
12 12 12
Hence, [3].

4
Chapter QA
DIVISIBILITY AND GCD-LCM 1.2

14. Given HCF = 3 × 5 and LCM = 2 × 3 × 5 × 7. 12k + 1


 If a and b are the two numbers, As is a composite number, only 65 and 77
5
ab = 2 × (3 × 5)2 × 7
are the possible values.
Also a = (3 × 5) × k and b = (3 × 5) × m, where k and m
Hence, 162 and 192 are the only 2 numbers that sat-
are non-zero non-unit integers.
isfy the given conditions.
Then a = 2 × 3 × 5 = 30 and
Hence, [3].
b = 3 × 5 × 7 = 105. Hence, [2].
Alternatively,
20. Let HCF be h and LCM be , .
15 is HCF, so, 15 should divide both the numbers in
I. Let the numbers be ah and bh.
the pair. Only in option [2], 30 and 105 both are divis-
Then abh = , and (a + b)h = m
ible by 15.
or (a – b)h = n
Using these ah and bh can be uniquely
15. Gifts were equally divided among 10 & 6 students.
determined. Thus, I is true.
 Least number of gifts is the LCM of 10 and 6, i.e.,
30. Hence, [2]. II. If HCF = LCM then the two numbers are equal
and same as HCF or LCM.
16. Let the HCF of x and y be g. Thus, II is true.
So, HCF of y and x – y is also g. LCM ,
Also product of 2 numbers = HCF × LCM. III. = a prime i.e., = P.
HCF h
 (xy) – [y(x – y)] = g × 18 Then one of the numbers is equal to h and other
 y2 = 18g = 32 × 2g is equal to , . Thus, III is true.
 3 and 2 must be factors of y. Hence, [4].
 y cannot be 16, 27 or 4.
Hence, [2].
Alternatively, PRACTICE EXERCISE 2
Let, LCM of x and y is p
Let, LCM of y and (x – y) is q 1. Let us choose any prime number greater than 6 and
 y is a factor of p & q. check. When 7 is divided by 6, remainder = 1. When
|p – q| = 18 11 is divided by 6, remainder = 5.
 y divides p and q, y divides 18. The only such option Hence, [2].
is 18.
2. The bells will chime together after a time that is equal
17. Solution: Any number, which when divided by 6, 15 to the LCM of 18, 24 and 32 = 288 min = 4 hr and 48
and 17 leaves a remainder of 1, is of the form {k × min. Hence, [2].
LCM(6, 15, 17) + 1}, where = 1, 2, 3… and so on.
k × LCM(6, 15, 17) + 1 = 510k + 1 3. We can use options to solve this problem. In option
It is given that (510k + 1) is a multiple of 7. (4), 84 + 53 = 137. This when divided by 3, 4 & 7
When k = 1, we get 511, which is a multiple of 7. leaves remainders as 2, 1 & 4 respectively. No other
Hence, [2]. option satisfies this. Hence [4].

18. If (k + 4) is divisible by 7, then = 3, 10, 17, 24, … 1


It is given that k + 2n is divisible by 7. 4. Red light flashes once in every of a minute
3
If = 3, then the value of n, which is greater than 2 2
such that (3 + 2n) is a multiple of 7 is 9. Green light flashes once in every of a minute
5
Hence, [1]. Taking LCM of both the figures, we can find the time
1
19. Since the number (n) is divisible by both 2 and 3, required by both to flash together = LCM of and
2 3
it must be divisible by 6. Let the number be of the = 2.
form 6k. Also, (2n+1) is divisible by 5 i.e. [2(6k) + 5
1]  12k+1 is divisible by 5. For 12k+ 1 to be divisi- Therefore, they both flash once every two minutes.
ble by 5, 12k must end in a 4 or a 9. But 12k can never 60
Hence in 1 hour, they will flash = 30 times.
2
end in a 9. Thus, 12k ends in a 4. Hence, [1].
The values 12k can take are 24, 84, 144, 204….
Thus k can be 2, 7, 12, 17, 22, 27 …. 5. The difference of these two numbers will be com-
6k can be 12, 42, 72, 102, 132, 162 and 192. (As pletely divisible by ‘n’. Therefore, 1535 is divisible by
6k < 200) n, and hence, option 4 is correct as 307 × 5 = 1535.
12k + 1 Hence, [4].
And can take values 5, 17, 29, 41, 53, 65
5
and 77.

5
CATapult
NUMBERS

6. Elements of set A would have the property of being 1


b 16n l b6l
2
+ 7n + 6
less than the common multiple of the given divisors 12. is an integer when is an in-
n n
(2, 3, 4, 5 and 6).
Thus, every element of set A is of the form 60k – 1 teger.
Between 0 and 100, only 59 satisfies the above con- This is possible when n = ±1, ±2, ±3 or ±6. Thus, the
dition. number of values for n = 8. Hence, [4].
Hence, [2].
13. If we were to express 64 as product of two positive
7. The L.H.S. is an addition of ‘n’ terms. Each of the integers, we can get the following combinations: (64
terms is a product of 4 numbers. × 1), (32 × 2), (16 × 4), (8 × 8).
Each of the 4 numbers is either equal to 1 or –1. The Thus, we find that P + Q cannot be 35.
product of these 4 numbers will be 1 or –1 (depend- Hence, [4].
ing upon the count of 1 & –1)
Eg. 1 × 1 × 1 × 1 = 1, 1 × (–1) × (–1) × 1 = 1, 14. Let m = 5a and n = 5b
1 × 1 × 1 × (–1) = –1, 1 × (–1) × (–1) × (–1) = –1. (a & b are integers)
The sum of all the terms is 0. It is possible when an Option (1): m – n = 5a – 5b
equal number of 1’s & –1’s are added. = 5(a – b)  Always divisible by 5
Eg 1 + (–1) + (–1) + 1 = 0, 1 + 1 + (–1) + (–1) + 1  0 Option (2): m2 – n2 = (5a)2 – (5b)2 = 25(a2 – b2) 
Thus, total number of terms must be a multiple of 2 Always divisible by 25
i.e. even. Hence [1]. Option (3): m + n = 5a + 5b = 5(a + b)  May or may
not be divisible by 10.
8. The time taken by the white spots on all three wheels Hence, [3].
to simultaneously touch the ground again will be
equal to the LCM of the times taken by the three 15. n(n2 – 1) = (n – 1)n(n + 1). If you observe, this is the
wheels to complete one revolution. product of three consecutive integers with middle
The 1st wheel completes 60 revolutions per minute. one being an odd integer. Since there are two consec-
This means that to complete one revolution, it takes utive even numbers, one of them will be a multiple
60 of 4 and the other one will be multiple of 2. Hence,
= 1 second. the product will be a multiple of 8. Also since they are
60
The 2nd wheel completes 36 revolutions per minute. three consecutive integers, one of them will definitely
This means that to complete 1 revolution, it takes be a multiple of 3.
Hence, this product will always be divisible by (3 × 8)
36
= 0.6 seconds = 24.
60 Hence, [3].
24
Similarly, the 3rd wheel takes = 0.4 seconds.
60 16. Solution: Given: N = 774958A96B
Therefore, LCM of 1,
3 2
, will be
6
= 6 seconds For a number to be divisible by 8, the last 3 digits
Hence, [3].
5 5 1 have to be divisible by 8. Here, 96B should be a multi-
ple of 8. The set of values that satisfies this condition
9. Since 899 is divisible by 29, so you can directly divide are 960 and 968  B = 0 or 8
the remainder of 63 by 29, therefore, giving 5 as the For a number to be divisible by 9, the sum of all the
remainder. Hence, [1]. digits has to be divisible by 9. Here, (7 + 7 + 4 + 9 + 5
+ 8 + A + 9 + 6 + B) = (55 + A + B) should be a multiple
10. HCF of 60, 84 and 108 is 12. Hence, 12 students of 9.
60 If B = 0, then (55 + A) should be a multiple of 9, thus
should be seated in each room. So for subject A, A should be 8.
12
= 5 rooms. If B = 8, then (63 + A) should be a multiple of 9, thus
84 A should be 0.
For subject B, we would require = 7 rooms
12 Hence, [2].
108
For subject C, we would require = 9 rooms 17. As n(n + 1) forms two consecutive integers, one of
12
Therefore, minimum number of rooms to satisfy our them will be even and hence the product will always
condition = 5 + 7 + 9 = 21 rooms. be even. Also, the sum of the squares of first n natural
Hence, [4]. numbers is given by n(n + 1) (2n + 1)/6. Hence, the
product will always be divisible by this.
11. The number formed by the last 3 digits of the main Also, we can verify that the product is always divisible
number is 354. The remainder is 2 if we divide 354 by by 3. However, we can find that the option (4) is not
8. So, the remainder of the given number is also 2 if necessarily true. Only under certain situation does it
we divide it by 8. Hence, [3]. hold good. e.g. if n = 130, (2n + 1) = 261
Hence, [4].

6
Chapter QA
DIVISIBILITY AND GCD-LCM 1.2

18. The LCM of 7, 12 and 16 is 336. The nearest multiple


of 336 to 1856 is 1680. So 1684, when divided by 7,
12 and 16, leaves a remainder of 4. This is the clos-
est such number to 1856. Hence, the number to be
subtracted from 1856 to get 1684, must be the least
such number. So, the answer is (1856 – 1684) = 172.
Hence, [4].

19. From statement I, we get to know that if the number-


indicating the class strength is divisible by 8 adding
12, then it will also be divisible by adding 4.
So statement I alone is sufficient to answer the ques-
tion.
Statement II only tells us that the number of students
is not divisible 8.
So using statement II alone we cannot answer the
question.
Hence, [1].

20. Using statement I alone, possible values of x are 4, 12


and 36.
So using statement I alone, we do not get a unique
value of x.
Using statement II alone, possible values of x are 4,
8, 10, 14 ..... and so on.
So using statement II alone, we do not get a unique
value of x.
Using both statements together, we know that the
product of 2 numbers = LCM × HCF = 36 × 2 = 72. So,
72
x= = 4.
18
Hence, both the statements are required to answer
the question.
Hence, [3].

7
CATapult
NUMBERS

QA-1.3 INDICES AND SURDS

PRACTICE EXERCISE
43 9
1. Solve: c)
9
= 3 b3l= 3 b l=
1 2 1 4 1×3 9 4 43 9
a) 100 33 3
+ 3
2 3 3 3 ×3 9 3 9

b) 211 = 2048 d) 4/27


2 2 2 2 2 2
c) 0.64. 2 × 2 × 2 × 2 ÷ (2 × 10 ) 6
22 6 4
= 27 = 4/27
= 26 ÷ 100 = 0.64
6
33
1

. b 64 l = 271/3 ÷ 641/3 = 3/4 e) 10


3 27 3
d)
4
3
40 2
2
× 2 × 3 25 = 3 40 × 25 = 3 8 × 5 × 25
2
e) 236
= 2 × 5 = 10
7 f) x = 16
f) 2 = 128.
6
4 = 12 16
8 12 6 8 + 12 - 6 14 7
2 ×2 ÷2 = 2 = 2 =2
g) 4 7 -4 2
25
g) x = 4/3
1 ( 7 - 2) 2 - 7 5 ( 7 - 2) 2 - 7 4 7 -4 2
h) t = –1 ( 2 - 7) × ( 7 - 2)
+ 25 = 25 + 25 = 25

2. a) 33 = 27 1 ( 7 - 2) 2 - 7 5 ( 7 - 2) 2 - 7 4 7 -4 2
+ 25 = 25 + 25 = 25
( 2 - 7) × ( 7 - 2)
b) n = 9

c) 1/8 4. Find the positive square root of:


a) 2 + 6
d) 400000000. 45 × 252 × 252 = 45
= 4 × 1004 = 400000000
b) 2 2 + 3
e) 54. 24 × 36 ÷ 23 ÷ 33 = 2 × 33 = 54
c) 1 + 2 6
9
f)
32 5. (1.2)4 is greater
g) 1
6. (x8)11 < y11
h) 0.5  x8 < y  256 < y.
Hence, [1].
3. Simplify: 7. Let the number of trees be n

a) 7 2 - 5 3 2 + 3 6  n2 = 5625 n= = 75 Hence, [3].

2 2 - 33 6 - 23 2 + 5 2 = 7 2 - 53 2 + 3 6
8. =
4 3
b)
27
3 3 4 3 = = =3 . Hence, [2].
9 + 27 = 27

8
Chapter QA
INDICES AND SURDS 1.3

9. (–2)–1/3 is the only negative option. =2+ = 2 + 6i, which is not an integer
Hence, [2].
(2)–3/2 = and (3)–1/2 = 13. The given expression is
–1/2
 (3) > (2)–3/2

=
= = =

= =  improper fraction.
–1/2
(3) = =
Hence, [2].

14. Put a = 5  b = 4
–1/2
 >3 . Hence, [3].  = lies between 2 to 3

10. [ ]=2

 = =2

Substitute in options, only option [3] gives 2.


= (22 – 21) –
Hence, [3].
= 2 – 4 = –2 4×4
3n +
4×4
3n–3 256
Hence, [2]. 15. 2n n =
6×4 –6 3
Alternatively,
Suppose x = 0
256
 =
3
we have –
256
= – 4 = 2 – 4 = –2.  =
3

11. a = 256 × 336 × 1144 256


 =
= 22(254 × 336 × 1144) 3
b = 254 × 337 × 1145
= 3 × 11(254 × 336 × 1144)  4n + 3 = 256
c = 258 × 338 × 1144  4n × 64 = 256
= 24 × 32(254 × 336 × 1144) n=1
d = 257 × 336 × 1145 Hence, [2].
= 23 × 11(254 × 336 × 1144)
 c > d > b > a. Hence, [2]. 16. We make all the powers equal.

2 2 = ]2 g = ]64g12
1
6
1 1
12

12. 1] 27 +
3 3 = ]3 g = ]81g12
1 4
1 1
12

= + = 10 3  not an integer
4 4 = ]4 g = ]64g12
1 3
1 1
12

2] + 36
6 6 = ]6 g = ]36g12
1 2
1 1
12

= (–2) + 6 = 4 integer

Comparing bases, ]81g12 = 3 3 is the largest.


1 1
3] + –36
Hence, [2].

9
CATapult
NUMBERS

17. At each stage, displayed entry can be thought of as


a pair of x. Each reciprocation reverses sign of expo-
nent of x. Each squaring doubles the exponent.
Thus each pair of squaring : reciprocal multiplies the
exp by –2.

 Final exponent = y= .


Hence, [3].

18. 1] As n3 is odd, it implies that ‘n’ will be odd as cube


of an odd number has to be odd and cube of an even
number has to be even. Therefore, Statement 1 is
true.
2] As ‘n’ is odd, n2 will also be odd.
Therefore, Statement 2 is also true.
Hence, [3].

19. 5a = 26 (5a)2 = 676


52a =   
 Also given, 125b = 676
(53)b = 676 53b = 676 .... (II)
Equating (I) and (II) we get
52a = 53b
As bases on the L.H.S and R.H.S are equal, the pow-
ers have to be equal.
 2a = 3b
 Hence, [3].

20. a44 < b11


a=2
 244 < b11
This is only possible when b > 24
Thus, answer can be obtained using statement B only.
Statement A does not give relevant information to ob-
tain accurate answer. Hence, [1].

10
Chapter QA
LINEAR AND QUADRATIC EQUATIONS 1.4

QA-1.4 LINEAR AND QUADRATIC EQUATIONS


PRACTICE EXERCISE Now, it is said that (Margin of loss this year) = 2
x (Margin of victory last year).
1. If x < y, y – x/2 = 3(x – x/2). Therefore, (2.5x – 260000) = 2(260000 – 2x).
x 2 Solving this equation we get, x = 120000.
 = . Hence, [1]. This means that 120000 votes were not cast for
y 1
theparty in the previous assembly polls.
2. In this case, since x, y and z are distinct positive Therefore, the number of votes cast for the party
integers, our aim is to figure out which of the = 260,000-120,000 = 140,000. Hence, [3].
answer choices cannot be expressed as the sum
of 3 integers uniquely. For eg. 6 can only be ex- 7. The worst scenario is when the other four get an
pressed as (1+2+3). 7 can only be expressed as equal and the winner gets more votes than each
(1+2+4). But 8 can be expressed as either (1, 2, of number of votes. Let the winning candidate get
5) or (1, 3, 4). Hence, [4]. x votes.
261 – x
3. Let the three numbers be a, b, c, such that a is  x > 4
the largest and c is the smallest.  x > 52  x = 53. Hence, [1].
a-c+b = a+b+c
Now – 14 8. x = (108.22 + 1042 - 208 × 108.2) (108.22 + 1042 + 104 × 108.2)
3 3 (108.2 3 - 104 3)
 a – c + b = a + b + c – 42
x =
2 2 2 2
(108.2 + 104 –2 × 104 × 108.2) (108.2 + 104 + 104 × 108.2)
 c = 21. Hence, [3]. 3 3
(108.2 – 104 )
3 3 2 2
[a – b = (a – b) (a + b + ab)]
4. Let the number of boys be x and the number of
2
girls be y (108.2 – 104) 2 2 2
x = (108.2 – 104) [(a – b) = a + b – 2ab]
 x + y = 100 ..... (i)
1 x = 108.2 – 104
Number of boys who failed in Maths =
3x x = 4.2
1
 Number of boys who passed in maths = x –  : x + 0.8 D = b 4.2 + 0.8 l = b 5 l = 25. Hence, [2].
2 2 2
3 x - 3.2 4.2 - 3.2 1
2
x =
3x 9. The total red tiles initially = 2 b m l – 1
Similarly, number of girls who passed in English Total number of tiles = b m l
2 n

2 3 n
= y –
5y= 5y As number of red tiles > number of green tiles
2 3 (since the room has odd number of tiles and corner

3 x + 5 y = 64 tiles are red)
 10x + 9y = 960 ..... (ii)  Total red tiles required to form an alternate
bm l + 1
2

Multiplying equation (i) by 9 and subtracting n 2


+ 2
pattern = = m 2n
from (ii) 2 2n
x = 60. Hence, [3].  Number of green tiles removed
= m +2n - 2 b m l + 1 = m + n - 4mn
2 2 2 2
+ 2n2
5. If x and y are the two unequal parts, then 2n n 2n2
x2 – y2 = 27(x – y)
= (m - 2n)2 - n . Hence, [2].
2 2

 x + y = 27 ( a x  y). Hence, [3]. 2n


2
10. x 2
y z2 ... (Since, x + y + z = 0;
6. Let x be the number not cast for Praja Party in yz + zx + xy
the previous polls. So the number of votes not x3 + y3 + z3 = 3xyz)
cast for the party in this assembly polls would be 3 3 3

1.25x. This means that the number of votes cast = x + y + z = 3xyz = 3. Hence, [4].
xyz xyz
for the party in the two polls would be (260000 1 1 1
– x) and (260000 – 1.25x) respectively. 11. x3 + y3 + z3 = 0
 x 3 + y 3 = - z 3 & _ x 3 + y 3 i = (- z 3 ) 3
Margin of victory in the previous polls = (votes 1 1 1 1 1 3 1

cast) – (votes not cast) = (260000 – x) – x =


(260000 – 2x). 1 1
 x + y + 3y 3 x 3 (x 3 + y 3 ) = –z
1 1

Margin of loss in this year’s polls = (votes not


cast) – (votes cast) = 1.25x – (260000 – 1.25x)  (x + y + z)3 = 27xyz. Hence, [1].
= (2.5x – 260000).

11
CATapult
ALGEBRA

12. The best to solve this question is the method of 19. Let the cost of each orange be ‘a’, each bananna
1 1 5 be ‘b’ and each apple be ‘c’.
reverse substitution. Since + = , the
4 6 12  3a + 5b + 2c = 40 ...... (1)
answer is option 3.
and 4a + 10b + 4c = 70
Hence, [3].
i.e. 2a + 5b + 2c = 35 ...... (2)
(1) – (2) gives
13. If the numbers are (x – 2), x and (x + 2), then
a = 5 i.e. 10a = 50. Hence, [2].
3(x – 2) – 2 = 2(x + 2).  x + 2 = 14, which
incidentally is the third number. Hence, [2].
20 . First number = (34 + x)
Second number = (43 – x)
14. Let one number be x, so the other number would
Product = (34 + x) (43 – x) = 1482
be(x + 4). We have to find 2 integers whose sum
 1462 + 9x – x2 = 1482
is 10 and the product is 21.
 x2 – 9x + 20 = 0
X(x + 4) = 21 or x = 3. Hence, [1].
 (x – 5) (x – 4) = 0
 x = 5 or x = 4
15. The given equations are
If x = 5, first number = 39 and second number
2x − 8y = 3 …(i)
= 38.
kx + 4y = 10 …(ii)
If x = 4, first number = 38 and second number
Suppose ax + by = c and dx + ey = f are two
= 39.
equations involving two variables x and y.
Hence the smaller number is 38. Hence, [2].
a b c
If d = e ! f then we get no solution for the

given equations
c 3 b -8
= =
f 10 and e 4
b c
 e ! f

So the above two equations will have not solution


2 8
if k = - 4

2×4
 k = – 8 = –1.

So for k = –1 the given equations have no solution.

Hence, [4].

1
16. Sita takes of the total mints kept there. This
3
means the total number of mints in the bowl
should be a multiple of 3. There is no option that
satisfies this logic. Hence, [4].

17. Using the options, the sum of 25 & 16 is 41.


Difference of 54 and 41 is 13. Hence, [2].

18. Let the three consecutive integers be (x – 1), x


and (x + 1)
Now (x – 1) (x) (x + 1) = 3x × 40
 x2 – 1 = 120 [ a x cannot be zero]
 x2 = 121
x = + 11
If x = 11, the numbers are 10, 11 and 12, and
the largest number is 12.
If x = –11, the numbers are –10. –11, –12 and
the largest number is –10.
 |Largest number| = 12 or 10.
Hence, [4].

12
Chapter QA
PERCENTAGES, PROFIT & LOSS, INTEREST 1.5

QA-1.5 PERCENTAGES, PROFIT & LOSS, INTEREST

PRACTICE EXERCISE 1 16. Let the price of 1 unit of electricity be Rs.x Varun ini-
tially consume y units.
1. 1% loss Therefore he spent initially Rs.xy on electricity.
When the price of electricity decreased by 20%, the
2. 4 pcpa new price per unit of electricity is Rs.0.8x.
Varun decides to spend 10% less i.e. 0.90xy
1 Let the new consumption of electricity be U units
3. 33 3 %  0.90xy = 0.8x × U

4. profit, 25%
U=
5. 30% (SP is 117% of CP and also 90% of MP. So, MP =
 U = 1.125y
117/90 CP = 1.3 CP.)
He increased his consumption by 12.5%. Hence, [4].
5 2
6. 3 or 1 3 or 1. 6 17. For fixed rate amount to be paid after three years =
(1.06)3 × 300000
7. no profit no loss (60% of 40 = 40% of 60) = 1.191 × 300000 = Rs.357304.8
For floating rate
8. 20% (CP = 600. 4% reduction in Rs.750 will make the Amount for first year = 1.07 × 300000
SP = Rs.720. So profit% = 20% ) = Rs.321000
Amount for second year = 1.06 × 321000
9. 25% = Rs.340260
Amount for third year = 1.05 × 340260
10. 20 pcpa = Rs.357273
(He was charged Rs.120 as interest on Rs.2400 for 3 Hence floating rate is beneficial by
2400 × 1 4 × R 357304.8 – 357273 = Rs.31.8. Hence, [3].
months = ¼ year. So, 120 = 100 )
11. Original value = 3925 + 2355 = Rs.6280
Depreciation = Rs.2355
18. 1100 =

 % depreciation = = 37.5%. Hence, [1].


1100 =
12. 25% of the good mangoes represent 261
261
 Total number of good mangoes = = 1044
0.25
87% of the total mangoes is represented by 1044
1044
 Total number of mangoes = = 1200.
0.78
Hence, [3].

13. Cost price of 15 books = 8 × 15 = Rs.120


12.5
Discount = = Rs.15. Hence, [4].
100
14. Let the population of each part be x
 0.80x + 1.15x = 39390
39390
x= = 20200. Hence, [2].
1.95
 P = Rs.1000. Hence, [2].
15. = 3024  R = 5%
19. Let CP of 5 dozen mangoes be Rs.x.
SP = Rs.156 and Gain = 0.3x
 = 611  T = 3.5 years.  156 – x = 0.3x  x = 120
 SP of 60 mangoes = 120 × 1.6 = Rs.192
Hence, [3].  SP per mango = Rs.3.2. Hence, [3].

13
CATapult
ARITHMETIC

20. Selling price = 800 × 1.25 = Rs.1000 5. Let there were x men and y women present in the party.
1000  x + y = 55 ..... (i)
 Marked price = Rs.  1053. Hence, [2].
0.95 40% of the women wore earrings, so the number of
women wearing earrings was 0.4y.
Hence, total number of earrings was 0.8y.
PRACTICE EXERCISE 2 60% of women and 50% of men wore goggles.
Total number of goggles =
0.5x + 0.6y = 1.5 the number of earrings.
4 1 1 i.e., 0.5x + 0.6y = 1.5(0.8y)
P × 3.75 × 3 160 × 2 ×2
12 2 2  0.5x = 0.6y ..... (ii)
1. =
100 100 Solving (i) and (ii), we get, x = 30 and y = 25.
 P = Rs.80 Hence, [4]. Hence, 10 women wore earrings.
 total number of earrings were 20. Hence, [3].
6. Suppose there are 100 students in the school;
then 40 of them being boys, 60 are girls.
2. Here P = 21600. Number of successful students = 40.
20
Number of successful girls = × 60 = 12.
100
 P = 21600.
 Percentage of boys who passed = ×
100 = 70. Hence, [3].
 P = 21600.
500 500
7. Having secured , Teena got × 100
 P  29630. Hence, [1]. 600 600
 83.33%.
Hence Beena scored (approximately) 85.33%, or
3.
85.33
equivalently × 600 = 512 marks (out of
100
600).
Then, Meena, having secured 75.33% (10% less
75.33
than Beena), scored × 600 = 452 marks
100
(1.32x – 120) – (x) = 200 (out of 600).
 0.32x = 320.  x = Rs. 1000. Hence Seema got 452 + 16 = 468 marks (out of
Therefore, the initial cost of the camera is Rs.1000 468
Hence, [2]. 600), which is × 100 = 78%.
600
Hence, [4].
4. 4.15% may not be an easily identifiable percentage.
Alternatively,
But 4.15% × 2 = 8.3% 
1
b 1 . 8.33% l Maximum marks possible in the exam = 600
12 12  1%  6 marks.
1
 4.15%  Thus, Teena  500 marks
24 1
Similarly 3.84% × 2 = 7.68%  Beena (+2%)  500 + 2(6) = 512 marks
1 25 13 Meena (–10%)  512 – 10(6) = 452 marks
 3.84%   M.P. = S.P. Seema  452 + 16 = 468 marks
25 26 26
also C.P. = S.P. 468
24  % = 78%.
6
25 25 Hence, [4].
 M.P. = C.P.
26 24
8. Let the number of women wearing 1 colour, 2 colour
25 and 3 colour dresses be a, b and c respectively.
 = ×
24 b + c = 70% of the female population.
 a = 30% of the female population = 1200.
1 1200
 = =1+  Female population =
12 0.3

Now
1
= 8.33%  Female population = 4000. Hence, [1].
12
 Mark up percentage = 8.33%. 9. Let the marks obtained in English and Hindi be x and
Hence, [4]. y respectively.
x + 1.5x + 48 + 2y = 256 ... (i)
0.5x + 2.5x + 48 + 2y = 280 ... (ii)

14
Chapter QA
PERCENTAGES, PROFIT & LOSS, INTEREST 1.5

From (i) and (ii) we get


x = 48, y = 44
Marks in Math and Science = 256 – (48 + 44) = 164
164
 Required percentage = × 100 = 82%.
200 = ..... (ii)
Hence, [2].
Equating (i) and (ii),
10. E’s C.P. = 500000 × 0.85 × 0.9 × 0.82 × 1.11.
= 348151.5. Hence, [1].
 =
11. Let the shopkeeper buy goods worth Rs.x.
655
 × 100 = R
bx - l
1 x 3 80 5000
 × x × + × × +
3 3 8 100  R = 13.1%. Hence, [1].

80 110 – loss% True weight


×y× = x 15.
100
=
100 100 100 False weight
Where y is the factor by which the price of remaining
goods is increased. 1
 100 – loss% = × 100
5x 80 1.2
 0.4x + 0.2x + ×y× = 1.1x 1000
xy 12 100 = = 83.33
12
= 0.5x  y = 1.5
3  loss% = 100 – 83.33
 The goods should be marked at 50%. Hence, [1]. 2
= 16.67% or 16 %. Hence, [2].
12. Let the merchant buy 50x watches at Rs.6000 3
per 12 pieces and the same number Rs.15000 16. Suppose the price of sugar is Rs. 10 per kg and the
per 25 pieces. consumption of sugar is 10 kg. Therefore, the current
 Cost of 50x watches @ Rs.6000 per dozen expenditure on sugar = Rs. 100. If the price of sugar
increases by 50%, new price = Rs. 15 per kg. If the
= = 5x × 5000 increase in expenditure is 20%, the new expenditure
on sugar will be Rs. 120.
Cost of 50x watches @ Rs.15000 per 25 pieces 120
 New consumption of sugar = = 8 kg
15
Therefore, the family should reduce its consumption
= = 2x × 15000 of sugar by 20%.
Hence [2].
 Cost of 100x watches = Rs.55000x
Now selling price of 100x watches = Rs.68000x
17. CP of the merchant who calculates his % profit on
 Profit, 68000x – 55000x = 13000x = Rs.39000 3760
 x = 3. CP = = Rs.3200
1.175
 He bought 100 × 3 = 300 watches. Hence, [1].  His profit = 0.175 × 3200 = Rs.560

13. Let CP be Rs.x  MP = 1.8x Profit of the merchant who calculates his % profit on
SP of 15 articles = 12 × 1.8x = 21.6x SP = 0.175 × 3760 = Rs.658
 Discount = 0.9 × 21.6x = 19.44x  Difference in profit = 658 – 560 = Rs.98.
 Profit on 15 articles = (19.44 – 15)x = 4.44x
4.4x 18. P(1 + r)2 = 6272 ………………. (I)
 Profit % = × 100 = 29.6%. Hence, [3].
15x P(1 + r)3 = 7024.64 …………….. (II)
Equation (II) divided by equation (I) yields
14. Let ‘P’ be the initial investment.
Scheme I: 7024.64
1+r= = 1.12
Amount at the end of 3 years 6772
Therefore, r = 12%. Putting the value of r = 12% in (I),
we get
= = = ..... (i) 6272
P= 2 = 5000
Scheme II: 1.12
Let ‘R’ be the rate of interest for the third year. Therefore, the required answer is 5000.
 Amount at the end of three years

15
CATapult
ARITHMETIC

19. Suppose the CP of one bat = 4. 2. The price of the house after ‘n’ years = 80,00,000 ×

Therefore, the SP of one bat = = 5.


= 80,00,000 ×

Therefore, profit % = × 100 = 25%. The value of his investment after ‘n’ years = 40,00,000
Hence [4].
× b1 + l = 5,00,000 × (n + 8)
12.5n
20. Suppose the Cp of the smaller diamond = a and the CP 100
of the larger diamond = b.
Therefore we have –  5,00,000 × (n + 8) > 80,00,000 ×
CP SP
Smaller a 1.2a
 (n + 8) > 16 ×
Larger b 0.9b
This equation is satisfied for 3 and all numbers great-
Total a+b 1.2a + 0.9b er than 3.
Therefore, the required answer is 3.
 Total loss = 0.1b – 0.2a
9 20 3. Let the rate of interest be r% p.a.
Loss% = 1 %= % P×r
11 11  = 105 ….. (I)
20 100
Loss 11 20 1 1 The principal amount for the 2nd year will be P + 105
 = = × =
CP 100 11 100 55 + 45 = P + 150
 We have – (P + 150) × r
0.1b - 0.2a 1  = 1765.50 – (P + 150)
100
=
a+b 55 (P + 150) × r
 5.5 b – 11a = a + b i.e. = 1615.50 – P ….. (II)
100
 4.5b = 12a
Dividing equations I and II, we get
b 12 8
 = = P 105
a 4.5 3 =
0.9b 3 b P + 150 1615.50 - P
Required ratio = = =
1.2a 4 a  1615.5P – P2 = 105P + 15750
3 8 2  P2 – 1510.5P + 15750 = 0
 Required answer = × =  (P – 1500) (P – 10.5) = 0
4 3 1
Hence, [2].  P = 1500 or P = 10.5
105 × 100
If P = 10.5, then r = = 1000%.
10.5
PRACTICE EXERCISE 3 If P = 1500, then r =
105 × 100
= 7%.
1500
Hence [4].
1. Suppose P is the principal and r is the rate of interest
per annum. 4. Let the CP of mangoes per kg be Rs. X. Let the quanti-
The ratio of the compound interest to simple interest ty sold on profit be y kg.
= 331 : 300. Suppose the compound interest is 331x  X × 0.15 × 40 = X × 0.2 × y
and the simple interest is 300x over the 3 years.  y = 30 kg
We have, Hence [3].
Year 1 2 3 5. Suppose the Marked Price = 100.
Simple interest 100x 100x 100x Therefore the Selling Price after three successive dis-
counts of 10%
Compound in- 100x 100x(1 + r) 100x(1 + r)2 = 100 × 0.9 × 0.9 × 0.9 = 72.9
terest The Selling Price after two successive discounts of
Therefore, we have x% = 100 × (1 – x)(1 – x) = 100(1 – x)2
100x(1 + r) + 100x(1 + r)2 = 231x  100(1 – x)2 = 72.9
 200x + 100x(3r + r2) = 231x  (1 – x)2 = 0.729
 r2 + 3r = 0.31  1 – x = 0.729 = 0.8538
Solving for r, we get r = 0.1 or r = 10%.  x = 0.1462 or 14.62%
Therefore, the required answer is 10%. Hence, [2].

16
Chapter QA
PERCENTAGES, PROFIT & LOSS, INTEREST 1.5

6. Rate of inflation = 1000% 5 2


 The amount of increase = 10 times the original Solving we get P = – or P =
2 5
amount. 2
Thus, the new amount would be (10 + 1) = 11 times As P cannot be negative, P =
2 5
the original amount. Now AP = 441
4
Present cost of the article = 6 units. A× = 441
Thus, cost after 2 years = 6 × 11 × 11 = 726 units. 25
Hence, [4]. A = 2756.25
Hence, [1].
7. Number of leap years in the 20th century (1901 – Alternatively,
2000) = Number of leap days (Feb 29th) = 25 Working backwards, using the options option [4] and
Number of days in the 20th century = (100 × 365) + [2] cannot be the answer because after one cycle,
25 = 36525. their value will be less than the selling price given af-
ter the 2nd up down cycle. Option [3] after cycle 1 will
Thus, required percentage = b l × 100 =
25
become 2059. Now 2059-1944.81 = 115.19. How-
36525
ever, after the 2nd up down cycle 2059 will reduce by
0.0684%. Hence, [3].
a value just not much less than 441. So after, cycle 2,
it will be definitely less than the given selling price of
8. Let the maximum marks in each paper be 100. So he
1944.81. Hence option [1] i.e. 2756.25 is the correct
gets 6x, 7x, 8x, 9x and 10x in each of the papers re-
answer.
spectively. Then 60% of total marks = 40x 300 =
40x x = 7.5
12. 40 × 0.75 = 30 % of men earn > 25,000 a year.
Therefore, the percentage marks in each paper is 45
So percentage of women earning > 25,000
%, 52.5 %, 60 %, 67.5 % and 75 % respectively. So,
= 45 – 30 = 15%.
in 4 papers he scored than 50 % marks. Hence, [3].
Total percentage of women = 60%.
15 1
9. Let the total population be x. Then the amount re- So fraction of women earning > 25000 = = .
60 4
ceived right now is: 0.6x × 600 = 360x which is 75% Hence, [2].
of the amount. Therefore, the total amount required
is 480x. Hence, the remaining amount = 480x – 360x 13. Let M.P. = Rs.100
= 120x. Since discount is 15%, S.P. – 100 15 – 85
120x Let the C.P. be Rs. ‘x’
So, required contribution per head = = 300.
0.4x As the profit percentage is 2
Hence, [1].
85 - x
2= × 100
10. Total weight of fresh grapes = 20 kg x
20 × 10 2x = 8500 – 100x
Weight of the solid part = = 2 kg
100  102x = 8500
In dried grapes, water is 20%, so solid part is 80% 250
2  x =
Therefore, total weight of dried grapes = × 100 = 3
2.5kg
80 Now, let us suppose that S.P. is Rs. ‘y’. To ensure prof-
Hence, [3]. it is 20%.
250
y-
11. Let the original price be A 3
20 =
Also let x% = P 250
Now, 3
 A – A(1 – P)(1 + P) = 441 5000 25000
 A – A(1 – P)2 = 441 = 100y –
3 3
 AP2 = 441 ......... (I) 30000
Also, A[(1 + P)(1 – P)]2 = 1944.81  100y =
3
A[(1 – P)2]2 = 1944.81 ......... (I) 100y = 10000
Dividing (II) by (I) we get y = 100
^1 - P2h2
So, the seller should sell at the marked price or retail
=
441
= 4.41 price to ensure a profit of 20%. Hence, [4].
2
P 1944.81
14. Let us assume that the total production cost is Rs.
1 - P2 100.
= 2.1
P
So, component A’s share in this would be Rs. 30 and
1 – P2 = 2.1P
that of B would be Rs. 50.
10 – 10P2 = 21P
Other expense s = 100 – (30 + 50) = Rs 20
10P2 + 21P – 10 = 0
The product is currently sold at 20% profit = Rs. 120.

17
CATapult
ARITHMETIC

Now due to change in international scenario, Therefore, number of final voters with P = 40 – 6 + 15
Cost of component A = 1.3 × 30 = 39 = 49 votes
Cost of component B = 1.22 × 50 = Rs. 61. Number of votes with Q = 60 – 15 + 6 = 51 votes.
Therefore, the total cost of production of the product Therefore, margin of victory for Q = (51 – 49) = 2.
= (39 + 61 + 20) = Rs. 120 (Note that no change has Hence, there were 100 voters in all. Hence, [1].
been indicated in other expenses.)
It is further said that selling price cannot be increased 20. Let the present ages of Arun and Barun be 4x and 10x
beyond 10%. Therefore, the maximum selling price respectively.
can be Rs. 132. 4x + y 1
12  =
Therefore, the maximum gain = = 10%. 10x + y 2
120 8x + 2y = 10x + y
Hence, [1].
y = 2x
15. The cost of component A = (1.2 × 30) = Rs. 36 and So, in ‘y’ years, Barun’s age increases from 10x to 12x
that of B will be (0.88 × 50) = Rs. 44 Percentage increase in Barun’s age
So the total cost of production = (36 + 44 + 20) = Rs. 12x - 10x 2x
= × 100 = × 100 = 20%
100 Since the selling price is not altered, i.e. Rs. 120, 10x 10x
the gain will be the same as the original one, i.e. 20%. Therefore, the required answer is 20.
Hence, [2].

16. The equations are as follows:

2000 b
x
l + 2000 b y l = 700 which can be
100 100
simplified to x + y = 35……………(1)

2000 b
x
l + 3000 b y l = 900which can be
100 100
simplified to 2x + 3y = 90………..(2)
On solving (1) & (2), we get, x = 15.
Hence, [2].

17. Let the price per metre of cloth be Re 100.


The shopkeeper buys 120 cm, but pays for only 100
cm. In other words, he buys 1.2m forRs. 100. So his
100
CP per meter = = 83.33 Rs.
1.2
Now he sells 80 cm but charges for 100 cm. In other
words, he sells 80 cm for Rs 100. On this, he offers
a 20% discount on cash payment. Therefore, the per
metre selling price = Rs 100.
100 - 83.33
Therefore, % Profit = × 100 = 20%
83.33
Hence, [1].

18. Assume that he hasRs. 100. In this, he can buy 50


oranges or 40 mangoes.
Therefore, the price of an orange is Rs 2 and that of a
mango is Rs 2.50.
Now if he decides to keep 10% of his money for taxi
fares, he would be left with Rs. 90.
Now if he buys 20 mangoes, he would spend Rs. 50
and will be left with Rs. 40, in which he can buy 20
oranges. Hence, [4].

19. Let there be 100 voters in all.


So initially there were 40 of these who promised to
vote for P, while 60 of them promised to vote for Q. On
the last day, (15% of 40) = 6 voters shifted their inter-
est from P to Q and (25% of 60) = 15 voters shifted
their interest from Q to P.

18
Chapter QA
RATIO, PROPORTION, AND VARIATION 1.6

QA-1.6 RATIO, PROPORTION AND VARIATION

PRACTICE EXERCISE 1 Total quantity of liquid in the container = 3x


361
 Required percentage = × 100 = 30.4%
1188
1. Let Aman’s current age be a years and Hence, [2].
Boman’s current age be b years.
a 3 5.
 =
b 4 Original Present
 4a – 3b = 0 Wages 22x 25x
i.e. 36a – 27b = 0 ...... (1) Labourers 15y 11y
Five years hence,
a+5 7 22x × 15y 6
= Ratio of total wages = =
b+5 9 25x × 11y 5
 9a – 7b = –10
i.e. 36a – 28b = –40 ...... (2) If the present bill is Rs.5000, the original was
(1) – (2) gives Rs.6000. Hence, [4].
3b 3 × 40
b = 40 and a = = = 30. Hence, [4]. 6. Let the incomes be 4x and 3x.
4 4
So, their expenses will be (4x – 600) and (3x – 600)
2. Let, pd and pg be the prices of diamond and gold re-
spectively.
Let, wd and wg be the weights of diamond and gold (dividendo)
respectively.
 x = 600
 pd = kwd2 and pg =  Their incomes are Rs.2400 and Rs.1800.
k, are constants of proportionality. Hence, [3].
Let w be the weight of each ring.
For ring with 3 gms diamond: 7. If V is the value, d is the diameter and t is the thick-
ness, V  d2t  V = Kd2t
9k + (w – 3) = a ..... (i)
Ratio of diameters = 4 : 3 and ratio of thickness t1 : t2
For ring with 4 gms diamond:
16k + (w – 4) = b ..... (ii) V1 K × 16 × t 1 4 16t 1 t1 9
 =  =  =
For ring with 5 gms diamond: V2 K × 9 × t2 1 9t2 t2 4
25k + (w – 5) = c ..... (iii)  Ratio = 9 : 4. Hence, [4].
Using (i) and (ii) : 7k – =b–a
8. The required ratio
Using (ii) and (iii) : 9k – =c–b
rb - 1 l r3
4 27
a+c 2
k= – b. 3 8 4rr
2 = ×
b l
2
Price of 1 gm diamond, 4 3 9
a+c 3
rr 4r
4
- 1
pd = k(1)2 = k = – b. Hence, [3].
2 19 4 19
= × = Hence, [3].
3. Suppose the quantities of the three types of apples to 8 5 10 .
be mixed are x, y and z respectively. 9. Let his marks be 100, 90, 80, 70 and 60 in the five
5x + 6y + 8z subjects. Therefore, he scored 400 marks in total.
 =7
x+y+z This constitutes only 60% of the total marks. Hence,
5
 5x + 6y + 8z = 7x + 7y +7z total marks = × 400 = 667, which incidentally is
3
 z = 2x + y
Therefore multiple answers are possible. Hence, [4]. the maximum marks in all 5 subjects. Since the total
marks in each subject is the same, hence maximum
4. Suppose the volume of each vessel = x 667
5x marks in each subject will be = 133. Out of this,
 Total quantity of water in the container = + 5
3x x 361 9 50% is the passing marks. In other words, to pass in
+ = x a subject he needs to score 66.5 marks. We can see
11 12 396
that only in one subject he scored less than this, viz.
60. Hence, he passed in 4 subjects. Hence, [3].

19
CATapult
ARITHMETIC

10. Let the marks in the subjects be P, M, B and C respec- get, a = 9x and b = 31x. In a pack of 52 cards, the only
tively. possible value for ‘b’ could be 31. Hence, [2].
Assuming maximum marks per subject to be 100.
B:C=5:7 16. A2 = k1B3 & B = k2C1/2
P:M=1:1
where k1,k2 are some constants.
and B : M = 3 : 2
 B : M : C = 15 : 10 : 21  B3 = k23 C3/2  A2 = k1k23C3/2
and P : B : M : C = 10 : 15 : 10 : 21
10x + 10x + 15x + 21x
 Average marks =  A = k C3/4
4
56x
= = 14x Now 4 = k × (81)3/4
4
 14x = 64 k= .
64 32
x= =  4.57
14 7 Now 3 = × (C)3/4
 10x  45, 15x  68.5 and 21x  96
Thus, the student has scored more than 50% in 2
subjects. Hence, [2].  (C)3/4 =

11. Total charge = fixed + variable


TC = m + kx; where m and k are constants. C =
 300 = m + k × 20
Also, 550 = m + k × 45
b9l
8/3 8
Solving for m and k; we get m = 100; k = 10. = = (4.5) 3
Hence, for 100 grams, total charge 2
= 100 + 10 × 100 = Rs.1100. Hence, [3]. Hence, [3].
12. Let R = k1 + k2n 17. Since the number of coins are in the ratio 2.5 : 3 : 4,
[R is rent, n is number of passengers and k1, k2 are the values of the coins will be in the ratio (1 × 2.5) :
some constants] (0.5 × 3) : (0.25 × 4) = 2.5 : 1.5 : 1 or 5 : 3 : 2.
 2200 = k1 + 10k2 ---------------- (1) Total Amount = Rs 210. Let the value of each type of
also 3400 = k1 + 20k2 -------------- (2) coin be Rs 5x, 3x, 2x. The average value per coin =
(2) – (1)  k2 = 120 210
 by (1), .
10x 210
k1 = 1000. Hence, [3]. So, the value of 1 Rupee coins will be 5x × = Rs
10x
105
13. Equation will be of the form: 700 × 25 = F + 25V and So, the total number of 1 rupee coins will be 105.
600 × 50 = F + 50V. Hence, [4].
Solving we get V = 500, F = 5000.
For 100 boarders total cost 18. Let the original weight of the diamond be 10x. Hence,
= 5000 + 50000 = 55000. its original price will be k(100x2) . . . where k is a con-
55000
Average expenses per boarder = = 550. stant. The weights of the pieces after breaking are x,
100 2x, 3x and 4x. Therefore, their prices will be kx2, 4kx2,
Hence, [1]. 9kx2 and 16kx2. So the total price of the pieces = (1 +
4 + 9 + 16)kx2 = 30kx2.
= 3 bx – l.
x x Hence, the difference in the price of the original di-
14. If x < y, y –
2 2 amond and its pieces = 100kx2 – 30kx2 = 70kx2 =
x 2 70000. Hence, kx2 = 1000 and the original price =
Therefore, = 100kx2 = 100 × 1000 = 100000 = Rs. 1 lakh.
y 1
Hence, [1]. Hence, [3].

15. Let the number of cards with Mushtaq be ‘a’ and let Answer for questions 19 and 20.
the number of cards with Iqbal be ‘b’. If Mushtaq
gives Iqbal x cards, then we have the equation : b + Let x be the free luggage allowance and Rs. y be the fixed
x = 4(a – x) i.e. 4a – b = 5x. Conversely, if Iqbal gives rate per kg for excess luggage.
Mushtaq x cards, then we have the equation : b – x = Let Raja carry r kgs and Praja carry p kgs  r + p = 60
3(a + x) i.e. b – 3a = 4x. Solving the two equations we Then (r – x)y = 1200
(p – x)y = 2400,

20
Chapter QA
RATIO, PROPORTION, AND VARIATION 1.6

r–x 1 3
and (60 – x)y = 5400   5. Suresh’s inheritance = × 70000
p–x 2 3+6+5
 2r – 2x = p – x 2r – p = x .....(1) = Rs.15000.
p–x 6
2400 4 Dinesh’s inheritance = × 70000
= = 3+6+5
60 – x 5400 9
= Rs.30000.
 9p – 9x = 240 – 4x  9p – 5x = 240 ..... (2) 5
From (1) and (2) Hemant’s inheritance = × 70000
3+6+5
9p – 5(2r – p) = 240  9p – 10r + 5p = 240 = Rs.25000.
 14p – 10r = 240  7p – 5r = 120 ..... (3) 2
r + p = 60 Suresh’s investment = × 15000
3
Solving simultaneously, = Rs.10000.
p = 35 and r = 25 2
Substituting in [1], Dinesh’s investment = × 30000
5
2(25) – 35 = x  x = 15 kgs.
= Rs.12000.
13
19. Hence, [3]. Hemant’s investment = × 25000
25
= Rs.13000.
20. Hence, [2].
Suresh’s contribution
10000 10 2
= = = .
10000 + 12000 + 13000 35 7
PRACTICE EXERCISE 2
Hence, [1].
a a b c 1 1 1
1. = × × = ×2× = Answers to questions 6 to 8:
d b c d 3 2 3
b b c d 1
= × × =2× ×3=3 The best way to solve this sum is to work backwards. Let us
e c d e 2
assume that Alphonso’s total property was of Rs.x.
c c d e 1 1 3
= × × = ×3× =
f d e f 2 4 8
Widow Mother Ben Carl Dave Total
abc a b c 1 3 3 Share
= × × = ×3× = .
def d e f 3 8 8
Alphonso x/2 x/6 x/6 x/6 x
Hence, [1].
Ben x/12 x/24 x/24 x/6
a b c a+b+c Carl 5x/48 5x/48 5x/24
2. = = = =r
b+c c+a a+b 2 (a + b + c) Dave 15x/96 15x/96 15x/48
1
if a + b + c  0, then r = 6. Since Alphonso’s wife is also the mother of Dave, the
2
a b c x 15x 63x
If a + b + c = 0; = = = r = total share of this lady would be + = .
b+c c+a a+b 2 96 96
– (b + c)
= –1 And this share is equal to 1,575,000. So x = 2400000
b+c or 24 lakhs. This is the worth of the total property.
1 Hence, [4].
Hence, or –1. Hence, [3].
2
5x 24
7. Carl’s total share was =5× = 5 lakhs.Hence,
3. Let the minimum runs to be scored by the fourth bats- [4].
24 24
man be x.
x
 × 20000 = 15000 8. The ratio’s of the property’s owned by the widows of
(10 5 + 5 + x)
+
1 5 15
 x = 60. Hence, [4]. the 3 sons = : : = 8:10:15.
12 48 96
Hence, [2].
4. 1 woman’s share = Share of 3 boys
1 man’s share = Share of 3 boys + Share of 1 boy
 7 men, 11 women and 5 boys will be equivalent
to 28 + 33 + 5 = 66 boys
Share of 66 boys = Rs.79.20
Share of 1 boy = Rs.1.20
Share of 1 man = Rs.4.80. Hence, [3].

21
CATapult
ARITHMETIC

For answers to questions 9 and 10: 7 : 1. So first should receive 7 coins.


Hence [2].
Using the given data, we can draw the following table.
16. A’s Monthly Rent : B’s Monthly Rent : C’s Monthly Rent
= 22 × 4 : 16 × 8 : 32 × 6
= 11 : 16 : 24 (Dividing by 8)
C’s monthly rent = Rs.600
24
× Total rent = 600
Now, all the questions can be answered. 11 + 16 + 24
24
 × Total rent = 600
Female white collar employees 3.2x 16 51
9. = = . 600 × 51
Male blue collar employees 2.2x 11  Total rent = = Rs.1275.
24
Hence, [1]. Hence, [3].

10.  0.8x = 48  x = 60 17. 85% of the total profit is divided in the ratio 2 : 10 : 5.
 Required difference = 4.8x – 48 85% of total profit = 2x + 10x + 5x = 17x
= 4.8 × 60 – 48 = 240. Hence, [2]. 17x
Total profit = = 20x
85%
11. Let bag 1 have x mangoes and bag 2 have y mangoes.  Share of Ram in the profit
x mangoes y mangoes = 2x + 15% of 20x = 2x + 3x = 5x
x x Share of Shyam in the profit = 10x
Step 1 y+ Share of Ghanshyam in the profit = 5x
2 2
Now, (10x + 5x) – 5x = 500
x x
But :y+ =y:x 10x = 500  x = 50
2 2  Total profit = 20 × 50 = Rs.1000.
Step 2
2
y+
2
x
1
y +
1
x Hence, [1].
3 3 3 3
18. Let the total profit be Rs.x
2c m
2 2 x+y  0.18x – 3500 = 100
y+ x
3 3 3  0.18x = 3600
 Required ratio = = = 2 : 1.
 x = 20000.
1c m
1 1 x+y
y+ x Hence, [3].
Hence, [1]. 3 3 3
19. Let Ram invest Rs.5x and let Seeta invest Rs.6x.
Mass
12. Volume = After 3 months.
Density
4 5 7 Ram withdrew Rs.x and Seeta withdrew Rs.x
 Ratio of volumes = : : = 2 : 5 : 14  For Ram: capital × period = 5x × 3 + 4x × 9 = 51x
4 2 1
For Seeta: capital × period = 6x × 3 + 5x × 9 = 63x.
2
 Volume of first oil = × 1050 = 100 lit. 63x
Hence, [1].
21  Seeta’s share = × 22800 = 12600.
114x
Hence, [1].
1 1 1
13. The ratio is : : i.e. 18 : 3 : 2 20. Let the period of investments of A, B, C, D be tA, tB, tC
3 18 27
and tD respectively.
Now 18x + 3x + 2x = 345  7tA : 12tB : 14tC : 10tD
 x = 15  18x = 270. Hence, [2].
= 14 : 6 : 28 : 30.
1
14. The percentage of people below 35 years of age =  tA : tB : tC : tD = 2 : : 2 : 3.
2
64.75 1
Therefore, total number of people = 617.76 million tB 1
0.96 = 2 = .
Total number of females = 617.76 × 0.3 × = tD 3 6
1.96
90.8 million. Hence [2].  tD = 6 × tB = 6 × 2 = 12 years.
Hence, [3].
5+3 8
15. Each eats = loaves. Ratio of money that
3 3
8 8 7 1
they should share is 5 – :3– = : or
3 3 3 3

22
Chapter QA
MEANS AND WEIGHTED AVERAGES 1.7

QA-1.7 MEANS AND WEIGHTED AVERAGES

PRACTICE EXERCISE 1 7. Total marks scored by the student in 10 papers =


(80 × 10) = 800. If we exclude the papers in which
1. Arranging in ascending order, he scored the highest and the lowest marks, then the
21, 25, 25, 25, 26, 27, 28, 28, 29, 30 total marks scored by him in remaining 8 papers = (81
The number 25 appears 3 times × 8) = 648. Hence, his total in these two papers in
 The mode is 25 which he scored the highest and the lowest marks =
26 + 27 (800 – 648) = 152. Since his highest is 92, his lowest
The median is = 26.5 will be 152 – 92 = 60. Hence, [2].
2
Hence, [2].
8. Let x be the total number of students and S be the
2. (GM) = AM × HM sum of scores of all the students excluding the high-
2
est score. Average marks lost = 13, therefore, average
2
(GM) 6 marks scored = 7.
 HM = = = 4.5 Therefore,
AM 8
Hence, [3]. S = 7(x – 1)
Minimum marks lost by any student is 3. Therefore,
3. Let the two numbers be a & b. the highest score = 17.
2ab Therefore,
 Harmonic mean = and S + 17 = 8x
a+b Eliminating S, we have
7x + 10 = 8x
Geometric mean = ab Therefore, x = 10.
Ratio =
HM
=
2ab
×
1
=
12 Hence, [4].
GM a+b ab 13
9. Let the numbers be A, A+1, A+2, and so on.
2
ab 12 Therefore, (5A+10)/5 = A + 2 = n
 =
a+b 13 and (7A + 21)/7 = A + 3 = n + 1. Hence, [2].

ab 6 10. Let the three numbers be a, b and 15 such that a is


 =
a+b 13 the smallest and 15 is the largest.
Using option [3] we get a = 4 & b = 9 a + 15
b= , i.e., a + 15 = 2b ...... (1)
2
ab 4×9 6
 LHS = = = = RHS. Hence, [3]. It is also given that a + b = 15 ...... (2)
+
a b +
4 9 13  b = 10 and a = 5. Hence, [1].
4. Although the number of students in group D is more
than in any other group, we still cannot say anything 11. Here, n = 20, x = 65
about the average weight of this group as nothing is Rxi
mentioned about the average weights of any of the x =  Rxi = x .n = 20 × 65 = 1300
n
groups or of individual students. Hence, [4].
 Correct = (Incorrect di) – (Incorrect values)
5. Although one student is shifted from group A to group + (Correct values)
B, the number of students in the class and the total = 1300 – 96 + 69 = 1273
weight of the students remain the same. Therefore, 1273
Correct Mean = = 63.65.
the average weight of the class remains the same Hence, [1].
20
Hence, [3].
12. (10b + a) – (10a + b) = 1.8 × 10. So b – a = 2.
6. The total weight of any group will vary according to Hence, [2].
the number of students in that group. Hence, the total
weight of group A and C which has (15 + 25) = 40 will 13. The total marks of 60 students = 60 × 90 = 5400
be twice that of students in group B which has 20 stu- The total marks of 55 students = 55 × 85 = 4675
dents. However, it is clear that if all the students are Total of top 5 students = 725
of same weight, then the average weight of all groups The second highest is < 140 i.e < 139
remains same irrespective of how many students are ( the marks are integers)
present in each group. Hence, clearly the statement 3 If x is the top score, then
is false. Hence, [3]. 725 < x + 139 + 139 + 139 + 139
725 < x + 556

23
CATapult
ARITHMETIC

 x > 169 a+b+c


 minimum score of the topper can be 169. But AM = which cannot be found out.
3
Hence, [2].
Thus, III is false.
Hence, [2].
14. Average marks in 3 subjects = 48
Out of these, I got 64 in one subject. 20. I. Average of a set of numbers is greater than the
So in the other two subjects, I must have scored 48 × smallest and smaller than the greatest number
3 – 64 = 80 of the set. Thus, I is true.
For minimum number of subjects in which I failed, II. Consider five numbers a, b, c, d and e whose
I need to score minimum 70 marks (2 × 35 = 70) in a+b+c+d+e
these subjects. average is .
5
But since I have scored 80 I can pass in these 3 sub-
jects. Now if each of them is increased by k then we
Now, average of all 7 subjects = 40 have average
 Average of the other 4 subjects
7 × 40 - 3 × 48 =
= = 34
4
For minimum number of subjects, I must have failed a+b+c+d+e 5k
= + = old average + k.
only in one subject and passed in the other three sub- 5 5
jects. Hence, [1]. Thus, II is true.
III. Consider three numbers x, y and z whose aver-
15. Let the actual marks be 5x, 6x, 7x and 8x. Out of these x+y+z
age is .
only 5x and 8x could be equal to 80. 3
But if 5x = 80, then x = 16 and 7x = 112 which is not Now if each of them is multiplied by m then
possible. xm + ym + zm
 8x = 80 and x = 10. the new average =
Hence the actual marks are 50, 60, 70 and 80 and the 3
= mc m = m × old average.
average is 65. Hence, [1]. x+y+z
3
16. 24 men contribute Rs.480. Thus, III is true.
Let the contribution of ‘X’ be Rs.k. Hence, [4].
2 b l +3=k
480 + k
25
960 + 2k + 75 = 25k
PRACTICE EXERCISE 2
1035 = 23k 15 × 20 + 20 × 35 + 17 × 25
1035 1. Average wage =
 =k k = Rs.45 15 + 20 + 17
23 = Rs 27.4.
Hence, [4].
480 + k = 480 + 45 = Rs.525. Hence, [2].
Ratio of 2nd – Ratio of mixture
2. Ratio = (formula)
17. Increase after 5 years should have been 7 × 5 = 35 Ratio of mixture – Ratio of 1st
years. But since the average remains the same, dif- 1
ference in their age is 35 years. Hence, [2]. 13 1 2
= = : = 1 : 2 i.e.,
2 13 13
18. Total weight increases by 10 × 1.5 = 15 kg 13
Weight of the new man = 68 + 15 = 83 kg
Hence, [4].

19. Consider a set of three numbers a, b and c


a+b+c
I. AM =
3
3
Thus, a + b + c = 3AM. But GM = abc
which cannot be found out. Thus, I is false.
a+b+c
II. AM =
3
 Sum of the numbers = a + b + c = 3AM. Hence, [3].
Thus, II is true.
3
III. GM = abc .

24
Chapter QA
MEANS AND WEIGHTED AVERAGES 1.7

3. Therefore, quantity of A = 10x + 3y and quantity of B


= 2x + 9y
(Number) (Avg.Weight) Total Weight 10x + 3y
fi xi xfi In order for A : B to be 1 : 1, =1
2x + 9y
Men 30 60 1800 On solving, we get 8x = 6y or X : Y = 3 : 4
Women 25 50 1250 Hence, [4].
Children 15 40 600 7. Let him mix 3 kg, 4 kg and 5 kg of dry fruits at Rs. 100,
Total 70 3650 Rs. 80 and at Rs. 60 per kilogram respectively. Hence,
his effective cost of the dry fruits per kilogram should
Rx i f i be the weighted average
 Average weight of all =
fi = (3 × 100 + 4 × 80 + 5 × 60)/(3 + 4 + 5)
3650
=  52.14. 920
70 = .
12 920
Hence, [1]. In order to make a profit of 50%, × 1.5 = Rs
12
115/kg.
4. Let the cost price of tea be Rs.100. Then Hence, [4].
106 87

18 × 100
8. The cost of mixture = = Rs.10 per kg.
180
102 Ratio of mixing the cheap and dear types = 5 : 1
x 3x

15 : 4
15
 The required quantity is × 380 = 300 kg.
19 10
Hence, [2].

5. Since his SP of (spirit + water) = Rs.75/litre and he


ultimately makes a profit of 37.5%, his CP of (spirit +
water) = 75/1.375 = Rs.54.54. This should indeed be 3x – 10 : 10 – x
the weighted average of the costs of spirit and water.
So if we alligate, we can get the ratio of spirit : water 3x - 10 5
=
(assuming that cost of water is 0). 10 - x 1
 3x – 10 = 50 – 5x
 x = Rs.7.5.
Hence, [2].

9. 1st Solution 2nd Solution


30 45

45 – x : x – 30
Hence, [2].
45 - x x - 30
6. Let there be 12 L in Container 1 and Container 2 each.  =
1 2
Container 1: 12 L Container 2: 12 L  90 – 2x = x – 30
A = 10 L A=3L  x = 40.
Hence, [2].
B=2L B=9L
Suppose the 2 containers get mixed in the ratio x : y.

25
CATapult
ARITHMETIC

10. 40 × 0.75 = 30 % of men earn > 25,000 a year. So


percentage of women earning > 25,000
= 45 – 30 = 15%.
Total percentage of women = 60%.
15 1
So fraction earning > 25000 = = .
60 4 4 x – 30
Hence, [2]. i.e., =
3 30 – y
11. Let the capacity of each cup be 100 ml. So 300 ml of i.e., 3x + 4y = 210 ....(i)
alcohol is taken out from the first container and poured and
into the second one. So the first vessel will have 200
ml of alcohol and the second one will have 500 ml of
water and 300 ml of alcohol. So the ratio of water to
alcohol in the second vessel is 5 : 3. Hence, propor-
tion of alcohol in B = 3 : 8. Now if 300 ml of mixture is
removed from the second container, it will have 300 ×
3 x – 25
5 3 i.e., =
= 187.5 ml of water and 300 × = 112.5 ml of 4 25 – y
8 8
alcohol. Now if this mixture is poured in the second i.e., 4x + 3y = 175 ....(ii)
vessel, that vessel would have (200 + 112.5) = 312.5 Solving the equations (i) and (ii), simultaneously, we
ml of alcohol and 187.5 ml of water. Hence, ratio of get, y = 45 and
alcohol to water in this container = 312.5 : 187.5 = 5 : x = 10.
3. Hence, proportion of water = A = 3 : 8. Now again by alligation rule,
Therefore, we find that A = B.
Hence, [3].
Note: In any case, the amount of water in the first
container will be equal to the amount of alcohol in the
second container. Thus, their proportions in the two
containers will be equal. i.e., To get mixture with 27.5% alcohol, mixtures A
and B need to be mixed in the ratio 1 : 1.
12. Glucose on being sweetened by 100 times will have Hence, [4].
a sweetness of 74. Let x be the amount of sucrose
(to the nearest gram) added to 1 gram of saccharin 16. Suppose the volume of pure milk = x litres
(1 × 675 + x × 1)/(1 + x) > 74. 73x < 601  Water added = 0.2x litres
Solving, we get x = 8 (since x has to be an integer).  His CP = Rs.10x; Fine = Rs.200; His SP = 10(x +
Hence, [2]. 0.2x) = 12x
 (10x + 200) – 12x = 100
13. 1 g of glucose, 2 g of sucrose and 3 g of fructose will x = 50 litres
have a sweetness of (1*0.74 + 2*1 + 3*1.7) / 6 = 1.3. Volume of the mixture he sold = 1.2x = 1.2 × 50 = 60
Hence, [1]. litres. Hence, [4].

14. Let the no. of students in classes be x, y, z respectively 17. Let the capacity of the bucket be ‘a’ litre.
then.
ba –7l
2
25
83x + 76y = 79 (x + y)  =
a 36
 4x – 3y = 0
a–7 5
 x : y = 3 : 4 and  = !
76y + 85z = 81 (y + z) a 6
 5y – 4z = 0 42
y : z = 4 : 5.  a = 42 litre or litre.
11
Hence, x : y : z = 3 : 4 : 5 But a > 7 litre  a = 42 litre.
 x = 3k, y = 4k and z = 5k. Hence, [1].
Now, average for all the three classes is
(83 × 3k + 76 × 4k + 85 × 5k)/(3k + 4k + 5k) = 81.5. 18. If x litre of soda was stolen then
Hence, [2]. 5–x 9
=
3+ x 7
15. Let mixtures A and B contain x% and y% alcohol,
respectively.  35 – 7x = 27 + 9x
Then by alligation rule, we get,  16x = 8
 x = 0.5 litre.
Hence, [1].

26
Chapter QA
MEANS AND WEIGHTED AVERAGES 1.7

19. Variant I : A1 =
3
B , D = 2A1, C1 =
7
B PRACTICE EXERCISE 3
4 1 1 4 1
3
 = 1. Male Female
7
100 25
5
Variant II : = =
7
Also, C1 = C2
80
3 5
 + =  C2 = 35 gms.
7 7
 A1 = 15 gms.
 Quantity of D in one unit of variant I is 2 × 15 = 30 gms. 55 20
Hence, [1]. 11 : 4
Alternatively, Now 11x + 4x = 45
Let quantity of B in variant I be x gm. x=3
3x 7x 3x  number of female labourers per day
So, A1 = and C1 = and D1 = .
4 4 7 2 = 4 × 3 = 12.
Quantity of C in variant II = A2 Hence, [3].
3x 5
and A2 = A1 + 10 = + 10 Alternatively,
7 4 Let x, y be the number of male and female labourers
 C1 = A employed by the company.
5 2
From the given conditions,

7x
=
7
b 3x + 10 l  100x + 25y = 80(x + y)
4 5 4 and x + y = 45
7x 21x Solving the above equations, we get y = 12.
 = + 14 Hence, [3].
4 20
x = 20
2.
3
 Quantity of D = × 20 = 30 gm.
2
20. WI  Average weight of Section I
WII  Average weight of section II
WI + WII = 90 where WI <WII
Let weight of Deepak and Ponam be D and P kgs re- Ratio = 3 : 1
spectively 1
 Number of girls = × 400 = 100. Hence, [1].
50 × WI + D -P (3 + 1)
Then = WII
50 3. Let 15 kg of alloy A and 5 kg of alloy B be mixed.
Alloy A can have either 6 kg iron and 9 kg copper or 9
50 × WII - D + P
and = WI kg iron and 6 kg copper.
50
Alloy B can have either 4 kg copper and 1 kg zinc or 1
 50(WII – WI) = D – P
kg copper and 4 kg zinc.
Using Statement A alone:
 The different possibilities of the amount of iron
50 × 1 = D – P .... (i)
copper and zinc in the final mixture is as follows.
Thus D and P can take various values
So, Statement A alone is not sufficient.
Using Statement B alone:
50 × WI + D 50WII - D
= .... (ii)
51 49
Since values of WI and WII are not known
We cannot find the value of D
Combining both the statements,
values of WI and WII can be found and hence value of
D and P can be found, using (i) and (ii).
Hence, [3].

Hence, [2].

27
CATapult
ARITHMETIC

4. Let the cost price per kg of Tur dal, Masoor dal, Chana 484 + 56 540
dal and Urad dal be t, m, c and u respectively. And (New average) max = =
14 14
 t + 2m + 3u = 35 × 6 = 210 = 38.57 years. Hence, [2].
Also, m + 2c + 2u = 43 × 5 = 215
Adding the two equations, we get t + 3m + 2c + 5u = 425 9. Total age of the group = 49 × 20 = 980.
425 After two women leave and three join, the average
 Required answer = = Rs. 38.63 remains unchanged.
1 +3+2+5
Hence [3]. Let total age of three women be x.
5. Let the ages of the professor and assistant professor (49 × 20) - (45 + 50) + x
be ‘p’ and ‘a’ respectively. = 49
21
Let the average age of a student be ‘x’ and let the total
number of students in the class be ‘n’.  x = 144
 p + a + nx = 17(n + 2) ... (I) 144
p+a  Average = = 48 years.
3
Also, –x=n+2 Hence, [4].
2
Alternatively,
 p + a – 2x = 2n + 4 … (II)
The two women had total age equal to 95 which is 3
Subtracting (II) from (I), we get nx + 2x = 15n + 30
less than the average contribution. So the 3 women
 x(n + 2) = 15(n + 2)
who joined should also have total contribution 3 less
 x = 15
than their average contribution. Hence, their average
Therefore, the required answer is 15 years. 3
Hence [2]. is less than the average of the group by =1
3
 average age of the 3 women
6. Let the present ages of the man, his wife, their son = 49 – 1 = 48 years.
and their daughter be ‘m’ years, ‘w’ years, ‘s’ years
and ‘d’ years respectively. 10. Let the total number of employees be 12x.
]m – 10g + (w – 10)  Number of men = 7x and number of women = 5x.
 Number of male managers = 20% of 7x = 1.4x and
10 years ago, = 25
2 number of female managers = 35% of 5x = 1.75x
 m + w = 70  Total managers = 1.4x + 1.75x = 3.15x
(m - 5) + (w - 5) + (s - 5) 3.15
5 years ago, = 21  Required percentage = × 100 = 26.25%
3 12
Hence [2].
 70 – 10 + s – 5 = 63
s=8 11. Quantity of wheat in 400 kg of mixture
m+w+s+d = 0.6 × 400 = 240 kg
At present, = 20
4 Quantity of barley = 400 – 240 = 160 kg
 d = 80 – 70 – 8 = 2 Let, x kg of barley be added to 400 kg of the mixture
 Required age difference = 8 – 2 = 6 years. 1
53
Hence [3]. 240 3 160 8
= = =
160 x+ 2 140 7
7. Let the average of the seven men be x 46
3
10 × 6 + (x + 3)
x= 240 × 7 = 8 × 160 + 8x
7 x = 50 kg.
60 + x + 3 = 7x Hence, [2].
63
x= = 10.5
6
 Total amount = 10.5 × 7 = 73.5. Hence, [1].

8. Total age = 15 × 36
After the two left, total age = 15 × 36 – 2 × 28
= 540 – 56 = 484 years.
Let the age of the person who returns be x years.
484 + x
Then new average =
14
Now x lies between 20 and 36 (both inclusive)
484 + 20 504
 (New average) min = = = 36
14 14
years.

28
Chapter QA
MEANS AND WEIGHTED AVERAGES 1.7

Alternatively, Alternatively,
We can alligate on the % of wheat in the two mixtures. Let the capacity of cask be ‘a’.
In the mixture of wheat and barley, wheat = 60% After the first draw, quantity of wine left =a – 5.
In barley, wheat = 0% Quantity of wine a–5
 
Quantity of water a
 After the second draw, the ratio will be,
b a – 5 l = 361 = 361
2

a 361 + 39 400
a–5 19
 
a 20
 a = 100 litres

14.
160 20 Milk Water Total
= =
3 3 Initial Volume 10 4 14
160 20 (in litre)
Required ratio = : =8:1
3 3
1 Volume after boiling 10 1 7
Amount of barley = × 400 = 50 quintals. (in litre) 3+ 10 +
8 2 2

12. If the capacity is 12x, the cask contains 7x of A and 5x of B. 10 20


When 9 gallons are drawn off, it contains The required proportion is = .
7 27
Hence, [3]. 10 +
b7x - × 9 l of A and b5x × 9 l of B.
7 5 2
12
- 12 15.
When 9 gallons of B are added, the cask contains Alcohol Grape juice Alcohol : Grape

b7x - 21
l of A and b5x - 15
+ 9 l of B.
Tank 1 20% 80% 1:4
4 4 Tank 2 40% 60% 2:3
21
7x -
4 7 The ratio of grape juice to alcohol in half filled wine
 =
15 9 4 3
5x - 4
+9 + 3×

5 5 17
glass = =
This ratio is 7 : 9. x = 3 1
+ 3×
2 8

Capacity = 12x = 36 gallons. Hence, [1]. 5 5

13. Let the quantity of wine be a, i.e., the capacity of the So, the half capacity of the glass = 17 + 8 = 25
cask be ‘a’ So, full capacity = 25 × 2 = 50
Now rest 25 parts has to be filled with alcohol.
b 1- l
2
5
So the ratio of the grape juice to alcohol
Quantity of wine a
i.e., = = 17 : (8 + 25) = 17 : 33.
1 - b1 - l
2
Quantity of water 5 Hence, [4].
Alternatively,
b1 - 5 l
2 a
As the wine glass is filled to half by taking two parts
361 a
= 2 ;
from tank 1 and three parts from tank 2 and the sec-
1 - b1 - l
39 5 ond half is equivalent to five parts which is pure alch-
a olol, the percentage concentration of grape juice is:

b1 - l = b1 – l
361 361 5
2
5 2 2 × 80 + 3 × 60 + 5 × 0
– = 34%.
39 39 a a 2+3+5
Therefore, the ratio of grape juice to alcholol is 34 : 66
b1 - 5
l= 361
=
19 5
; =
1
= 17 : 33.
a 400 20 a 20
a = 100 litres. 16. The order of measures is A, B, C, A + B, B + C,
Hence, [3]. C + A, A + B + C.
Given A + B + C = 180
Hence, average of the 7 measures =
; (A) E
+ (B) + (C) + (A + B) + (B + C) + (C + A) + (A + B + C)
7

29
CATapult
ARITHMETIC

4 4 720
= (A + B + C) = × 180 = kgs. 19. Let the number that has not been considered be p.
7 7 7
Hence, [3].

17. In 240 litres of adulterated milk, amount of pure milk 9 285


 = 12 =
is 225 litres and amount of water is 15 litres. 23 23
 He is selling 240 litres for the cost of 225 litres. n – 1 could be 23 or a multiple of 23.
240 - 225 15 Let n – 1 = 23  n = 24
 Profit percent = × 100 = × 100
1 225 225 24 (25)
= × 100.  – p = 285  300 – p = 285  p = 15.
15 2
Since profit percent needs to be halved, the profit Hence, [2].
1
percent should be × 100.
30 Wine left
20. Stage
Let the amount of pure milk to be added be x. Original quantity
15 1 10
 × 100 = × 100 0
225 + x 30 10
 x = 225 litres 8
1
Hence, [3]. 10
Alternatively, 8 6
2 ×
If water is mixed into pure milk, the profit % is equal 10 10
to the % of water in the mixture as water is obtained 8 6 4
at free of cost. 3 × ×
10 10 10
To have the profit %, the % of water is to be halved in
8 6 4 2 4×3×2×1 4!
the mixture. 4 × × ×  4 = 4
This can be achieved by doubling the amount of milk 10 10 10 10 5 5
as the amount of water is not changed.
1
In this case, amount of water = × 240 = 15 Wine
1 + 15  = =
litres Water
 Amount of milk = 240 – 15 = 225 litres.
To have the % profit 225 litres of milk should be add- Hence, [2].
ed to the mixture.

18. Cost price of milk = Rs.5/litre


100
Cost price of mixture = × 6 = Rs.4.5/litre
1
133
3
Quantity of water 5 – 4.5 0.5 1
 = = = i.e.,
Quantity of milk 4.5 – 0 4.5 9

 Milk : Water = 9 : 1.
Hence, [1].

30
Data Interpretation
&
Logical Reasoning
Chapter DI
BASICS OF DATA INTERPRETATION 1.1

DI-1.1 BASICS OF DATA INTERPRETATION


PRACTICE EXERCISE 1

Answers to questions 1 to 4:

1
Since D was ranked I in round 4, his amount in round 3 should have been 4 × 1024 = 256. Since C was ranked
1
II in round 4, his amount in round 3 should have been 2 = 512

Since B was ranked III in round 4, his amount in round 3 would have remained as 1024.
Since A was ranked IV in round 4, his amount in round 3 should have been 1024 + (1024 – 256) + (1024 – 512)
= 2304
We can do similar back calculations and find out how much each person had at the end of each round as follows:
Amount with each person
Round A B C D
4 1024 1024 1024 1024
3 2304 1024 512 256
2 2304 512 128 1152
1 1152 128 1664 1152
Initially 288 1568 1664 576
Using this, all the questions can be answered.

1. Hence, [1].
2. Hence, [2].
3. Hence, [1].
4. Hence, [4].

Answers to questions 5 to 8:
The robot can take the following paths:
Sr. No Path Number of components carried
1 A-B-E-G 460
2 A-B-E-F-G 560
3 A-B-F-G 420
4 A-B-D-F-G 500
5 A-B-D-E-G 540
6 A-B-D-E-F-G 640
7 C-A-B-E-G 530
8 C-A-B-E-F-G 630
9 C-A-B-F-G 490
10 C-A-B-D-F-G 570
11 C-A-B-D-E-G 610
12 C-A-B-D-E-F-G 710
13 C-D-F-G 400
14 C-D-E-G 440
15 C-D-E-F-G 540
33
CATapult
DATA INTERPRETATION

5. It can be seen that the robot carries fewer than or equal to 550 machine components along paths numbered
1, 3, 4, 5, 7, 9, 13, 14, and 15. Therefore, the required answer is 9.

6. It can be seen that the robot visits all the 7 storage locations along path 12 and carries 710 machine com-
ponents and visits only 4 locations along the path 13 and carries 400 components. Therefore, the required
answer is 710 - 400 = 310.

7. It can be seen that the robot carries exactly 540 components along paths 5 and 15. Therefore, the required
answer is 2.

8. From the table, we can see that path number 8 is the path through which maximum number of components
(630) can be carried such that it doesn’t involve the path between B and D. Therefore, the required answer
is 630.

Answers to questions 9 to 12:

Let x = number of books on poetry.


Then number of books on philosophy = 2 × 3500 – (x + 1800) by condition 3 i.e. 5200 – x
10x
Then total number of Marathi books on poetry = 100 by condition 6

By condition 5, Number of English books on philosophy = 70 × (5200 - x)


100

So, Number of Marathi books on philosophy = 30 × (5200 - x) = 10 × 10x by condition 2


100 100
Solving the equation x = 1200 and 5200 – x = 4000

34
Chapter DI
BASICS OF DATA INTERPRETATION 1.1

We update the table as follows.

9. Hence [4]

10. The total number of books on Dramatics cannot exceed 6000 and the number of Marathi books on Dramatics
cannot exceed 50% of 6000, that is, 3000. Hence, [2].

11. From the updated table, statement I is clearly true.


The sum of total number of Marathi books other than those on Dramatics is 5980. Hence by condition 1,
statement II is also true.
Hence, [3].

12. Let x be the number of books on dramatics, then y = 18600 + x.


Now 4000 < x < 6000
And ratio increases as x increases.
Thus min possible ratio = 4000 : 22600 = 20 and
113
max possible ratio is 6000 : 24600 = 30 30 = 1
123 > 150 5
hence option [1] and [4] may not hold
Max possible ratio = 30 30 = 1
123 < 120 4
Option [2] is clearly not true as maximum possible ratio is < 1
4
Hence [3].

35
CATapult
DATA INTERPRETATION

Answers to questions 13 to 16:


Suppose the sales of the company in 2017 was ‘100x’ and the expenses of the company in 2017
was ‘100y’. Therefore we have the following:

Since the company suffered losses in all the five regions of India in 2017, expenses in all the regions must have
been greater than the sales in all the regions.

North: 28y > 30x or y > 30 x or y > 1.07x


28

East: 21y > 20x or y > 20 or y > 0.952x


21
West: 25y > 25x or y > x

South: 17y > 15x or y > 15 x or y > 0.8823x


17

North-East: 9y > 10x or y > 10 or y > 1.1111x


9
This means that if y increases above 0.8823x, the company suffers a loss in South. If y further increases and
exceeds 0.952x, the company suffers a loss in East. If y further increases and exceeds x, the company suffers a
loss in West. If y further increases and exceeds 1.07x, the company suffers a loss in North. Finally if y exceeds
1.1111x, the company suffers a loss in North-East.

13. If the company suffers a loss in all the five regions of India, y > 1.1111x. If sales of the company in 2017
was 100, expenses must have been greater than 111.11. Therefore the minimum percent loss margin suffered
by the company in India
= 111.11 - 100 × 100 = 11.11%.
100
Hence [2]

14. From the explanation given above, it can be seen that the company starts making losses in the North-East
last i.e. only after y exceeds 1.1111 x. Therefore the percent loss margin made by the company in 2017 was
lowest in the North-East.
Hence [3]

36
Chapter DI
BASICS OF DATA INTERPRETATION 1.1

For A.15 and A.16

The company suffered a total loss margin of 50% in 2017. Suppose the total sales of the company in 2017 was
200. Therefore the total expense of the company in 2017 was 300.
Therefore we have,

Region Sales Expense Loss Percent Loss Margin

North 60 84 24
24
60 × 100 = 40%

East 40 63 23
23
40 × 100 = 57.50%

West 50 75 25
25
50 × 100 = 50%

South 30 51 21
21
30 × 100 = 70%
7
North-East 20 27 7
20 × 100 = 35%
Total 200 300

15. Hence [1]

16. It can be seen that the company suffered lowest percent loss margin in North-East in 2017. If in 2018, the
sales of the company in the North-East increases to 27, there will be a situation of break-even in the North-
East. Sales of the company in North-East in 2018 was 10% of total sales of the company in India. Therefore
the total sales of the company in India should be 27 = 270.
0.1
Required percent increase in the sales of the company = 270 - 200 × 100
200
= 70 × 100 = 35%
200
Hence, [2]

PRACTICE EXERCISE 2

We know that the price per kg of Pineapple in 2014 was Rs. 70. Using this, the price per kg of Pineapple can be
calculated for 2013, 2012 and 2011. From its price in 2011, the price per kg of the remaining fruits can be calculated
for 2011. Then, their prices for the remaining years can also be calculated. Thus, we get the final table as follows:
Price per kg in a Year
2011 2012 2013 2014
Apple 30 36 45 50
Mango 60 69 92 115
Pineapple 45 54 63 70
Guava 32 40 42 48
Strawberry 50 55 66 69

Using this, all the questions can be answered.

37
CATapult
DATA INTERPRETATION

1. Required average = 32 + 40 + 42 + 48 = 40.5. Hence [2]


4

2. Required percentage = 36 × 100 = 72%. Hence [3]


50

3. The maximum increase in the price per kg of Strawberry was in 2013 over 2012, that is, Rs. 11. Hence [1]

4. Percent increase in price per kg in 2014 over 2011 for:


Apple = 20 × 100 = 66.6%
30
Strawberry = 19 × 100 = 38%
50
Pineapple = 25 × 100 = 55.5%
45
Mango = 55 × 100 = 91.6%
60
Hence [4]

Answers to questions 5 to 8:
Using the information in the scatter plot, we can generate the following table (Note: Loss and % Loss are entered
as negative values in the table)

Now all the questions can be answered.

5. It can be seen that class C is the only class that makes more than 50% loss.
Hence [2].

6. Hence [4].

38
Chapter DI
BASICS OF DATA INTERPRETATION 1.1

7. Total income from FE = 6 + 7 + 1.5 + 5 = 19.5 Lakhs


Total income from SE = 6.5 + 7.5 + 4.5 + 1.5 = 20 Lakhs
Total income from TE = 5 + 5.5 + 2 + 5.5 = 18 Lakhs
Total income from BE = 3.5 + 5 + 1 + 4 = 13.5 Lakhs
Therefore, the average income is lowest on BE.
Hence [4].

8. From the explanatory answer to the previous question, we know the sum total of the income of the four
classes from each of the four years.
Total expenditure on FE = 3.5 + 5.5 + 3 + 5.5 = 17.5 Lakhs
Total expenditure on SE = 5 + 3 + 6 + 7.5 = 21.5 Lakhs
Total expenditure on TE = 7.5 + 1.5 + 5 + 3.5 = 17.5 Lakhs
Total expenditure on BE = 1 + 4.5 + 1 + 7.5 = 14 Lakhs
Therefore, the sum total of the income from two years is greater than the sum total of expenditure.
Hence [3].

9. Only New Zealand has never managed 3.5. All the others have, for instance England (2005), Australia (2001),
India (2009), Sri Lanka (2009), Pakistan (2005), South Africa (2003), West Indies (2004). Hence [4].

10. Australia have the highest average among the given countries in 2000, 2001, 2002, 2003, 2006 and 2007,
i.e. 6 times. Hence [3].

11. Only England (2009), Sri Lanka (2008 and 2009) and India (2008 and 2009) have managed to get a higher
RPO than Australia in any year. Hence [3].

12. This has been achieved 20 times (Australia – 7 times, India – 4 times, South Africa – 3 times, England and
Sri Lanka 2 twice each, Pakistan and New Zealand once each). Hence [3].

13. This has been achieved 16 times (Thrice each by South Africa, West Indies and New Zealand, twice each by
Sri Lanka, Pakistan and England, and once by India). Hence [1].

14. During the given period, both West Indies and New Zealand have scored the lowest average 4 times each.
Hence [4].

15. 4 teams i.e. England, Australia, Sri Lanka and South Africa averaged more than India in 2002. Hence [2].

16. 16 patterns were produced in red (All but a, g, m and q). Hence [3].

17. 16 patterns were produced for women in green and red (All but a, g, i, and m). Hence [2].

18. “Men, White” was produced in 17 patterns, the rest of the options in 16 patterns each. Hence [1].

19. There was excess production in 5 varieties: “Men, White”, “Men, Red”, “Women, Red”, “Children, Black”,
“Children, Green”. Hence [4].

39
CATapult
DATA INTERPRETATION

DI-1.2 CALCULATION BASED DI


PRACTICE EXERCISE

Answers to questions 1 to 4:
Based on the graph, we can list the temperature readings for each day as follows:

Using this, all questions can be answered.

1. Required average
= 32 + 30 + 31.5 + 33 + 31 + 29 + 29.5 + 28 + 30.5 + 31 + 33 + 31.5 + 32 + 34 + 34.5 = 31.37.
15
Hence, [2].

2. Maximum temperature increased consecutively for 3 days on 9-11 Apr and 13-15 Apr. Hence, [2].

3. Average of period 1 = 31.17


Average of period 2 = 31
Average of period 3 = 29.33
Average of period 4 = 31.83
Average of period 5 = 33.5
Hence, [3].

4. The sharpest increase in the maximum temperature was recorded on 9-Apr.


Hence, [4].

5. The number of runs scored by the batsman in 1999 = 8571 – 7728 = 843
The number of balls faced by the batsman in 1999 = 9875 – 8924 = 951
The number of fours hit by the batsman in 1999 = 854 – 770 = 84
The number of sixes hit by the batsman in 1999 = 111 – 97 = 14
Therefore, the number of runs scored by the batsman in 1999 in the form of fours and sixes = 14 × 6 +
84 × 4 = 420
Therefore, the number of runs scored by the batsman in 1999 in the form of singles and doubles = 843 –
420 = 423.
The number of balls faced by the batsman in 1999 in which he did not hit a four or a six = 951 – 14 – 84
= 853
The maximum number of dot balls will be obtained when he scores most of the 423 runs in the form of
doubles. He could have scored maximum 422 runs in the form of doubles or he could have faced maximum
= 211 balls in which he scored two runs each. In that case, he would have scored one run in the form of
a single in one ball.
Therefore, the maximum number of dot balls = 853 – 211 – 1 = 641
Therefore, the required answer is 641.

40
Chapter DI
CALCULATION BASED DI 1.2

6. We have the following for the different two-year periods:

We need to compare the averages for 1995-96 and 1998-99.


The average for 1995-96: 2055 = 50.12
44 - 3
The average for 1998-99: 2737 = 55.86
55 - 6
Therefore, the average is maximum for 1998-99.
Therefore, the required answer is 1998.

7. It would help to note that we need to consider only those years for which the cumulative strike rate is
higher than the previous year. It can be seen that in 1994, 1995, 1996, 1997, 1998, 1999 and 2001, the
cumulative strike rate is higher than in the previous years. Therefore, we need to consider only those years
for calculations.
We have,

It can be seen that the runs scored by the batsman in 1995 and 1998 were more than the number of balls
faced. Therefore, the strike rate in those two years was greater than 100. For all other years, the strike rate
was less than 100.
The strike rate in 1995: 444 × 100 = 101.37
438
The strike rate in 1998: 1894 × 100 = 102.16
1854
Also, the cumulative strike rate in 1990 was 97.95 and the cumulative strike rate in 1991 was 81.59. There-
fore, the strike rate in 1991 cannot be greater than 100.
Therefore, the required answer is 1998.

41
CATapult
DATA INTERPRETATION

8. If we want the RPO of the batsman to be greater than 5.4, his strike rate will have to be greater than 5.4
6
× 100 = 90
From the answer to the previous question, we have seen that the strike rate of the batsman in 1995 and
1998 was greater than 100. We have also calculated the number of runs scored and the number of balls
faced for a few other years in the answer to the previous question. From that,

We need to check for other years, namely 1989, 1990, 1991, 1992, 1993 and 2000.
In 1989, he played only one match and his strike rate was 0. By 1990, he had played 12 matches and his
cumulative strike rate was 97.95. Therefore, his strike rate in 1990 was definitely greater than 90. His cu-
mulative strike rate in 1991 was 81.59 (which was less than the lower limit) and his cumulative strike rate
up-to 1993 registered a decrease every year. Therefore, his strike rate in 1991, 1992 and 1993 cannot be
greater than 90.
We need to check only for the year 2000. The number of runs scored by him in 2000 = 9899 – 8571 =
1328, and the number of balls faced by him in 2000 = 11499 – 9875 = 1624. Therefore, his strike rate in
2000 = 1328 × 100 = 81.77, which does not lie in the required range.
1624
Therefore, the years in which the strike rate (and hence RPO) is in the required range are 1990, 1995, 1998
and 2001.
Therefore, the required answer is 4.

42
Chapter DI
CALCULATION BASED DI 1.2

9. We have,

It can be seen that the batsman hit maximum number of fours as well as sixes in 1998.
Therefore, the required answer is 1998.

10. The sum of the total international trade of these 10 countries for 2012 and 2014 is simply the sum of all
the entries corresponding to those two years.
Therefore total international trade in 2012 = 60 + 20 + 30 + 50 + 10 + 70 + 80 + 45 + 70 + 25 + 70 +
25 + 10 + 10 + 15 + 35 + 70 + 70 + 25 + 35 = 825
Similarly total international trade in 2014 = 80 + 50 + 80 + 45 + 45 + 30 + 50 + 30 + 20 + 55 + 75 + 25
+ 65 + 30 + 15 + 40 + 65 + 80 + 15 + 20 = 915
Therefore percent increase = 915 - 825 × 100 = 90 × 100.
825 825
This is between 10% and 11%.
Hence [2]

11. Total imports in 2014 = 80 + 80 + 45 + 50 + 20 + 75 + 65 + 15 + 65 + 15 = 510 million dollars


Total imports in 2015 = 25 + 35 + 80 + 70 + 80 + 50 + 20 + 70 + 15 + 75 = 520 million dollars
Therefore the required increase = 520 – 510 = 10 million dollars
Hence [1]

12. It can be seen that no country registered a consistent increase in exports in 2013, 2014 and 2015 over the
previous year.
Hence [1]

13. Trade surplus is the situation when export is greater than import. It can be seen that for all countries, export
in at least one year is at least equal to or less than imports. Therefore the required answer is 0.
Hence [1]

43
CATapult
DATA INTERPRETATION

14. Required ratio is: 600×10, 000, 000 6000 = 253.87


=
13, 000×0.9090×2, 000 26×0.9090
Hence [2].

15. We have,

Effectively we have to compare ratios 9 , 6 , 16 , 18 , 11 . It can be seen that 9 is between 80% and
11 13 11.5 13.5 18 11

90%, 6 is between 40% and 50%, 16 is almost 140%, 18 is 133.33% and 11 is between 60%
13 11.5 13.5 18
and 70%.
Therefore the required ascending order is B-E-A-D-C
Hence [3]

16. The required ratio is: 11500×2000×0.9440 = 115×944 = 23×59 = 1357 : 1566
13500×2000×0.9280 135×928 27×58
Hence [4].

44
Chapter DI
CALCULATION BASED DI 1.2

17. We have the following

Effectively we have to compare the following ratios:


9 16 18 11 or 9 16 18 11
110×0.9450 , 115×0.9440 , 135×0.9250 , 180×0.9760 103.56 , 108.56 , 124.875 , 175.68

or 1 1 1 1
11.XX , 6.XX , 6.XX , 15.XX
Clearly the required ratios for C and D are greater than that for A and E. We need to compare the ratios
for C and D closely.

and 16 = 1 18 = 1 . Therefore the ratio for C is greater than the ratio for D.
108.56 6.785 and 124.875 6.9375

Therefore the required descending order is C-D-A-E.


Hence [2].

45
CATapult
LOGICAL REASONING

LR-1.1 TYPES OF ARRANGEMENTS


PRACTICE EXERCISE 1

SET 1
Answers to questions 1 to 4:

Using the information given, we can generate the following cases:

Now all the questions can be answered.

1. Hence [1]
2. Hence [2]
3. Hence [3]
4. Hence [4].

SET 2
Answers to questions 5 to 8:

Using the information given, we can generate the following table:

Now all the questions can be answered.

5. Hence [4]
6. Hence [2]
7. Hence [3]
8. Hence [2]

46
Chapter LR
TYPES OF ARRANGEMENTS 1.1

SET 3
Answers to questions 9 to 11:
Position Col
1 2 3 4 5 6
Horse
A       Go
B       Bl
C       Pu
D     
E       Wh
F    
Col Gr Bl Go Wh ? Pu

Three horses finished between F and D. So they must have been either 1st and 5th, or 2nd and 6th. But the
latter alternative is impossible as B finished 2nd. So F and D finished 1st and 5th(not necessarily in that order).
We also know that the horse wearing yellow finished just ahead of C and just behind E. Hence the horse in yellow
cannot be in 1st or last position. It cannot be in 2nd or 4th position as these are already occupied by Blue and
White respectively. It also cannot be in 3rd position as it is just behind E and we know that B is in 2nd position.
Hence the yellow horse must be in 5th place, which also implies E is in 4th place (and wearing white) and C in
last place. By elimination, A can only be 3rd.
Now we can proceed to answer the questions:

9. The order has to be _ B A E _ C. Of the answer choices, only (iv) F B A E D C fits this.

10. If D was in yellow, it could not be in 1st place (as 1st is in grey). Hence F must have won. Hence (ii).

11. Clearly, B, A and E finished between F and D. Hence (iii)

SET 4

Answers to questions 12 to 15:

Using the information given, we can generate the following cases:

47
CATapult
LOGICAL REASONING

Now all the questions can be answered.

12. Hence [1]

13. Hence [2]

14. Hence [1]

15. Hence [3]

PRACTICE EXERCISE 2
SET 1
Answers to questions 1 and 2:

From statements (6) and (3), we get

From statements (5) and (4), we get

48
Chapter LR
TYPES OF ARRANGEMENTS 1.1

From statement (2), we get the following possibilities:

From statement (1), men and women are sitting alternately. Therefore, C, F and E all are of the same gender.
From statement (7), if the delegate from Britain is a woman, C, F and E all are women.
Now all the questions can be answered.

1. Therefore, the required answer is 3.

2. Therefore, the required answer is 6.

SET 2
Answers to questions 3 to 6:
Using condition (3), we get the following.

W M W

M
M

W W

M W M

49
CATapult
LOGICAL REASONING

From condition (2) and (4), we can see that exactly one Indian sits along each of the four sides of the table. From
conditions (6) and (2), we get the following:

W M W

M
M
Yuang(chinese) (Indian)
Hritik/
Ranveer

W W
(Indian) (American)
Kareena/ Janet/
Alia Heather
M W M

Now, one Indian, one American and one British have to be accommodated in each of the remaining sides. Using
other conditions, we get the following arrangement:

(Indian) (American)
Kareena/ Boris Janet/
Alia W M Heather W
(British)

M
M
Yuang(chinese) (Indian)
Hritik/
Ranveer

W W
(Indian) (American)
Kareena/ Janet/
Alia Heather
M W M
(American) (British) (Indian)
Alfred Margaret Hritik/
Ranveer

Now all the questions can be answered.

3. It can be seen that Alia can be seated opposite Janet, Heather or Alfred. Hence, [3].

4. It can be seen that either Janet or Heather are seated farthest to Yuang. Hence [4]

5. It can be seen that Boris and Margaret are facing each other. Hence [1]

6. Hence [2].

50
Chapter LR
TYPES OF ARRANGEMENTS 1.1

SET 3
Answers to questions 7 to 10:

From statements (5) and (6), we can deduce that Tushar is from Assam, who was ranked first and has specialized
in Operations. That means he is less than 25 years old.

From statement (4), Bhanu is ranked either 2nd or 3rd. That means from statements (2) and (7), Amarendra is
fifth and Mandar is sixth.

From statement (8), Chandrashekhar is from Maharashtra and was ranked 2nd or 3rd.

From statement (1), Mandar is from Rajasthan and has specialized in Marketing. That means Amarendra is from
Orissa and has specialized in Finance.

Therefore, from statements (2) and (3), Jayant is from Kerala and Bhanu is from Gujarat. The person ranked 3rd
i.e. Bhanu or Chandrashekhar has specialized in Human Resources.

Hence we can draw the matrix as follows:

Now all the questions can be answered.

7. Hence [1]

8. Hence [3]

9. Hence [3]

10. Hence [4]

51
CATapult
LOGICAL REASONING

SET 4
Answers to questions 11 to 14:
Information given is as follows

Since Sameer is an alumnus of NIM-C, Prakash is a businessman and Gayatri’s father is a consultant, we have

It can be seen that one of Sameer and Tushar is a Consultant and the other is a Marketing Professional. Prakash
is a businessman. Therefore, Rajesh is a banker and Gayatri is not his daughter. Therefore, we have

It can now be seen that Shweta is the daughter of Prakash, who is an alumnus of NIM-B.

52
Chapter LR
TYPES OF ARRANGEMENTS 1.1

Now all questions can be answered.

11. Hence [2]

12. Hence [2]

13. Hence [3]

14. Hence [4].

53
CATapult
LOGICAL REASONING

LR-1.2 CONDITIONALITIES & GROUPING


PRACTICE EXERCISE

1. Two professors, B and C from Physics department are selected. That means A and D are not selected.
According to condition 5, O must be selected. According to condition 1, S must not be selected. According
to condition 2, P must not be selected. Therefore the two professors from Mathematics department must be
Q and R. Therefore so far we have,

Since B and R have been selected, N cannot be selected according to condition 3. Therefore second professor
from Chemistry department can be L or M.
Therefore the members of the committee can be selected in two different ways. Therefore the required answer
is 2.

2. According to condition 3, exactly two of B, N and R must be selected. Since N and O are selected, exactly
one of B and R must be selected. Therefore we have the following possibilities:

Possibility 1:
The other professor from Physics department can be A, C or D. Accordingly we have the following possibilities:

Possibility 2:
Second professor from Mathematics can be P, Q or S. Accordingly we get the following possibilities:

54
Chapter LR
CONDITIONALITIES & GROUPING 1.2

Therefore total required number of ways = 2 + 1 + 1 + 1 = 5.


Therefore required answer is 5.

3. If P and R both are selected from Mathematics department, Q and S are not selected. Therefore D cannot
be selected (condition 4). Similarly A cannot be selected (condition 1) and C cannot be selected (condition
2). Therefore there is only one possible candidate from Physics department, which violates the requirement
of having at least two.
Therefore the required answer is 0.

4. If A and L both are selected, S must be selected (condition 1). Exactly two of B, N and R must be selected.
Therefore we have the following possibilities:

Therefore required total number of ways = 2 + 2 + 1 = 5.


Therefore required answer is 5.

5. Only Option [4] satisfies all conditions and is possible. Hence [4].

6. All of the given groups are possible. Hence [4].

7. Paul is present in the group. Therefore, Anna is also present in the group. The other woman in the group is
Cheryl.
Using condition 4, John cannot be present. The other two men are Shawn and Albert. If Albert is present,

55
CATapult
LOGICAL REASONING

then John has to be present, which violates our earlier conclusion. Therefore, no such group is possible.
Hence [1]

8. If Anna is present in the group, according to condition (1), Paul is also present. According to condition (5),
John has to be present in the group. It is given that Albert is present in the group and Shawn is not pres-
ent. According to condition (3), Terry cannot be present in the delegation. Since Shawn cannot be present,
the fourth man in the delegation has to be Roger. Therefore the four men present are Paul, John, Roger and
Albert.
Among women, it is given that Anna is present. Since Roger and John are present in the delegation, according
to condition (4), Cheryl cannot be present. The other woman in the delegation can be one of Brenda, Dana
or Heather.
Therefore the delegation can be selected in 3 different ways.
Hence [2]

9. If Mary has been selected, John cannot be selected. Also, Steve cannot be selected because he cannot serve
in a committee with any woman. The other two members of the committee can be selected from among
Chris, Bill, Janet and Kathy.
Hence [3]

10. If Janet and Kathy have been selected, Steve cannot be selected. The third member of the committee can
be selected from Mary, Chris, John and Bill.
Hence [4]

11. If Chris and Bill refuse to serve, at least one woman must be selected. Therefore, Steve cannot be selected.
As a result, the committee can have at most one man.
Therefore, we have the following possibilities:
i. Committee with all the three women: All of Janet, Mary and Kathy serve on the committee. This can be done
in only one way.
ii. Committee with two women and one man: The only man who can be selected is John. Therefore, Mary cannot
be selected. As a result, the other two women, Janet and Kathy, should be selected. This can be done in
only one way.
Therefore the committee can be selected in two ways.
Hence [2]

12. If John has been selected, Mary cannot be selected. If there is at least one woman on the committee, Steve
cannot be selected.
The other two members of the committee will have one man and one woman.
One man can be selected out of Chris and Bill. One woman can be selected out of Janet and Kathy.
Therefore, the total number of people is 4.
Hence [1].

13. If Bill and Vishal ride together in the same car, the other two cars must be driven by Vineeta and Peggy. The
three Sharma brothers ride in different cars. Therefore, the two Hamilton sisters will have to ride in different
cars.
Hence [3].

56
Chapter LR
CONDITIONALITIES & GROUPING 1.2

14. If Peggy and Chelsea are together, Barbara and Bill have to travel in different cars because at least one
member of each family must be present in each car. Therefore, options [1] and [4] are ruled out. Vikram,
Barbara and Varun cannot travel together in the same car because at least one adult has to be present in
the car. Therefore, option [3] is also ruled out.
Hence [2]

15. If Bill and Vineeta are together in one car, the other two cars must be driven by Peggy and Vishal. Therefore,
they cannot travel in the same car. Hence [2]

16. If the three Sharma children travel in different cars, out of four parents, the Sharma parents cannot be to-
gether in the same car. This is because in that case one car will have no member from the Hamilton family.
However, Hamilton parents can travel together in the same car. Hence [2]

57
Verbal Ability
CATapult
VOCABULARY

VA-1.2 QUESTION TYPES


PRACTICE EXERCISE 1 10. In sentence (3), the phrase should be ‘brought’
and not ‘bought’ to book. Hence, (3).

1. ‘Rudimentary’ means ‘very basic or elementary’. 11. A speaker can ‘imply’ (indicate or suggest) something
Its opposite is ‘developed’; hence B-E. ‘Lethargic’ and the listener can ‘infer’ (derive by reasoning or
means ‘lazy or sluggish’; ‘fortified’ means ‘defend- conclude) the intended meaning from the speaker’s
ed’, and ‘sincere’ means ‘honest’. Hence, (2). words or tone. In sentence (1) we have to use
‘implied’. ‘Climactic’ deals with a climax, ‘climatic’
2. ‘Dolorous’ means ‘mournful’ or ‘causing pain or deals with the weather. Hence, we have to use
sorrow’. ‘Cheerful’, which means ‘happy’ is op- ‘climactic’ in (2). ‘Loath’ means ‘reluctant’, whereas
posite to the word ‘dolorous’; so A-D. ‘Astute’ ‘loathe’ means ‘to dislike intensely’. In sentence
means ‘clever or discerning’; ‘corrupted’ means (3) we have to use ‘loath’. A person who proves
‘contaminated or polluted’ and ‘convenient’ means his or her ‘mettle’ displays courage or stamina.
‘favourable’. Hence, (3). This has nothing to with ‘metals’, i.e., gold, silver,
3. ‘Conscientious’ means ‘meticulous’ or ‘scrupulous’; copper, etc. In sentence (4) the appropriate word
hence B-E. ‘Festering’ means ‘rotting’; ‘hypothetical’ is ‘mettle’. The correct sequence is AABB. Hence,
means ‘assumed’; and ‘serrated’ means ‘ragged or (3).
indented’. Hence, (2). 12. ‘Miners’ are ‘people working in a mine’, while a
4. ‘Prognosis’ means ‘a forecast or a speculation’. ‘minor’ is ‘a child’. In (1) we have to use ‘miners’.
‘Prediction’ is similar in meaning to ‘prognosis’; An ‘award’ is handed out in recognition of one’s
hence B-E. ‘Aesthetics’ refers to ‘artistic taste’; achievements. A ‘reward’ is given out when some-
‘deluge’ refers to ‘flood’; and ‘vogue’ refers to one finds a missing item or person. So, in (2), the
‘current trend or fashion’. Hence, (2). correct word is ‘reward’. ‘Proscribe’ means ‘to ban’,
while ‘prescribe’ means ‘to recommend the use of’.
5. ‘Nefarious’ means ‘extremely wicked or sinful’.
In sentence (3) we have to use ‘proscribe’. ‘Noise’
‘Respectable’, meaning ‘worthy of respect’ or
refers to ‘a harsh or unwanted sound’. Since the
‘admirable’, is opposite in meaning to ‘nefarious’;
audience enjoyed the guitar, we should use ‘sound’.
thus the C-D pair. ‘Inadvertent’ means ‘accidental’;
The correct sequence is BAAA. Hence, (1).
‘comprehensive’ means ‘having an extensive range’
and ‘affluent’ means ‘wealthy’. Hence, (2). 13. ‘Grisly’ means ‘horrible’; ‘grizzly’ means ‘greyish’.
In (1) we have to use ‘grisly’. A speech that is
6. ‘To take off after’ someone means ‘to chase them
‘heartening’ gives you courage or confidence while
down’. ‘Take after’ someone means ‘to resemble
one that is ‘heart-rending’ makes you feel terribly
someone in appearance or character’. Sentence
sad. Hence in (2), ‘heartening’ is correct. You ‘install’
(1) should read, ‘She is a good singer; she takes
equipment, while you ‘instil’ feelings or attitudes.
after her aunt in that respect’. Hence, (1).
In sentence (3), ‘instil’ is correct. A free item is
7. When you ‘stand off’ someone you ‘prevent him a ‘complimentary’ gift, but items or people that
or her from attacking you’ or ‘you avoid someone’. go well with each other are ‘complementary’. The
When you ‘stand over’ something you ‘postpone word we should use in (4) is ‘complimentary’. The
it’. The sentence should read, ‘We must stand correct sequence is BBBA. Hence, (3).
over the deal, we can always complete it later’.
14. The literal meaning of a word is its ’denotation’;
Hence, (2).
the broader associations we have with a word are
8. ‘Make-believe’ is a singular word and is used with its ’connotations’. As two words are being linked
a hyphen. It refers to an imaginary scenario. Hence, to ‘failure’, their broader associations must be
(1). referred to. In (1) we have to use ‘connotation’.
9. Sentence (4) should read, ‘He took a lot of con- ‘Continuous’ refers to actions that are uninterrupted.
vincing, but eventually he gave way’, meaning after ‘Continual’ actions are not continuous; they are
a lot of convincing he yielded. Hence, (4).

60
Chapter VA
QUESTION TYPES 1.2

only repeated periodically. Hence, we should use 15. Fell : ADJ. cruel; deadly and dangerous
‘continually’ in (2). An official group that deliber-
16. Smart : V. cause or feel a sharp pain
ates, like the Council on Foreign Relations, is a
‘council’; all the rest are ‘counsels’: your lawyer, 17. Latitude : N. freedom from rules or limitations
advice, etc. In (3) we have to use ‘counsel’. ‘Gaffe’ 18. Wax : V. grow; increase
means ‘an embarrassing mistake’, and should not
19. Base : ADJ. contemptible; morally corrupt; inferior
be mixed up with ‘gaff’, which means ‘a hook used
in quality or value
for landing large fish’. In (4) ‘gaffe’ is correct. The
correct sequence is AAAB. Hence, (4). 20. Rank : ADJ. offensive in odour or flavour; decaying
15. ‘Jibe’ is ‘a taunting or sarcastic remark’ while ‘jive’ 21. An ‘altercation’, meaning ‘a heated or angry dis-
is ‘a genre of music and dance’. In (1) we have pute’, can result in a criminal case. An ‘alteration’
to use ‘jibe’. ‘Defective’ means ‘faulty’, ‘deficient’ refers to ‘a change or modification’.
means ‘lacking in something’. In (2) we have to
An object that isn’t moving is ‘stationary’. A ‘sta-
use ‘defective’. You ‘defuse’ a dangerous situation
tionery’ shop stocks writing material.
by treating it like a bomb and removing its fuse;
to ‘diffuse’, in contrast, is ‘to spread something’. ‘Batten down the hatches’ is an idiomatic expres-
In (3) we have to use ‘defuse’. A ‘descent’ is ‘a sion, which means ‘to prepare for an emergency’.
downward slope’. ‘Dissent’ means ‘disagreement’. A ‘baton’ is a ‘small rod’.
In (4) we have to use ‘descent’. The correct se- To ‘ascribe’ is ‘to attribute something to someone’.
quence is BAAA. Hence, (3). To ‘subscribe’ is ‘to agree to something’ or ‘to
pledge for something’.
Thus, the correct sequence is ABAB. Hence, (1).
PRACTICE EXERCISE 2
22. ‘Alumnus’ is singular, ‘alumni’ is plural. Here the
plural form is required.
1. Default : N. failure to act or be present; failure to
‘Sensual’ relates to physical desires while ‘sensuous’
honour a promise or pay a debt
refers to aesthetic or artistic qualities in things
2. Brook : V. tolerate; endure like art or music or in this case architecture.
3. Catholic : ADJ. wide-ranging or broad; liberal; ‘Indifference’ refers to ‘an apathetic view on things’.
universal ‘Ambivalence’ means ‘a clash of two different
4. Sheer : ADJ. thin and transparent views’. Here, we should use ‘ambivalence’.

5. Buffet : N. tremendous blow; slap ‘Anxious’ and ‘eager’ are used interchangeably but
anxious has a negative connotation, as it refers to
6. Deliberate : V. consider; ponder; discuss nervousness. When you look forward to something
7. Intimate : V. hint or suggest; give information that you know is going to make you happy, you
are ‘eager’, not ‘anxious’.
8. Pluck : N. courage
Hence, the correct sequence is ABAB. Hence, (1).
9. Rent : V. past tense of rend (disturb (the air)
sharply with noise) 23. To ‘straighten’ is ‘to become straight’ or ‘put in
order’. To ‘straiten’ is ‘to constrain options’ or ‘to
10. Transports : N. strong emotion; rapture, ecstasy,
put one in difficult circumstances’. Here, we should
etc.
use ‘straitened’.
11. Bid : V. commanded; ordered
A ‘sojourn’ is actually ‘a temporary stay in one
12. Riddled : V. pierced with many holes place’. If you’re constantly on the move, you’re
13. Slight : N. insult to dignity; snub not engaged in a ‘sojourn’. Hence, we should use
‘journey’ here.
14. Scrape : N. an embarrassing or difficult situation
caused by one’s own mistake A ‘benefactor’ gives benefits; a ‘beneficiary’ receives
them. Hence we should use ‘beneficiary’ here.

61
CATapult
VOCABULARY

A ‘crevice’ is ‘a tiny crack in a wall’; a ‘crevasse’ is 5. Humour : N. the quality of being amusing or comic
‘a huge crack in a sheet of ice’. Here, we should
V. comply with unreasonable wishes
use ‘crevice’.
to keep someone content
Hence, the correct sequence is BABB. Hence, (4).
6. Low : ADJ. of less than average or requisite height
24. ‘Urbane’ means ‘courteous, polite, refined or suave’.
V. (of a cow) make a mooing sound
‘Urban’ means ‘related to a city’. The tone in a
letter has to be ‘urbane’ and not ‘urban’. 7. Rest : N. the remaining part of something

‘Invaluable’ means ‘precious’; ‘inexpensive’ means V. cease work or movement in order to


‘of very little value’. An imitation off the streets is relax
usually ‘inexpensive’. 8. Converse : N. a situation, object or statement
‘Carousal’ is ‘a wild drunken party’. A ‘carousel’ is that is just the opposite of another
either ‘a merry-go-round’ or ‘a rotating platform’. V. engage in conversation
Here, it refers to the merry-go-round.
9. Purchase : V. acquire something by paying for it
‘Repatriate’ means ‘send someone back to their
N. firm contact or grip
home country’; ‘expatriate’ means ‘a person living
outside his/her native country’. In this case, the 10. Partial : ADJ. incomplete; existing only in part
correct word should be ‘repatriate’.
ADJ. having a special liking for some
Hence, the correct sequence is ABAA. Hence, (3). thing
25. ‘Peddle’ means ‘to carry small articles from place 11. ‘Incriminate’ and ‘adjure’ are both verbs, whereas
to place for sale’ while ‘pedal’ means ‘to use in the given context we require an adjective. ‘In-
one’s feet to turn the pedals of a bicycle etc.’ criminate’ means ‘to accuse or blame’ and ‘adjure’
Thus ‘peddle’ should be used in the first sentence. means ‘to charge or command earnestly’. Among
‘Confidantes’ mean ‘women to whom secrets are the given options, only ‘puerile’ is an adjective.
entrusted’ while ‘confidence’ means ‘a relation ‘Puerile’ means ‘childishly foolish’ or ‘immature and
of trust or intimacy’. Thus only ‘confidantes’ trivial’. In both the sentences, the word ‘puerile’
can correctly fit the second sentence. ‘Invoking’ can fit perfectly. Hence, (2).
means ‘citing as authority’ while ‘evoking’ means
12. ‘Placative’ means ‘intended to appease or pacify’.
‘eliciting or drawing forth’. Since the lawyer used
‘Egregious’ means ‘extraordinary in some bad way;
the self-defence principle, only ‘invoking’ correctly
glaring’. ‘Congenital’ means ‘inborn or pertaining to
completes the third sentence. ‘Faint’ means ‘weak’
a condition present at birth’. ‘Placative’ can fit the
while ‘feign’ means ‘give a false appearance of’.
first blank; the sentence will imply that the target
Since he tried to appear ignorant, only ‘feign’ fits
is somewhat unachievable and has been set only
the fourth sentence correctly. Thus the correct
to appease or pacify certain people. The second
sequence is AAAB. Hence, (1).
sentence implies that since the Prime Minister
has decided to send token and combat-avoiding
PRACTICE EXERCISE 3 units, he must be trying to placate or appease
Afghanistan. Therefore, ‘placative’ can fit in the
1. Desert : N. waterless, desolate area of land blank. Hence, (1).

V. abandon in a treacherous manner 13. First let us look at the meanings of the words
given as options. ‘Epitaph’ is ‘a short piece of
2. Founder : N. person who establishes an institution
writing about a dead person, usually inscribed on a
or settlement
gravestone’. ‘Epigram’ is ‘a short witty saying that
V. fail or sink expresses an idea in a terse manner’. ‘Epigraph’ is
‘a quotation at the beginning of a book / chapter
3. Season : N. weather-based division of the year
or an inscription on a building or statue’. The first
V. add spices to food blank should have the word ‘epigrams’ because it
4. Husband : N. a married man (with respect to is followed by a short and witty quotation. From
his wife) the context, it is clear that the second blank should

V. use economically

62
Chapter VA
QUESTION TYPES 1.2

also have the word ‘epigrams’. The third blank 21. The relationship is that of a quality and an animal
should have the word ‘epigraph’ because a short or thing associated with it in idiomatic phrases,
and witty saying is often used at the beginning of a i.e. ‘as gentle as a lamb’, and so on. But (4) does
book or a chapter. One would not use an epigram not fit this pattern: the correct phrase is ‘as sly
on a tombstone. The inscription on a tombstone as a fox’, not ‘jackal’. Hence, (4).
is usually about the person who is buried there.
22. Only option (3) matches all the meanings correctly
Thus, the fourth bank would have the word ‘epi-
with all the usages. Hence, (3).
taph’. Only (5) gives the correct sequence of the
words. Hence, (5). 23. Only option (1) matches all the meanings correctly
with all the usages. Hence, (1).
14. First let us look at the meanings of the words given
as options. ‘Quite’ means ‘to a considerable extent 24. Only option (2) matches all the meanings correctly
or degree’. ‘Quiet’ means ‘making very little noise’. with all the usages. Hence, (2).
‘Ordinance’ is ‘a law or rule made by a government 25 . Only option (2) matches all the meanings correctly
or authority’. ‘Ordnances’ are ‘military weapons or with all the usages. Hence, (2).
ammunition’. The first blank requires an adverb
that would qualify the adjective ‘democratic’. So,
only ‘quite’ can be used here. The second blank
should have the word ‘quiet’ because it refers
to the Parliament’s silence on certain issues. An
‘ordinance’ and not ‘ordnances’ can be passed by
the Parliament. Therefore, the correct sequence of
words is ‘quite, quiet, ordinances’. Hence, (4).
15. The question words are antonyms of each other,
as are the ones in options (1), (2) and (4). (3)
does not have this relationship. Hence, (3).
16. ‘Invective’ and ‘praise’ are antonyms. The pairs
in options (1), (3) and (4) are also antonyms of
each other. The correct option is (2), as ‘brook’
and ‘tolerate’ are synonyms. Hence, (2).
17. The capitalized pair are synonymous, as are the
ones in options (1), (2) and (4). However, the pair
in (3) are opposites: ‘stentorian’ means very loud
or booming. Hence, (3).
18. A ‘soirée’ is a ‘gathering’, but a ‘mallard’ is not a
‘hammer’. The pairs in options (1), (2) and (3) are
synonyms, like the capitalized pair. Hence, (4).
19. A ‘donor’ shows ‘generosity’. The same relation-
ship holds for options (1), (2) and (3). However,
a ‘snob’ does not show ‘truculence’, i.e. defiance.
Hence, (4).
20. A ‘caterpillar’ grows into a ‘butterfly’. All the
options have the relationship of a young animal
to its adult form. However, in option (3), a ‘filly’
refers specifically to a female young horse, and
not a young horse in general. Hence, (3).

63
CATapult
READING COMPREHENSION

VA-1.3 SPEED READING


PRACTICE EXERCISE
from the penultimate sentence of the passage.
1. This question goes directly to the heart of the Hence, [4].
matter – the purpose. Locate the topic sentence
of the passage and the rest should be easy. Sen-
tences 5 and 6 of paragraph 1 clearly indicate the 10. Iron ore, soybeans and sugar have been mentioned
answer. Hence, [2]. in the very first line of the passage. Items in [1]
and [4] are mentioned in the last sentence of
the penultimate paragraph. The third paragraph
2. See paragraph 2, sentence 5, which indicates what discusses how expensive croissants and bikes
statement 3 states. Hence, [3]. are, but doesn’t mention that they are exported.
Hence, [3].
3. Quickly scan what Carol Prives has to say about
the p53 gene in paragraph 3. Both [2] and [3] are 11. In the case of this passage, the first and the last
mentioned. Hence, [4]. paragraph clearly indicate the primary purpose of
this passage: to show that history can be treated
as a science, if it uses the comparative method.
4. As per the first sentence of the last paragraph,
Only option [3] states this. Hence, [3].
most p53 mutations are not inherited.
Hence, [2].
12. Paragraphs 4 and 5 indicate that I and II affect
5. Refer to paragraphs 5 to 7. [1] and [4] contradict economic prosperity. III cannot be inferred since
what is stated about the p53 genes of those who paragraph 5 shows that the Dominican Republic
have Li-Fraumeni syndrome. Nothing is mentioned prospered despite being ruled by a dictator. Hence,
about a bad p53 gene destroying the good p53 [1].
gene, so [2] is wrong. Only [3] can be inferred,
based on the ribbon example. Hence, [3].
13. The last paragraph clearly indicates that a scientific
method isolates the handful of factors that account
6. The last two sentences, the topic sentences of the for a majority of the variance; the paragraph argues
paragraph, and summation of the passage reveal that the comparative method is one such method
the answer. Hence, [4]. and can be used to make the study of history a
science. Only option [1] states this. Hence, [1].

7. There isn’t one main concern regarding the Brazilian


economy that has been highlighted in the passage. 14. Statement I is too extreme and cannot be inferred
All three concerns i.e. [1], [2] and [3] have been from the passage. The whole idea of the passage
talked about at separate places in the passage. is that innovation is fostered by getting as many
Hence, [4]. ideas and as many minds together. From this it can
be inferred that the more diverse the ideas, the
more chances are that they will interact in ways
8. The first two sentences of the sixth paragraph that result in more innovation, i.e. statement II.
reveal the answer. Hence, [1]. The third paragraph starts by saying that closed
environments, which were the norm for innovation
as mentioned in the second paragraph, make it
9. The sixth and the seventh paragraphs reveal how more difficult to explore the adjacent possible. It
inflation has been a problem for Brazil since the then goes on to state that big companies are now
early 1980s. Though its method of dealing with beginning to embrace open environments. So why
inflation has attracted a lot of foreign money, the should companies change? The reason for this
paragraph doesn’t state that this has given Brazil must be that they have reached their saturation
a significant edge. So, [4] is false. The second as far as innovation in closed environments is
sentence of the penultimate paragraph makes [1] concerned, i.e. statement III. Hence, [2].
correct. [2] can be calculated from the last sen-
tence of the first paragraph. [3] can be inferred

64
Chapter VA
SPEED READING 1.3

15. There are various phrases in the passage which the left wing radicals, so [2] is incorrect. Though
describe the way innovation occurs when many the author wonders whether the revolting Arab
ideas are placed together: ideas can ‘serendipitously’ nations will be able to maintain their tentative
(discovery by accident or fortunate coincidence) hold on democracy, he would not say that it is
connect. These are unplanned collisions. So, in- unlikely. Only [4] is correct, as it is stated in
novation occurs not by a plan but by unplanned the second paragraph: ‘These revolts are also
or lucky accidents. Hence, [1]. illustrative of the radical and disruptive political
power of new communication technologies, which
have achieved in some measure what decades of
16. The question is about the transition in how com- traditional political work could not.’ Hence, [4].
panies foster innovation. The earlier rationale is
stated in the second paragraph: the huge financial
reward will be the incentive for people to innovate. 20. ‘Harangue’ means a ‘verbal attack’, i.e. a ‘tirade’.
So the focus was on the incentive to innovate. ‘Qualm’, meaning ‘doubt’, ‘squabble’, meaning
Whereas in the new ways, the focus is on creating ‘quarrel’, and ‘trepidation’, meaning ‘anxiety’, are
conditions that will enable innovation to occur, i.e. not similar in meaning to ‘harangue’ at all. Hence,
open environments. Statement II mentions this. [3].
Statement I can be inferred as well, especially
from the last two sentences of the last paragraph.
Statement III can be inferred from the examples 21. Refer to the last sentence of the second paragraph:
of organizations like IBM and Procter & Gamble, the author states that the concept of determin-
as well as Web startups that encourage outsiders ism tends to be confused with similar concepts
to innovate on their products. Therefore, all three such as predictability and fate. He then goes on
statements are correct. Hence, [4]. to disentangle its true meaning from these other
concepts. Option [2] best captures this. Option
[1] is incorrect: though the author mentions the
17. In the first paragraph the author is highlighting the roots of determinism in the first paragraph, he
significance of the revolutionary forces gathering does not trace its history after that. Leibniz and
in the Arab world. In the second he analyses Laplace are mentioned only in the first and last
the reason behind it and also suggests how the paragraphs respectively and the passage is not
future might pan out – a full-fledged Arab de- about the difference between their concepts of
mocracy or another dictatorship – and stresses determinism. The author shows the differences not
the importance of international support to the the similarities among determinism, predictability
Arab democratic cause. Only option [2] captures and fate. Hence, [2].
the purpose of both the paragraphs. Options [1]
and [3] limit themselves to parts of the first and
second paragraph respectively. Option [4] relates 22. The last paragraph is a description of the ‘famous
to just a minor point made in the first paragraph. expression of determinism by Laplace’. Accord-
Hence, [2]. ing to Laplace, an intelligence that knew all the
forces acting in nature at a given instant and the
momentary positions of all things in the universe,
18. The author’s tone in the first paragraph is extremely and was powerful enough to analyse all this data,
positive, so ‘incredulous’, which means ‘disbe- could predict the future. The possibility is clearly
lieving’ is a bit too negative. The author already theoretical, since Laplace does not suggest that
seems to know what has happened, so ‘curious’, human beings have or ever could have such a
meaning ‘eager to know’, does not really fit. While capacity. Hence, [1].
the author is indeed ‘cheerful’, the word that best
describes his tone in the passage is ‘excited’: he
uses positively charged phrases like ‘epiphanic 23. Refer to the third paragraph: ‘we can imagine
moments of global history’ and ‘clarion call for that certain things are fated to happen, without
democracy’ to express his excitement about what this being the result of deterministic natural
is happening in the Arab countries. Hence, [2]. laws alone; and we can imagine the world being
governed by deterministic laws, without anything
at all being fated to occur’. Therefore [1] is true.
19. There is no hint in the passage as to how long Such a relationship between predictability and
the Arab nations were planning revolution. So [1] determinism, or between fate and predictability
cannot be inferred. According to the first paragraph, is not mentioned in the passage. Hence, [1].
no one predicted the Arab revolutions, not even

65
CATapult
READING COMPREHENSION

24. Refer to the first paragraph. [1] is not what


Leibniz’s Principle of Sufficient Reason is stated
to be (it is similar to Laplace’s views stated later
in the passage). [2] is correct, as stated in the
second sentence of the paragraph. However, [3]
is not quite true: philosophers of science do not
consider Leibniz’s Principle of Sufficient Reason
necessary for understanding determinism, but it is
an exaggeration to say that they reject it outright.
Hence, [2].

66
Chapter VA
UNDERSTANDING PASSAGES 1.4

VA-1.4 UNDERSTANDING PASSAGES


PRACTICE EXERCISE 1 10. ‘Paramount importance’ as mentioned in [1] is
too extreme an inference. [3] is the opposite of
what the author is stating. [4] is untrue as per
1. Option [2] contradicts the author’s emphasis
the passage, since it is internally flawed – the
on politics. [3] can be dismissed as it is a very
author does not acknowledge the scientific basis
general statement. [4] is close but is ruled out
of the polygraph. [2] is generalized, but true to
on account of the last phrase, which contradicts
the author’s attitude on the issue. Hence, [2].
what the author has said in the passage. Only [1]
sums up the author’s argument, thus providing a 11. Scan the 4th and 5th paragraphs. Since the Fifth
good conclusion. Hence, [1]. Amendment is related to the defendant’s rights
to refuse proceedings, this can only relate to his
2. Everyone who thought differently from those in
personal rights in some ways. [2] and [3] are ir-
power was silenced by extra-constitutional power.
relevant here. Between [1] and [4], only [1] relates
Hence, [2].
to an individual’s own constitutional rights. Hence,
3. Throughout the passage, the author’s main grouse [1].
against Nduom seems to be his lack of political
12. The question asks for the statement that is unlikely
vision. Hence, [2].
to be a factor that would sway public’s opinion
4. According to paragraph 5, those who wield political in favour of polygraph testing. [1], [2] and [3]
power do affect society when they make significant are likely to be factors that would do so. [4] is a
economic decisions. Hence, [3]. factor that may be concerned with the polygraph
5. Refer to the second sentence of the seventh testers and practitioners and not with the general
paragraph. Hence, [2]. public as such. Hence, [4].

6. Refer to paragraph 10. [1] best expresses the 13. The focus is clearly polygraphs and not the broader
liberal nature of democracy. Hence, [1]. issue suggested in [1]. [3] does not make sense as
per the passage. This is obviously a case against,
7. The author is merely critical of the views expressed not for polygraphs, so [4] is out. This leaves us
by Nduom. There is no hint of any kind of personal with [2], which succinctly states the theme of the
sentiment involved. Hence, [4]. passage. Hence, [2].
8. [1], [3] and [4] are negated because the passage
as a whole deals with the importance of politics
as a contributing factor towards the success of
democracy. Hence, [2].
9. The focus of the passage is clearly polygraph
testing. The judgment about forensic evidence
in general is not representative of the passage,
hence [2] is out. The passage does not focus on
the polygraph’s popularity. So [3] is out. Since the
passage does not sound optimistic about the future
of the polygraph, [4] is out. [1] gives a summary
of the passage. Hence, [1].

67
CATapult
READING COMPREHENSION

14.

PARAGRAPHS PRIMARY PURPOSE


1 To introduce the idea that great scientists ask questions that challenge conventional
thinking, using the example of Einstein’s curiosity
2 & 3 To elaborate on Einstein’s hypothesis and to show how it was related to earlier theories
4 To explain the challenges before Einstein and the qualities that make him special
5 This paragraph introduces the key idea that the author intends to put forth. The rest
of the paragraphs have been laying the foundation for the idea expressed in this para-
graph: great scientists take on path-breaking projects because of certain fundamental
beliefs or themata they hold about the functioning of the universe; in Einstein’s case
he strongly believed that the entire universe was governed by a few simple laws.
6 & 7 To develop the idea that great scientists hold on to themata like religious devotees
hold on to religious beliefs and use them to explain the workings of the universe

Each of the answer options is the primary purpose PRACTICE EXERCISE 2


of one of the paragraphs. The key lies in identi-
fying the build up of supporting ideas/arguments
towards the main/central idea, which is that great 1. As per the last sentence of the passage, the
scientists are guided in their work by beliefs that workers of the future will have income but not
are almost like religious beliefs (see paragraph 5). work. Hence, [2].
Hence, [3]. 2. As per the twelfth sentence of the last paragraph
15. Though the example of Einstein’s theory of relativity – But has the socialist thought about what he
takes up a significant portion of the passage, it only would do if owing to technological advance, the
serves to illustrate the central idea. So options [3] amount of human labour were catastrophically re-
and [4], which focus only on Einstein are wrong. duced – socialism has not taken into consideration
[2] sets up a false dichotomy – the passage does the possibility of an immense reduction of human
not suggest that science and religion are opposed labour in the wake of mechanization. Hence, [3].
to each other. Rather, the passage shows that great 3. As per the tenth sentence of the last paragraph
scientists’ beliefs in the methods and purposes of – The labour movements arose largely as a revolt
science are almost religious in nature. Therefore, against the conception of workers as commodities
only [1] is a suitable title. Hence, [1]. to be bought and sold without regard to their needs
16. Only [3] gives the correct word-meaning combi- as human beings – the labour movements arose
nations. Hence, [3]. as a revolt against the conception of workers as
commodities. Hence, [3].
17. ‘Triumph’ means ‘victory’. Hence, [4].
4. As per the fourth sentence of the last paragraph,
18. According to the fifth sentence of the passage: the chief purpose of competitive enterprise is to
But the magnificent……….only for time, the living realize the maximum profit. Hence, [4].
should look at life for only a short while. Hence,
[3]. 5. The first sentence of the last paragraph clearly
states that in the situation created by the rapid
19. ‘Rapture’ means ‘ecstatic joy’. Hence, [4]. extension of machine production, our object should
20. One sees ‘a reflection’ of any image that falls on be to limit the amount of leisure to that which
the surface of the water. So, by the last line the can be profitably used. Hence, [3].
author means that the mind is only a reflection 6. As per the first paragraph, the artist, the writer,
of the self. Hence, [4]. the scientist etc. do not consider their calling as
21. The passage talks about how wonderful it is to work and find pleasure in their work by using
be alive and how one should experience the joy their creative energy. Option 1 mentions about
of living, so ‘Alive and Kicking’ would be the most the utilitarian value which is a characteristic of
suitable title for this passage. Hence, [4].

68
Chapter VA
UNDERSTANDING PASSAGES 1.4

work and not occupation. So it can be negated. the loss of soil nutrients and not salinization. Thus
Though the third sentence mentions that occupa- option 2 is also incorrect. As per the fifth and sixth
tion absorbs time and energy as long as a person sentences of the first paragraph, natural fertility
gives it, option 3 incorrectly states that workers is quickly exhausted in the sub-tropical forest
choose to give their time and energy. Thus it can regions. However, since the question is related to
be negated. As per the eighth sentence of the first salinization, option 3 is incorrect. Hence, [4].
paragraph, their reward is doing their work. Thus
11. Though the first sentence of the passage talks
option 4 can also be negated. Hence, [2].
about soil erosion, the passage is about maintaining
7. Option 1 is true as per the second sentence of the fertility of soil. Thus option 1 is incorrect. The
the first paragraph. Option 2 is true as per the fourth sentence of the first paragraph only mentions
third sentence of the same paragraph. Option that soil fertility is renewed in areas of active vol-
3 is also true according to the fifth sentence of canism. However, agriculture in volcanic islands is
the first paragraph. However, as per the seventh not mentioned in the passage. Thus option 2 can
sentence of the first paragraph, there are some be negated. The importance of chemical fertilizers
forms of work which can hardly be differentiated is only stated in the penultimate sentence of the
from occupation. Thus option 4 is not true as per second paragraph. Thus option 3 is unsuitable as a
the passage. Hence, [4]. title for the passage. Throughout the passage, the
author mentions various ways in which soil fertility
8. The desire to make a profit is mentioned in relation
is lost and the various ways in which fertility can
to competitive enterprise and not occupation. Thus
be regained. Thus option 4 is the most appropriate
option 1 is incorrect. Nothing has been mentioned
title to this passage. Hence, [4].
about people taking up occupation because they
want to do something uncommon. Thus option 2 12. As per the fifth and sixth sentences of the first
is also incorrect. Work talks about utility in general paragraph, natural fertility is quickly exhausted in
and not something as being useful to society. Thus the humid tropical forest regions. Hence, [2].
option 3 can also be negated. As per the first
13. The example of the Nile river depositing a rich
paragraph, occupation demands constant initia-
layer of silt over the soil to restore its fertility is
tive and people taking up an occupation choose
mentioned in the first paragraph. Negate option
to give it time and energy as per their will. Thus
1. The last sentence of the third paragraph states
only option 4 correctly states the chief reason for
how bacteria can restore nitrogen to the soil, thus
a person taking up an occupation. Hence, [4].
restoring its fertility. Negate option 2. Nothing has
9. As per the first paragraph, if work is performed been mentioned about fertilizer fixation through
by a suitable agent, some amount of pleasure lightning. Retain option 3. The second paragraph
can be associated with it. Thus option 1 can be mentions how organic manures were used to
negated as work is not unpleasant at all times. restore the fertility of soil. Thus option 4 can be
Option 2 can also be negated as sometimes work negated. Hence, [3].
and occupation cannot be differentiated, and for
14. As per the second sentence of the second paragraph,
most people, the pleasure of occupation needs
crop rotation helped to maintain the condition of
the addition of the necessity provided in work.
the soil, and also to prevent the build-up of those
As per the fourth and fifth sentence of the first
insects and other plant pests that are attracted
paragraph, work has utility while occupation is an
to a particular kind of crop. Hence, [3].
end in itself. Retain option 3. The first sentence
of the paragraph itself states that there should be 15. The first paragraph only mentions that the annual
a distinction between work and occupation. Thus flooding of the Nile helped in restoring the fertil-
option 4 is incorrect. Hence, [3]. ity of the soil. Thus nothing has been mentioned
about the Nile water containing bacteria, excess
10. As per the last paragraph, salinization is a problem
of salts or nutritive material. This negates options
caused due to inadequate drainage of irrigated
1, 2 and 4. However, the last paragraph states
water. Option 1 is incorrect as it specifically men-
that areas that have insufficient rainfall but are
tions irrigation with well-water. As per the third
irrigated develop the problem of salinization due
paragraph, not practicing crop rotation leads to
to poor drainage. Thus it can be inferred that the

69
CATapult
READING COMPREHENSION

agricultural land in the Nile basin consists of heavy Retain it. Option 3 is not related to the author’s
soil with poor drainage properties. Hence, [2]. writing while no tragedy has been mentioned in
the passage. Hence, [2].
16. As per the last sentence of the penultimate
paragraph, plants with nodules on their roots are 22. As per the last sentence of the third paragraph,
known as legumes. Hence, [3]. many officials were selected through the exam-
ination system. Hence, [2].
17. The main aim of the author is to show how ants
from one community recognise each other though 23. As per the second paragraph, the statesmen of the
they are hostile to ants of the same species from Restoration did not want to create a new society
other communities. Option 1 is too general as the but wanted to restore the traditional society under
passage is specifically related to ants. Negate it. new conditions. Thus option 1 correctly mentions
Although the author wants us to learn to be more the primary objective of the Restoration. Hence, [1].
cooperative to our community members, it is just a
24. As per the last sentence of the first paragraph –
thought in the first paragraph and not the main aim
Thus, the only common area of agreement between
in writing the passage. Thus option 2 is incorrect
European and Chinese conservatism is the intent
as a title for the passage. Option 3 is wrong as
to conserve – only the aim of conserving is com-
the ants did not get drunk voluntarily but were
mon between Chinese and western conservatism.
forced by the author to become drunk by putting
Thus option 1 is correct. Retain it. Though Chinese
them into whisky. Also, the author conducted this
conservatism developed during the Taiping Revo-
experiment in order to test if each ant nest has
lution, it is not a similarity between the Chinese
some sign or password by which they recognize
and western conservatism. Negate option 2. As per
one another. Negate option 3. Option 4 is a suitable
the penultimate sentence of the first paragraph,
title for the passage as this is what the author is
western conservatism distrusts cosmopolitanism
trying to prove in the passage. Hence, [4].
while Chinese conservatism defends rational cos-
18. In all the three paragraphs, the author reiterates mopolitan order. Thus option 3 is also incorrect
that ants from one community attack ants from as cosmopolitanism is not a point of similarity
a different community though they belong to the between the two. Nothing has been mentioned
same species. Thus the attitude of ants towards in the passage about Chinese conservatism being
strangers of the same species may be categorized land oriented. Thus option 4 can also be negated.
as ‘hostile’ i.e. antagonistic. Hence, [3]. Hence, [1].
19. Since the ants would not voluntarily get drunk, 25. Tung-chin is the name of the Restoration period
option 1 is supported. Option 2 is also supported while I.Ching and Buddha are not mentioned in
by the anecdote as other ants carried away their the passage at all. Hence, [4].
drunk friends. Option 3 is also supported by the
26. As per the second paragraph, the main aim of
anecdote as the other ants were able to recognise
the Restoration was to revive old institutions and
their intoxicated comrades although the drunk
restore them to their original vitality so that a so-
ants were not in a position to give any sign or
ciety based on tradition, but under new conditions,
password of recognition. However, option 4 is not
could be created. Thus only option 3 correctly
supported by this anecdote as the drunk ants were
answers the question. Hence, [3].
not in a position to give any sign or password of
recognition. Hence, [4]. 27. According to the third sentence of the first para-
graph, western conservatives intended to preserve
20. As per the second sentence of the last paragraph,
Christian and aristocratic elements in European
chloroform was fatal for the ants. Hence, [4].
society. Also, the penultimate sentence of the
21. ‘Sophistry’ means ‘a false argument’, ‘whimsy’ same paragraph states that western conservatism
means ‘capricious humour’ and ‘hypocrisy’ means believed in the sacredness of private property
‘a pretence of having some desirable attitude’. and distrust of cosmopolitanism. Thus all except
Since the author is supporting his observations ‘cosmopolitanism’ was intended to be preserved
by conducting experiments, option 1 is incorrect. by the western conservatives. Hence, [3].
Option 2 correctly mentions the slight humour
that comes across while reading this passage.

70
Chapter VA
UNDERSTANDING PASSAGES 1.4

28. The Chinese examination system is only mentioned


in the last paragraph of the passage. Thus it is
not an appropriate title for the passage. Negate
option 1. The first two sentences clearly state the
aim of the author in writing this passage. Thus
option 2 is an appropriate title for the passage.
Retain it. Only the third paragraph mentions how
the officials rose to prominence. Thus it is not the
main purpose of the author in writing this passage.
Negate option 3. The Taipei Rebellion has been
mentioned in the first paragraph in order to show
when Chinese conservatism began. However, the
passage does not imply that Chinese conservatism
was an effect of the Taipei Rebellion. Thus option
4 can also be negated. Hence, [2].

71
CATapult
GRAMMAR

VA-1.5 ARTICLES AND PARTS OF SPEECH

PRACTICE EXERCISE 1 24 to 26: In the given sentences no article is required.


Uncountable nouns (things that cannot be
counted, for example concepts like life, hap-
1 to 5: In all the given sentences the article required piness, knowledge, etc) are not preceded by
is ‘a’ as the things or persons spoken of are all indefinite articles.
indefinite, i.e., they are not any particular person
or thing, they could be any person or thing. Hence, 27 & 28: The definite article ‘the’ should be used in
the rule of indefinite articles has to be applied. the given sentences. The definite article ‘the’
Note that the article does not change with the is used when a singular noun is meant to
gender of the noun (so, ‘a girl’, ‘a small boy’). represent a whole class. The nouns in the
Also, the indefinite article is always followed by sentences are singular and they represent
a singular noun. the whole class.

6 to 9: In all the given sentences the article required is 29: No article is required in the given sentences.
the indefinite article ‘an’. We use ‘an’ before a When ‘man’ is used in the general sense to
word beginning with a vowel (a, e, i, o, u) sound. denote the whole class, no articles are used
Here also the rule of indefinite articles has to before it.
be applied as the person or thing spoken of is 30 to 33: In the given sentences the article required is
not a particular one. Again, there is no plural ‘the’. ‘The’ is used with the names of gulfs,
form of the indefinite article ‘an’. rivers, seas, oceans, groups of islands and
10 to 14: In all the given sentences the article required mountain ranges.
is ‘a’. We use ‘a’ before words beginning with 34 & 35: The article required in these sentences is
a consonant sound in spite of their spelling ‘the’. We use ‘the’ before common nouns,
beginning with a vowel. In all the above which are the names of unique things, like
sentences words (union, university, U.S.) the sun, the moon, the sky etc.
begin with a consonant sound, that of ‘yu’
and the word (one) starts with a consonant 36. Both the sentences are correct. In A there is no
sound, that of ‘w’. The choice of article is need to use an article before ‘honest’ because when
actually based upon the phonetic (sound) we refer to abstract ideas in a general sense, the
quality of the first letter in a word, not on abstract noun is not preceded by an article. We
the written representation of the letter. If use an article before an abstract quality when it
the first letter makes a vowel-type sound, qualifies a person and since, ‘honest’ begins with
we use ‘an’; if the first letter would make a vowel sound, the indefinite article ‘an’ has been
a consonant-type sound, we use ‘a’. Here used correctly in B. Hence, [3].
also the rule of indefinite articles has to be 37. Sentence A is incorrect: ‘source’ should be pre-
applied, as the person or thing spoken of is ceded by the indefinite article ‘a’. The indefinite
not a particular one. article is generally used before singular countable
15 to 18: In all the given sentences the article required nouns. But in some cases uncountable nouns can
is ‘an’. We use ‘an’ before words beginning also be preceded by indefinite articles, e.g., a
with a vowel sound in spite of their spelling source of strength, a good night’s sleep, a working
beginning with a consonant. In all the above knowledge of mathematics, etc. In the second
sentences the words (hour, honest, heir) begin sentence, the definite article ‘the’ before ‘source’
with a vowel sound, as the initial consonant is correct because it refers to a particular source.
‘h’ is not pronounced. Hence, [2].

19 to 23: In all the given sentences the definite article 38. There is no need to use an article when we talk
‘the’ should be used. When we speak of a of things in general. In A the reference is to ‘peo-
particular person or thing, or the one already ple’ and ‘homes’ in general. So, the sentence is
referred to, we use ‘the’. In all the sentences correct. In B the reference is to a particular group
above a particular thing or person is spoken of people – the ones who have been displaced by
of, hence the rule of definite articles has to the flood, so in this sentence the definite article
be applied. ‘the’ should precede ‘people’. Hence, [1].

72
Chapter VA
ARTICLES AND PARTS OF SPEECH 1.5

39. An important rule about article usage is that we ject (as in A), the article is omitted. So, A should
sometimes use the article depending on the context. read: ‘…Pompey planned to give battle the next
When a place like school or hospital is visited for morning’. Similarly, in certain phrases consisting
its primary purpose, the article before the word of a preposition followed by its object (as in B),
is eliminated. But when the place is visited by the article is omitted. So, B should read: ‘…while
someone for some other purpose, the article ‘the’ on horseback’. Hence, [4].
is used. In A, the patient visits hospital for its pri-
47. In A, the use of the indefinite article before ‘word’
mary purpose, i.e., treatment, so ‘reached hospital’
is correct. Here it conveys the original numerical
is correct, whereas in the second sentence the
sense of one. ‘The cholera epidemic’ and ‘the village’
students went to the hospital to distribute gifts,
are also correct phrases as the reference is to a
so ‘the hospital’ is correct. Hence, [3].
particular epidemic and a particular village. In B,
40. ‘Advice’ is an uncountable noun and we do not the definite article before the noun ‘job’ is used
use an indefinite article before an uncountable for the sake of emphasis, to give the expression of
noun. So, sentence A should read ‘…he gave me a superlative. So, both the sentences are correct.
advice on’ or ‘…he gave me some advice on’. In Hence, [3].
B, since ‘piece’ qualifies the uncountable noun
48. Some nouns can be both countable and uncountable.
‘advice’, the indefinite article ‘a’ before ‘piece of
In A, ‘coffee’ is used as a countable noun – a cup
advice’ is correct. Hence, [2].
of coffee – hence, the article ‘a’ before it is correct.
41. When we use an adjective to talk about a group In B, ‘coffee’ is an uncountable noun and refers
of people or the entire class, we use the definite to the entire produce of coffee. Hence, no article
article ‘the’ before the adjective, for example, ‘the is required before it. So, both the sentences are
unemployed’ meaning all the unemployed people correct. Hence, [3].
as a group. Here the adjective ‘homeless’ actually
49. Sentence A is correct because we use the definite
represents the group of people who have been
article ‘the’ before names of months, days or weeks
rendered homeless. So, both the sentences require
when we refer to particular days/months/weeks.
‘the’ before the word ‘homeless’. Hence, [4].
When we refer to months or days in an indefinite
42. We normally use ‘the’ with the names of buildings, manner, we do not use any article before them.
except if the first name is the name of a place. So, B should not have ‘the’ before Monday. So,
Therefore, ‘The White House’ and ‘trip to Buck- only A is correct. Hence, [1].
ingham Palace’ are correct. Hence, [3].
50. An article is not used when a transitive verb is
43. When we talk about the location of a place within followed by its object; so ‘to lose heart’ is the
another place, we use ‘the’ with the words stating correct expression. An article is not used in certain
direction like, north, south, east and west. But phrases consisting of a preposition followed by its
when we compare the locations of two places, we object, so ‘at home’ is also correct. Hence, [3].
do not use ‘the’ with the words stating direction.
Therefore, in A, the correct expression should be PRACTICE EXERCISE 2
‘…Maldives is south of India…’. B is correct because
it shows Scotland’s location within Great Britain.
Hence, [2]. 1. Agree – The word refers to the action of consent-
ing; hence it is a verb. A verb is a word used to
44. When an abstract noun is used in a general sense, express an action or a state of being.
the noun is not preceded by an article. ‘Life’ in A
refers to the general condition of human existence; Helper Verbs: The verbs be (am, is, was, are,
hence it does not require an article. In B, the were), has, have, does and did when used with
reference is to a particular aspect of existence, ordinary verbs to indicate tenses, passive forms,
so ‘the life of a farmer’ would be correct. Hence, questions and negatives are called auxiliary verbs
[1]. or auxiliaries or helper verbs. Some other helper
verbs, also known as modal auxiliaries, are can,
45. Both the sentences are correct if the context in could, may, might, shall, should, will, would, must,
each is different. In A, the repetition of the articles ought, used (to), need, dare. These verbs express
is justified if the manager and the floor supervisor the modality of the main verb.
are two different people. In B the manager and
floor supervisor is one person who performs the Transitive & Intransitive verbs: Transitive means
two roles. Hence, (3). passing over. The action denoted by the verb passes
over from the doer or subject to some object. E.g.,
46. Both the sentences are incorrect. In certain phrases The boy ate a biscuit. (You ask the question what
consisting of a transitive verb followed by its ob- to the verb. If you get an answer, it is a transitive

73
CATapult
GRAMMAR

verb.) Intransitive means not passing over. The is an adjective (I-B). In C, the word indicates an
action denoted by the verb does not pass over action, hence it is a verb (IV-C). In D, ‘fast’ qual-
from the doer or subject to an object. E.g., The ifies the adjective ‘asleep’, hence it is an adverb
girl cries softly. (II-D). So, the sequence is I-B, II-D, III-A, IV-C.
2. Single – The word actually gives us more idea 12. In sentence A, ‘well’ is an interjection used to
about the nomination, i.e., it qualifies the noun. It introduce a sentence or resume a conversation
is, therefore, an adjective. An adjective is a word, (IV-A). In B, ‘well’ modifies the adjective ‘behave’,
which describes or modifies a noun or a pronoun. hence it is an adverb (I-B). In C, ‘well’ indicates
the act of springing or gushing, hence it is a verb
3. Nomination – This is the name of an idea, hence
(III-C). In sentence D, ‘well’ is an adjective as it
a noun. Therefore, a noun is a word used to name
qualifies the pronoun ‘she’ (II-D). So, the sequence
a person, thing or idea.
is I-B, II-D, III-C, IV-A.
4. For – This word shows the relation between two
13. In sentence A, ‘past’ refers to the history of the
nouns in the sentence, ‘nomination’ and ‘position’,
country and is used as a noun (III-A). In B, ‘past’
so it is a preposition. A preposition is a word used
qualifies the verb ‘marched’ and hence is an adverb
to show the relationship of a noun or a pronoun
(IV-B). In sentence C, ‘past’ indicates direction and
to another word in the sentence.
connects the two nouns ‘house’ and ‘post office’.
5. Ago – The word means ‘in the past time’ and So, it is a preposition (I-C). In D, ‘past’ qualifies
qualifies the adverb of time ‘long’; thus ‘ago’ is the noun ‘presidents’ and is an adjective (II-D).
an adverb. An adverb is a word used to modify So, the sequence is I-C, II-D, III-A, IV-B.
a verb, an adjective or another adverb.
14. ‘Out of slump’ in C is incorrect because it is
6. Alas – This word does not serve any particular missing an article. In D, ‘a temporary stimuli’ is
grammatical function. It is just used as an ex- wrong as stimuli’ is a plural word. The correct
clamation and can stand alone; therefore, it is an word in E should have been ‘effect’ (noun) and
interjection. not ‘affect’ (verb). There is no glaring error in A
7. Them – This word actually replaces the noun but as the only option left is B, (5) must be the
‘countries’, hence it is a pronoun. A pronoun is answer. Hence, [5].
a word used in the place of a noun or nouns. 15. As an adjective, ‘down’ implies ‘of or relating to
The noun that a pronoun stands for is called its a train(s) from a more important place or one
antecedent. regarded as higher.’ Thus, ‘down’ is used as an
8. And – This word acts as a link between the two adjective in D. As a preposition, ‘down’ implies ‘in
clauses; hence it is a conjunction. A conjunction a descending direction’. Thus, ‘down’ is a preposi-
is a word used to join words or groups of words. tion in A. As a noun, ‘down’ means ‘a downward
movement or descent’. So, it is used as a noun
9. In A, ‘like’ qualifies the noun ‘minds’ and means in B. When ‘down’ is used as a verb, it means ‘to
‘of the same kind and character’. It does the work throw down or subdue’. So, ‘down’ is a verb in C.
of an adjective (II-A). In B, the word ‘like’ links Thus, the correct order is I-D, II-A, III-B, IV-C.
‘your dress’ with ‘mine’. Thus, it does the work of Hence, [1].
a preposition (IV-B). In C, the word ‘like’ refers to
a kind or sort and is used to name the kind. It is 16. The easiest match than can be made is I-D. In
used as a noun here (I-C). In D, ‘like’ indicates D ‘above’ describes the information. This narrows
an action and does the work of a verb (III-D). So, down the options to [1] and [3]. The only possible
the sequence is I-C, II-A, III-D, IV-B. noun usage is in A, though it makes sense only if
‘above’ is taken to mean ‘heaven’. Both B and C
10. In A, ‘after’ is used as an adjective as it qualifies use ‘above’ as a preposition. So, actually there is
the noun ‘years’ (IV-A). In B, ‘after’ connects two no perfect answer. As [1] contains three correct
independent clauses, hence it is a conjunction matches, it can be marked as the answer. Hence,
(III-B). In C, ‘after’ shows the relation between the [1].
two nouns ‘life’ and ‘death’; thus it is a preposition
(II-C). In D, ‘after’ qualifies another adverb ‘soon’ 17. There can be no confusion about ‘MBA’ being a
and they both qualify the verb ‘reach’. So, after is noun (it is preceded by ‘the’), ‘it’ being a pronoun,
used as an adverb (I-D). So, the sequence is I-D, ‘helps’ being a verb, etc. The deciding word is
II-C, III-B, IV-A. ‘tackle’, which is also clearly a verb. Hence, [5].

11. In A, the word ‘fast’ is used as a noun and refers 18. In A, ‘witness’ is a verb as it is an action word
to the act of keeping away from food (III-A). In B, and in B it is a noun.
‘fast’ qualifies the noun ‘colour-bases’ and hence

74
Chapter VA
ARTICLES AND PARTS OF SPEECH 1.5

19. In A, ‘more’ qualifies ‘time’; hence it is an adjective. 30. The error is in part (1). The latter part of the
In B, ‘more’ is part of the adverbial phrase ‘any sentence indicates that something is being com-
more’, which modifies the verb ‘stand’; hence it pared to a ‘tourist’. So, in part (I) we need a
is an adverb. noun indicative of a person. Thus, ‘reader’ should
replace ‘reading’ in order to make the comparison
20. In A, ‘neared’ is the action of reaching his death
meaningful. Hence, [1].
and so it is a verb. In B, ‘near’ qualifies the verb
‘come’ and so it is an adverb. 31. There is an incorrect word form in part (I). The
adverb ‘badly’ has been incorrectly used after the
21. In A, the pair of ‘neither…nor’ joins two clauses,
verb ‘felt’. Actually, we need an adjective here
hence it is a conjunction. In B, ‘neither’ stands
which should modify the pronoun ‘we’. Therefore,
for ‘not one’ and does the work of a pronoun.
the adjective ‘bad’ should be used in place of
22. In A, ‘that’ points out the object it refers to and ‘badly’ to make the sentence correct. Hence, [1].
is used as a demonstrative pronoun. In B, ‘that’
32. There is an incorrect pronoun form in part (4).
is used with the noun ‘dress’ and it qualifies the
The sentence indicates that the three people are
noun, hence it is an demonstrative adjective. In
subjects i.e. are doing the action Therefore, the
case of confusion, remember, the easiest way to
pronoun should be in the nominative case, i.e.,
distinguish between a demonstrative pronoun and
‘he’, to make the sentence correct. Hence, [4].
a demonstrative adjective is that the former is
followed by a verb whereas the latter is followed 33. The error is in part (4). We require a reflexive
by a noun. pronoun here to indicate that the action has
happened on the doer. So, ‘he hurt himself’ will
23. In A, ‘even’ is used to show that something is
be the correct phrase. Hence, [4].
surprising or unusual and it qualifies the verb
‘know’; so it is an adverb here. In B, ‘even’ is an 34. There is an incorrect word usage in part (2).
adjective meaning ‘leaving no balance of debt on ‘Therefore’ means ‘for that reason’ but the sen-
either side’. tence indicates that apart from reading books,
the speaker also watches movies. Therefore, the
24. In A, ‘while away their time’ implies spending time
appropriate word should be simply ‘and’. Hence,
in a relaxed manner and so ‘while’ is a verb. In B,
[2].
‘while’ means ‘although’ and it joins two clauses
in the sentence. So, in B, ‘while’ is used as a 35. There is no error in the sentence. A relative pro-
conjunction. noun describes the preceding noun in such a way
as to distinguish it from other nouns of the same
25. In A, ‘up’ implies that prices have increased;
class. ‘That’ is a specific relative pronoun that tells
hence it is used as a verb. ‘Up’ in sentence B is a
us specifically which men the stories were about.
preposition as it placed before the noun ‘mountain’
Hence, [5].
and shows the relation between the pronoun ‘us’
and the noun ‘mountain’. 36. The error is in part (4). We need the gerund
‘shouting’ in place of ‘shout’ because it follows
26. In A, ‘wrong’ qualifies the verb ‘led’; hence it is
the preposition ‘by’. A gerund is a noun formed
used as an adverb. In B, ‘wrong’ is used as a
from a verb by adding ‘ing’ to it. Hence, [4].
noun and names the act of injustice meted out
to the innocent people. 37. The error is in part (4) of the sentence. The
adverb form ‘awhile’ is incorrectly used after the
27. In A, ‘high’ is an adjective that means ‘expensive’.
preposition ‘for’. As an object of the preposition,
In sentence B, ‘high’ is adverb noun that means
the noun phrase ‘a while’ is correct. Hence, [4].
‘peak’.
38. Here precision is the name of an attribute. Hence,
28. There is an incorrect word form in part (3). We
[2].
need a word to qualify the noun ‘techniques’;
therefore, the adjective ‘innovative’ should be used 39. In the given sentence, the word blessed signifies
in place of the noun form ‘innovation’, to make the quality of the noun light, so it is an adjective.
the sentence correct. Hence, [3]. Hence, [2].
29. The error is in part (1). Since similar items are 40. Honour is the name of a particular quality. Hence,
mentioned, the gerund ‘borrowing’ should be used [1].
to make it similar to the word ‘lending’. Hence,
41. The underlined word indicates an action. Hence,
[1].
[3].
42. But is otherwise a conjunction. Here it means
with the exception of and is used as a preposition.
Hence, [2].

75
CATapult
GRAMMAR

43. The underlined phrase is actually a group of words 23 & 24: The article ‘the’ is required in the sentences.
used with the force of a single preposition. It When we refer to nationalities as an entire class,
shows the relation between the noun services and the article ‘the’ is used before the names.
the facility that the musician has got. Hence, [2].
25 to 27: No article is required in the given sentences.
44. Here boost is a verb because it denotes an action. Articles are not used before school, college, church,
Hence, [4]. hospital etc. when these places are visited for
their primary purpose.
45. Peaceful is an adjective, which qualifies the noun
environment. Hence, [2]. 28 & 29: Articles are not used before names of materials.
Hence no article is used in the given sentences,
as silver and petroleum are names of materials.

PRACTICE EXERCISE 3 30 to 32: No article is required in these cases. We do


not use articles before names of continents, coun-
tries, states, cities, towns etc. The exceptions are
1 & 2: Sometimes ‘the’ is placed before a common proper nouns such as United States of America and
noun to give it the meaning of an abstract noun. United Kingdom as they are descriptive names.
Like, in the given sentences ‘the fighter in him’
means ‘the spirit like that of a fighter ’, ‘the 33 to 35: When a proper noun is considered a common
businessman in him’ means ‘the spirit like that noun, thereby highlighting the quality associated
of a businessman’. with the person, an article is used before the
proper noun. ‘The Shakespeare’ indicates the
3 & 4: We use ‘the’ before a proper noun only when it extraordinary literary ability of Shakespeare, ‘The
is qualified by an adjective or adjectival clause. Florence Nightingale’ indicates the extreme care
Here the reference is to a particular ‘Mr. Sharma’ and concern displayed by Florence Nightingale
and a particular ‘Milton’. and, in sentence 70, the oppressive qualities of
5: We do not use articles before the names of indi- Hitler are highlighted.
vidual mountains or islands. Some other concepts are given below.
6 to 8:We use ‘the’ with adjectives when they are • ‘The’ is placed before the names of holy books.
in superlative degree of comparison. In all For example, ‘the Bible’ and ‘the Quran’
the above sentences ‘the’ is required before ‘pret-
tiest’, ‘most intelligent’ and ‘best’. • No article is required before plural common nouns
that are used in the general sense. For example,
9 & 10: We normally do not use articles before the ‘tigers are dangerous’ and ‘men are often insen-
names of illnesses. So no article is required sitive to women’s feelings’.
before ‘jaundice’. Before ‘headache’ or cold’
we use ‘a’. So, ‘a bad headache’ is appropriate • No article should follow the phrases ‘kind of’ and
for sentence 10. ‘sort of’. Many students incorrectly place ‘a’ after
these phrases.
11 & 12: The article required in the given sentenc-
es is ‘the’. The definite article ‘the’ is used • Indefinite articles do not precede substances (that
before ordinals, i.e., first, second, third, etc. are uncountable) unless another countable noun is
between the two. For example, ‘a bottle of milk’
13 & 14: The article required in the given sentences and ‘an ounce of sugar’.
is ‘the’. use ‘the’ before the names of musical
instruments. • Articles don’t precede names of meals. For ex-
ample, ‘let’s have dinner’ and ‘will you meet me
15 & 16: In the given sentences ‘the’ is required. We use ‘the’ for lunch tomorrow?’
before an adjective when it is used as noun to represent
the whole class. ‘Rich’refers of the entire class of rich 36. The first and second blanks introduce ‘a wolf’ and
people; it is the same case with ‘weak’. ‘a lamb’. Thus, indefinite articles will be used to
fill the blanks. Since the lamb has gone astray
17 & 18: In the given sentences the article required is from a particular fold, we need to use the definite
‘the’. We use ‘the’ before an adverb in case of article ‘the’ in the third blank. The wolf and the
comparatives. lamb have already been introduced and we are
19 & 20:Articles are omitted before abstract nouns used referring to those particular animals only. Hence,
in the general sense, as in these sentences. ‘the’ is used for both the last two blanks. Thus the
correct sentence is: ‘A wolf, meeting with a lamb
21 & 22:No article is required in the given cases. We do astray from the fold, resolved not to lay violent
not use articles before the names of languages. hands on him, but to find some plea to justify to
the lamb the wolf’s right to eat him’. Hence, [1].

76
Chapter VA
ARTICLES AND PARTS OF SPEECH 1.5

37. The first blank will have an indefinite article as


we are not talking about a particular ‘bat’ but
just introducing it in the sentence. ‘Ground’ will
be preceded by ‘the’ as it represents a particular
locality in relation to the bat. ‘A’ is used in the
third blank as we are introducing a new character
i.e., ‘weasel’. ‘The’ is used in the fourth blank
and sixth blank as we have already mentioned
the weasel and bat respectively. Since ‘enemy’
represents a class of animals, ‘the’ is used to
fill the fifth blank. In the next part, ‘bird’ and
‘mouse’ are used in a general sense, and do not
refer to particular individuals. Thus, an indefinite
article precedes these singular countable nouns.
The correct sentence is: ‘A bat who fell upon the
ground and was caught by a weasel leaded to be
spared his life. The weasel refused, saying that
he was by nature the enemy of all birds. The bat
assured him that he was not a bird, but a mouse,
and thus was set free’. Hence, [4].
38. Only option [1] is grammatically correct. In options
[2] and [3] the use of definite article ‘the’ before
common nouns ‘house’ and ‘street’ is incorrect.
‘House’ in the latter part of option [4] should be
preceded by the definite article, as it has already
been mentioned in the first part of the sentence
and a specific house –‘old house’ is being spoken
of. Hence, [1].
39. B is wrong because it should be ‘rarely has the
economic ascent .... been watched’ rather than
‘rarely .... has been watched’. C is wrong: it should
be ‘the post war era’. Only A and D are correct.
Hence, [2].
40. In sentence A, there is a spelling error. ‘Immigrant’,
not ‘imigrant’, is the correct spelling. Sentence C
has a punctuation error, as it requires a comma at
the end. Sentence D needs the article ‘a’ before
‘owner of dry goods business’. Sentence E has a
tense error: it should be ‘would later become’,
not ‘would later became’. So, only B is correct.
Hence, [1].

77
CATapult
VERBAL REASONING

VA-1.6 JUMBLED PARAGRAPHS AND ODD SENTENCE

PRACTICE EXERCISE 1 to be appropriate opening sentences. Statement


C should follow statement D because statement
C elaborates the idea stated in statement D. Our
1. One can easily get the BC link since ‘they’ in susceptibility towards easy abstractions leads to
statement C refers to the people covered under our tendency to simplify discordant things in our
the National Rural Guarantee Scheme mentioned daily lives. Statement A takes the idea further by
in statement B. Statement A logically fits in as giving the example of India, hence it is the last
the concluding statement since it talks about the sentence in the sequence. So the correct sequence
criticism and carries the thought further. Thus, the is DCBA.
correct order is BCDA.
7. Statement C is the opening sentence of the
2. Statement D is the first sentence as it mentions paragraph as it states the reason why countries
how a company is entering into alliance as if adopted national anthems – which is the main
there is no tomorrow. This should be followed by topic of the paragraph. This should be followed
statement C as ‘that’ in statement C refers to the by statement B as it states one of the oldest
company’s alliances and also mentions the name national anthem. Here the BA link is obvious, as
of that company i.e. Elder Pharmaceuticals. ‘Hurry’ statement A refers to some unusual aspect of the
seems to be a trait associated with a company that song mentioned in statement B, thus providing
is fast growing. Statement A records the various us with the BA link. Also, statement D follows
products that the company markets and flows by way of contrast describing the common and
from statement B. So the correct order is DCBA. typical aspects of the national anthem. Thus the
3. Statement C should be the opening sentence be- correct sequence is CBAD.
cause it gives the general opinion and the spread 8. Logically speaking, either statements A or C have to
of modernization. The other sentences follow from be the first sentence of the paragraph. On reading
this hypothesis and try to validate the fact stated the sentences, we realize that there is a clear AB
in statement C. Statement A follows from C. The link as statement A mentions ‘the patch’ while
BD link is the most crucial one, with the word statement B states how this patch helps. The CD
‘industries’ connecting the two. Thus, the correct link is also clear as statement C mentions how
order is CABD. not all nicotine is bad while statement D states
4. The paragraph talks about the importance of time the uses of nicotine. Since the entire paragraph
in our lives. Hence, the logical flow of sentences is about nicotine and it uses, it makes sense for
would be ADCB i.e., firstly what is time, secondly the CD link to come first followed by the AB link.
how much time do we all have in our lives, then Thus the correct order is CDAB.
how do we spend most of our time and then how 9. Statement C has to follow A, because of the
we miss out on the precious time by indulging in contrast between the two. Since the author is
wasteful things. Thus the correct order is ADCB. referring to a joke in statement A, only statement
5. Statements A and C both seem plausible as the B can precede it as the author wants to know if
opening sentence. The most obvious link in the she is serious about the comment mentioned in
sequence is the DB link because of the quantifi- statement B. Statement D is then the last sentence
ers ‘partly’ and ‘mostly’. Statement B must follow of the paragraph as it explains why the speaker
statement D and not the other way round because asked such a question about Bill Gates. Thus the
the partial reasoning should be followed by the correct order is BACD.
complete reasoning. Thus the correct order is 10. Since the paragraph is about shopping for gold,
CADB. statement C is the opening sentence of the para-
6. Statement B cannot be the opening sentence because graph. This should be followed by statement D as
of the word ‘ultimately’, which requires some kind it mentions why people do not consider shopping
of a precedent. Statements C and D both seem for gold as spending but investing. You will see that

78
Chapter VA
JUMBLED PARAGRAPHS AND ODD SENTENCE 1.6

statement A has to follow statement D: ‘While this to root out immigrants. Statement 6 follows from
may be true’ has to refer to the belief expressed statement A as the last part implies that Malaysia
in statement D. Statement B is the last sentence is beginning to take a more rational approach to
of the paragraph as it states how the Dubai gold migrants labour while statement 6 states how the
trade is making shopping for gold even more country was fighting a losing battle against illegal
exciting. Thus the correct sequence is CDAB. immigration. Thus, the correct sequence is CBDA.
11. ‘His study’ in B refers to ‘Gregory King’s statistical 16. Statement 1 talks about computers being an inte-
study in statement 1. ‘Of these’ in statement D gral part of spacecrafts. This should be followed
refers to ‘five million men’ in statement B. The next by statement D as it mentions the various uses
figure is given in statement A (‘hundred thousand’). of computers in spacecrafts. Since statement
Statement C gives a favourable picture and then A mentions a contrast idea of the first manned
6 shows a contradiction with the use of ‘but’. So spacecraft not carrying any computers, it should
the correct sequence is BDAC. follow statement D. Statement C chronologically
follows statement A as it states how for nearly
12. Statement B should follow statement 1 because
fifteen years, no computers were used in space
‘this’ in statement B refers to the prediction of
missions. The CB link cannot be overlooked on
‘International organizations’. The BD link is also
account of the ‘yet now’ contrast it presents.
very clear: ‘this optimism’ refers to the hope that
Statement B is linked to the last sentence by
India’s progress will happen as predicted. AC is
the word ‘both’. So statement B should precede
a more plausible link than the CA link because
statement 6. Thus the correct order is DACB.
of the ‘not merely... also’ construction. Moreover,
why India is the ‘youngest nation’ (as stated in 17. Statement C begins the sequence as it specifies
statement C) is explained by 6. So the correct Maupassant’s ‘consistent but honest vision’ (grim,
sequence is BDAC. bleak,. . ). Statement B further explains how Mau-
passant depicted the human spirit. Statement D
13. The 1A link stands out. ‘Charles was absolutely
gives an example of his writings in the Boule de
honest’ in statement 1 is qualified by ‘He described
suif episode. Statement A must follow statement
what he believed to have happened’ in statement
D to continuethe Boule de suif episode, which
A. The DC link can be perceived too. The BD link
concludes in statement 6. Thus the correct se-
can be explained: ‘Miss Schlegel had lost her nerve,
quence is CBDA.
as any woman might’ in statement B is qualified
by ‘She had been got safely into the other car’ 18. Statement B explains the issue talked about in
in statement D. Thus the sequence is ABDC. statement 1 and hence must be the opening
statement. This is followed by statement A which
14. Statement A links to statement 1, as it contradicts
poses another question. Statement D continues the
what is stated in statement 1, using ‘yet’. Roh’s
same line of thought as in statement A. Statement
poor run in the elections mentioned in statement
C links to statement 6, which asks if the distinc-
A is supplemented with an example mentioned in
tion stated in statement C is really valid. Thus the
statement D. Then, again, there is a link in CB.
correct order is BADC.
The victory mentioned in statement B naturally
results in his becoming President, as mentioned in 19. Statement C qualifies best as an opening sentence:
statement 6. Thus, the correct sequence is ADCB. it links to statement 1, which specifies lifeguards’
limitations, and also provides a possible solution.
15. ‘Immigration officers encircled a building’ in state-
Statement A goes on to elaborate the working of
ment 1 indicates that they are looking for illegal
the computerized system. Statement D continues
immigrants, which is mentioned in statement C.
this line of thought. Statement B links to statement
There is a CB link as statement B mentions the
6, as they both mention two alternative scenarios
number of undocumented workers found. This should
that the computer has to distinguish between.
be followed by statement D as it mentions how
Therefore, the correct sequence is CADB.
people are willing to volunteer to root out these
illegal immigrants. Statement A is linked to state- 20. BD is the first link. It states that though life
ment D with the phrase ‘harsh as all that sounds’ may have improved for frequent travellers they
which implies that people were being irrational still have to put up with the mysterious rules of
mileage programmes. AC is the next link since

79
CATapult
VERBAL REASONING

‘the selling point’ mentioned in statement A is [4] is not an example of the point that the other
explained in statement C. Statement C then links three sentences are trying to make. An appropriate
to statement 6, as the former mentions what the example would be one in which a person has the
airlines promised, while 6 shows how that promise ability to negate something but chooses not to.
is not fulfilled in reality. Thus, the right sequence Hence, [4].
is BDAC.
26. [3] introduces the subject of the New Development
21. [2] is a better option to start the paragraph as Bank and India’s role in setting it up. [1] continues
it indicates why the major powers are increasing the topic of India’s role in setting up this bank.
their engagement with the Modi government. It [2] is the result of India’s stand in this project.
should be followed by [3] as it states that the Though [4] talks about the BRICS nations, it is
Russian Deputy Prime Minister visited India before in no way related to the setting up of the bank.
the British Foreign Secretary. ‘This guest from Hence, [4].
Moscow’ in [1] is clearly talking about Dmitry
27. [2] introduces the subject of Polavaram and men-
Rogozin. Though [4] also talks about the British
tions the problems involved. This will be followed
(so it can be connected to [2]), it cannot connect
by [3] and [4], as [3] mentions land submergence
to the other two statements. Hence, [4].
while [4] talks about the exact area of land that
22. Though [4] is a decent opening sentence, it does will be submerged. Sentence [1] talks about Tel-
not fit contextually with the remaining sentences angana and doesn’t mention Polavaram. Hence,
as they do not talk about the role of Governors. [1].
[3] is thus a better opening sentence than [4]. ‘It’
28. [1] talks about the daily bloodshed in Gaza. [4]
in [2] refers to the new government and also gives
goes on to mention that even those who accept
an idea as to how the new government could make
Israel’s right to defend itself would be shocked
use of the Sarkaria Commission recommendation.
by the ongoings. [3] mentions why that particular
This should be followed by [1] as it mentions what
house was attacked (the police chief of Gaza City
the new ruling Alliance is doing instead. Hence,
was hiding in that house). These three sentences
[4].
condemn Israel’s actions but the tone of [2] is the
23. [3] is the first sentence in the paragraph as it reports opposite. Hence, [2].
the incident of sprinter Dutee Chand and points to
29. Sentences [4]-[3]-[2] talk about the problems
the problems faced by sports and sportspersons
associated with minority shareholding whereas [1]
in this country. It is followed by [4] as it mentions
suggests a mandate. Hence, [1].
‘this incident’. [2] continues in the same manner
of reasoning and further highlights the failures of 30. The correct sequence is [2]-[1]-[4]. These three
the system. The tone in [1] is more positive as sentences are related to ‘organising information’.
compared to the rest of the sentences and talks [3] mentions a ‘debate’ about memory, which does
about androgen levels which are not mentioned not link to any of the other sentences. Hence, [3].
in any other sentence. Hence, [1].
24. [4] talks about the implications of human extinc-
tion and [3] continues that point. [2] concludes
PRACTICE EXERCISE 2
that it is thus necessary to prevent the threat of
extinction from becoming a reality. [1] is in con-
1. Only statement B mentions the full name of the
trast to the other sentences as it claims that a
video game. Thus it is the first sentence of the
potentially hazardous threat is not worth worrying
paragraph. There is a clear CA link as statement
about. Hence, [1].
C mentions Sony Playstation and Sega Saturn as
25. [2] opens the paragraph by stating what toleration the rivals of the N64 while ‘both companies’ in
is concerned with. [1] discusses the condition statement A refer to these companies. After reading
that needs to be met for something to be called D, it is obvious that ‘the company’ in statement
toleration instead of cowardice or weakness. [3] D refers to Nintendo and also states the reason
clarifies that condition. [3] mentions when toleration why parents are frustrated and why sales clerks
occurs, thus, they make a connected paragraph. are exhausted. Thus the correct sequence is BDCA.

80
Chapter VA
JUMBLED PARAGRAPHS AND ODD SENTENCE 1.6

2. The first statement is clearly C (you don’t need phrase ‘also pushing for’ which implies that be-
to be a cook to figure this out!), as it is the only sides encouraging people to grow mulberries, the
standalone sentence. This should be followed by Fair is also pushing for an accreditation. Thus the
statement A as it asks you to mix all the ingredi- correct order is ACBD.
ents mentioned in statement C. The phrase ‘mix
7. The AC link can be established because statement
well again’ implies that we need to mix again after
C talks of the two compositions mentioned about
adding sugar. Statement D is the last sentence as
in statementA. Statement B continues about the
it states that all the steps have been completed
Eroica symphony. Statement D is obviously a
and the mixture can be chilled until set. Thus the
development from statement B. Statement A has
correct sequence is CABD.
to be the opening sentence because it introduces
3. Only statement D can start the paragraph – all the topic of music and the two compositions. Thus
the others follow after some other statement, for the correct sequence is ACBD.
example, ‘this chemical compound’ is obviously after
8. The BD link is the most obvious one because ‘such
some statement referring to a chemical compound,
a thing’ in statement D refers to the activity men-
and so on. Since statement D mentions that Indal
tioned in statement B. Statement A should follow
was adding value to alumina, statement C comes
statement D as latter mentions transgression and
next because it mentions that ‘special alumina’
the former states the kind of transgression it is.
had to be imported and Indal started developing
Statement C summarizes the idea and also carries
this import substitute. ‘This chemical compound’
the idea forward. Thus, the correct sequence is
in statement A clearly refers to special alumina
BDAC.
and gives us more information about the uses of
special alumina. Statement B is the last sentence 9. The CB link stands out, as statement C talks
as it mentions how the technology required for about the relevance of physical force in the lives
making special alumina was developed in the of primitive savages and makes a contrast by
Belgaum centre. Thus the correct order is DCAB. talking about how physical violence in industrial
civilizations is not really justified. Statement A is
4. The paragraph cannot logically begin with statement
the opening sentence, as it introduces the topic
A. Statement B can be the opening statement as it
of physical force and gives a general view of the
introduces the topic. Statement C logically follows
role it plays in organized society. Statement D
statement B as the former mentions ‘this precious
continues the theme by stating the consequence
resource’ which refers to the oil in statement B.
of physical violence when it becomes the ‘ultimate
Statement D has to be the concluding sentence
arbiter’. Thus, the correct sequence is ADCB.
and ‘therefore’ is an indicator. Statement A can
follow statement C as it the latter mentions how 10. Statement A is the opening sentence, as it introduces
more energy will be taken from available sources the topic of discovering the possibilities of using
while the former states that it will need more recycled water. The BC link can be established,
investment. Thus, the correct sequence is BCAD. as statement B describes how water is purified
by the recycling system and statement C states
5. In this example you can directly spot statement
that ‘public health officials vouchsafe for the purity
D as a possible concluding sentence. The minister
and safety’ of such recycled water. Statement D
could not have viewed the garland of currency
follows next by stating a contrast that the use of
notes before opening the box, thus there is a BA
recycled water should be encouraged except for
link. This leaves us with statement C which is the
the purpose of drinking. Thus, the given sequence
first sentence of the paragraph as it mentions why
ABCD is the correct one.
the RBI governor had a box in his hand. Thus the
correct sequence is CBAD. 11. [3] talks about ongoing protests over the dhoti in
Tamil Nadu. [2] talks about some good that has
6. The AC link makes sense because the word ‘now’
come out of this incident and [4] explains it fur-
in statement C logically links to statement A.
ther. [1] does not fit anywhere in the paragraph.
Statements B and D flow directly from statement
Hence, [1].
C. The BD link is clear as statement B mentions
the vies of one of the organizers of the fair while
statement D follows statement B because of the

81
CATapult
VERBAL REASONING

12. [1] is the beginning sentence of the paragraph [3] gives the minuscule amount of organic matter
as it talks about the way Archie will die. This is that reaches the plains known as Challenger Deep.
followed by [4] that connects to [1] using ‘there- [1] questions the chances of organic matter going
fore’. [3] comes next as it starts elaborating on deeper still. [4] is positive in tone, whereas the
social problems of the US. Though [2] also talks rest of the sentences are negative, so though it
about Archie’s death, it starts a new sub-topic, as is on the same topic, it does not fit with the rest.
it talks about the life of his kith and kin one year Hence, [4].
after the tragedy. Hence, [2].
20. [4] and [3] are connected as [4] talks about the
13. [1] is the correct introductory sentence as it in- significance of minimum pay in mature democracies
troduces the idea of comparing abstract art with while [3] gives its significance in the developing
literature. This is followed by [4] as it compares world. [1] connects to [3] by supporting [3]. [2]
the Edgar Allan Poe’s poems with a painter’s talks about a division in opinion (regarding the
brush strokes. [3] further continues the same idea implications of a minimum-wage floor in the de-
to show how an abstract painter can experiment veloped world) which is not visible in [4]. Hence,
with unusual colours to portray the mood that he [2].
wants to portray. The introduction of photography
21. A careful reading of the paragraph will show that
in [2] is out of context. Hence, [2].
it is about the tomb of Qin Shi Huang, the first
14. [2] is the opening sentence of the paragraph as it emperor of China. [3] introduces the topic; [4]
explains the beginning of the Surrealist movement. mentions a historian’s description of the tomb,
This is followed by [3] and [1] as they express and [1] continues this description. Only [2] does
its beliefs. [4] mentions some famous artists and not fit into the sequence, as it links only to [3],
doesn’t belong to this paragraph. Hence, [4]. but not to the other two sentences. Hence, [2].
15. [2] states Nietzsche’s philosophy of listening to 22. 1 states how the best way to communicate and
music with our muscles. It is exemplified in [4]. share resources is to create a website. Statement
[1] states that all these actions can occur without D continues from the 1st statement to explain how
our knowledge or volition. Though [3] talks about a setting up a website can be a difficult endeavour.
philosopher too, it cannot connect with any other This should be followed by statement B since it
sentence except [2]. Hence, [3]. explains what things need to be done to set up a
website. There is a clear BC link as well as ‘all this’
16. [3] explains how our brains change when we
in statement C refers to the various poits mentions
learn a new task. [2] supports this statement by
in statement B. Statement 6 follows logically from
mentioning that the brain is flexible even after a
statement A, as statement A introduces the topic
particular age. [1] further strengthens the argument.
of a free web space provider, and 6 elaborates on
[4], which explains how our brains coordinate a
it. Therefore, the correct sequence is DBCA.
complex set of actions, does not fit with the other
sentences. Hence, [4]. 23. 1 talks about the discovery of the biologists at
the University of California and statement B de-
17. [4] is the opening sentence of the paragraph as it
scribes the experimental study. ‘The achievement’
defines what a ‘group’ is. It is followed by [3] and
in statement A refers to the discovery mentioned
[1], which mention shortcomings of the definition.
in statement B. thus there is a B link. Statement
[2] talks about the importance of the definition
D follows next with ‘this issue’ referring to ‘the
and is too positive to fit with [3] or [1]. Hence,
absence of a genetic mechanism...’ in statement
[2].
A, instead of statement C, as the question men-
18. [2] recommends actions based on a premise. [4] tioned in statement C is a question asked by the
and [1] ask questions that man will be faced creationists. Thus, the C6 link is clearly established.
with while undertaking those actions. [3] asks a Thus, the correct sequence is BADC.
completely different question as compared to [3]
24. Statement C elaborates on 1, so it begins the
and [1]; it is not about knowledge. Hence, [3].
sequence. Statement D links to statement C by
19. [2] mentions that any nourishment must come providing a contrast using the word ‘but’. Since
down from the ocean’s surface to the plains below. statement A describes how the Sunil Gavaskar

82
Chapter VA
JUMBLED PARAGRAPHS AND ODD SENTENCE 1.6

Legend of Cricket Bat is designed, it follows as it adds to the information given in the latter.
statement B. Thus the correct order is CDBA. Also, AB is a clear link as statement A states the
flaws with the current definition while statement
25. ‘They’ in statement B refers to the PETRO group,
B gives the new definition’s parameters. Thus, the
so we get the 1B link. ‘It’ in statement D refers to
correct sequence is CEABD.
‘the petroleum industry’ instatement A, so there’s
an AD link. 6 follows from statement C. Thus the 30. Statement B should begin the passage as it intro-
correct sequence is BADC. duces Mahatma Gandhi and is the only sentence
that contains his name. Statement C must follow
26. The BA link is quite clear - there is a chronological
statement B as it elaborates on the latter. EAD
link [‘a few minutes ago ...’, ‘now ...’] as well as
belong together as statements A and D support
the contrast in mood. The DC link is also clear
statement E. Thus the correct sequence is BCEAD.
as statement D states how unimpressed she was
by the exterior of the building while statement
C mentions the frescos of Giotto i.e the internal
artwork of the building. 6 clearly follows statement PRACTICE EXERCISE 3
C as the latter mentions how she was unable to
bring out the correct response after seeing the 1. Statement C should begin the paragraph as it
frescos while 6 mentions that she didn’t want to introduces friendship and gives some past infor-
be enthusiastic about unimportant monuments. mation about it. Statement E refutes statement C.
Thus the correct sequence is BADC. Therefore, CE is a link. Statement B should follow
27. The statistics in statement B link it to 1 which as it explains the current situation. Statement D
has contrasting statistics. The DA link is clear as elaborates the fact mentioned in statement B by
statement D asks a questions while statement A giving examples. So it should come next. State-
mentions how French entrepreneurs are asking this ment A rounds off the paragraph by saying that
question. Statement C follows next as it mentions the trend is catching on quickly. Thus the correct
Dover which is across the channel and where taxes order is CEBDA.
are low as implied by the phrase ‘fiscal paradise’. 2. Statement D states that rulers since the Pharaohs
It also connects to 6 which gives the answer to the have regularly cajoled and threatened upstream
question asked in statement D. Thus the correct nations. This introduces the topic and should come
sequence is BDAC. first. Statements A, B and E give examples of the
28. Statements A and D make a pair as both state the measures taken by Egypt to prevent tampering
advice given by Lucena. The word ‘also’, at the with Nile’s water. They should ideally be in the
beginning of statement D, makes it clear that the chronological order. Therefore, the sequence should
sequence should be AD. ‘Each grand master’ in be ABE. Statement C must follow statement B
statement E refers to Garry Kasparov and Vladimir [and cannot be placed after E] because statement
Kramnik, the two best players in the world, men- C talks about the same pact or treaty introduced
tioned in statement C. Therefore, the CE sequence. in statement B. Thus, the correct sequence is
However, statement B describes how jockeying is DABCE.
a time-honoured tradition while A states a specific 3. The paragraph should start with statement B
example of the same. Therefore, the link BA seems as it introduces the subject Betty Ford and her
natural. Thus, the correct sequence is BADCE. illness [on which the entire paragraph is based].
29. ‘This score’ in statement D refers to the ‘new The extent of the illness is given in statement A,
definition of superpower’ mentioned in statement therefore, it should come next. Between statement
B. Therefore, the BD link. A logical sequence would E and statement D, the latter does not appro-
be one where the paragraph starts by stating the priately link to statement A as it implies that
earlier definition, then moves to pointing out flaws cancer is an occupational disease of America’s
in the earlier definition and finally suggests a new political life, which doesn’t make sense. Having
definition. Statement C states the current defini- discussed the physical ordeals of Betty Ford, the
tion and is, therefore, clearly the beginning of the author introduces a contrast and discusses the
paragraph. Statement E must follow statement C psychological ordeals faced by her. This contrast
is shown by the use of the word ‘but’. Therefore,

83
CATapult
VERBAL REASONING

statement E should follow statement A. The extent ‘for example’. Then, statement B follows. So, the
of psychological ordeal is discussed in statement correct sequence is DCAB.
C. Therefore, statement C follows statement E.
8. The first sentence has to be statement C as it
Statement D talks more about the psychological
introduces the subject - the response. We see
ordeal and links itself to statement C, and is an
that statement D has to follow statement A. Since
appropriate conclusion to the paragraph. Hence,
statement B mentions how the entries were touch-
the correct sequence is BAECD.
ing tales and how it destroys a myth, it should be
4. ECD is a link. Statement C follows statement the concluding sentence of the paragraph. Thus
E because the former talks about the lack of the correct sequence is CADB.
electricity mentioned in statement E. Statement
9. With statement D which introduces the subject -
D comes next because it sums up the situation.
‘Reserve Bank’ - being the first statement, statement
Statement B should come before ECD [and not
B as the second falls into place. Since statement
after] as the ECD link constitutes an example of
C mentions what modification was made in its
how Indian voters have raised their expectations.
monetary policy, it follows next. Statement A is
So, statement B is a more appropriate starting line
the last sentence as it states what was changed
than an ending line. Statement A should come
by the RBI. Thus the correct sequence is DBCA.
after ECD as it explains how ‘he knew exactly
what he was missing’. Thus, the correct sequence 10. Only statement C can start the paragraph as it
is BECDA. states the names of the state airlines along with
the new airlines. This should be followed by
5. The paragraph is about lack of access to morphine
statement A as ‘most and many’ mentioned in it
in some countries. So the paragraph must start with
refer to these new airlines. Statement D further
statement C, which introduces Dr. Eric Krakauer
elaborates why their ride has been bumpy while
and the problem. Statement A talks about the
statement B provides the contradiction by stating
actual problem: misinformation about morphine.
that fliers have benefitted from this fierce com-
Therefore, statement A comes next. Statement D
petition. Thus the correct sequence is CADB.
explains the extent of the misinformation, so it
must come after statement A. Statement B gives 11. Statement A has to follow statement C, since it
information that contrasts with statement D and is a reaction to statement C. This should be fol-
also clears the misconception; therefore, it fol- lowed by statement D as it answers the question
lows statement D. Statement E explains how the put forth in statement A. There is a clear DB link
discussion has diverted the attention of people as statement D talks about going in the past and
from seeing the possible benefits of morphine. statement B gives an example of how grown-ups
Therefore, it is the last sentence. Thus the correct looked down on crying. Thus the correct sequence
sequence is CADBE. is CADB.

6. One clear link is FC, as statement C directly 12. The one sentence that introduces you to the passage
contradicts statement F. Another clear link is DA: – i.e., the logical first sentence of the passage - is
statement A contradicts statement D. Statements statement C. This can be followed by statement
E and B [in that order] explain the talent of the A as it mentions how even sapphire hawkers try
molecule and thus, must follow statement A. Thus to prove the genuineness of their sapphires. This
the correct sequence is FCDAEB. should be followed by statement D as it mentions
the gem shops in big hotels which contradict the
7. The paragraph talks about the problems associated
small-time hawkers in statement A. Statement B
with health care and possible solutions. Statement
is the last sentence of the paragraph as it states
D comes first as it initiates the discussion about
the success story of Thai precious rocks. Thus the
the problem. Statement D states that the solutions
correct sequence is CADB.
‘need not be complex’ and statement C states
that the simplest and perhaps the least obvious 13. Statement D is the first statement as it mentions
health programmes have proved effective in many racing’s biggest weekend. This should be followed
countries; thus the DC link. Then the examples are by statement A as it mentions the race – Classic
discussed. Statement A comes next as it is the ITC Cup. Since statement A mentions the prize
first example, which can be seen from the phrase money, it should be followed by statement B

84
Chapter VA
JUMBLED PARAGRAPHS AND ODD SENTENCE 1.6

which states how the horses taking part in this 18. One is likely to come across a signboard only
race are some of the best. (‘Also’ is another clue on reaching the village. So the CB link. On first
which suggests that statement B should follow impression, the sign board gives a warning. Thus
statement A). Statement C is the last sentence statement E follows from statement C. When one
as talks about riding the best horses which are is surprised about something, one will try to ‘make
already mentioned in statement B. Thus the correct sure’, so statement D should follow statement E.
sequence is DABC. Once you stop and read, you see what is actually
written. Thus the correct sequence is CBEDA.
14. This paragraph, comprising conversational state-
ments, is unusual but one can follow the same 19. Clearly statement A or B cannot start the sequence.
method. Statement D is the first sentence as it In fact, statement B provides the link to statement
mentions how one can see but cannot be felt. E with the contrast between what is ‘easy enough’
There is a clear DA link as statement A is a in statement E and the ‘hardest part’ in statement
question asked about the creature mentioned in B. Hence we have EB followed by the question in
statement D. Logically, one sees that statement statement A and its answer in statement D. Thus
B follows statement A, as it is the answer to the the correct sequence is EBADC.
question asked in A. Statement C is the last sen-
20. The most obvious link EC as statement E men-
tence since it states that the speaker is a cloud
tions ‘horizons’ and statement C elaborates upon
which is implied by the phrase ‘for such a one
it with the phrase ‘these horizons’. Again, there is
am I’. Thus the correct sequence is DABC.
a crucial link in AD. ‘Marginal product of capital’
15. There is a clear DB link as statement D mentions in statement A is explained in statement D, and
an inscription while statement B mentions in whose statement B closes the sequence. Thus the correct
memory this inscription is written. Statement C has sequence is ECADB.
to follow statement A, as it supplies the translation
21. Statement C the first sentence of the paragraph
that is said to be missing in A. Thus the correct
because the whole discussion centres round the
sequence is DBAC.
interaction of the teams. This should be followed by
16. It is difficult to find a starting or an ending point statement E which mentions that potential teams
in this sequence, therefore let us try and look for fail to spend time together. There is a clear AD
links. There is a BD link as statement B mentions link as the idea stated in statement A is carried
‘they look like perfect pets’ while statement D forward in statement D. Statement B states a
specifies what ‘they’ means i.e. lizards and pet contradiction to the AD link and thus follows them.
reptiles. The other link is DC, ‘salmonella’ being Thus the correct sequence is CEADB.
the common factor between the two sentences.
22. Statement C begins the sequence as it introduces
This should be followed by statement A as it
the subject of ‘mind’. This is elaborated in statement
mentions how people can get infected by these
A by the reference to ‘ancient times’. Statement E
bacteria. Thus the correct sequence is BDCA.
follows statement A by explaining this concept of
17. Statement B follows immediately after statement ancient times by using the word ‘then’. Statement
D, as it explains how the all-important space D follows statement E as it explains how the
(mentioned in statement D) can be utilized to difference between soul and spirit fluctuated over
maximize profits. This should be followed by time. Statement B is the concluding sentence of
statement C as it gives the example of grocery the paragraph as it mentions how the metaphor
stores and how they utilize space. There is a CE of the breath of life expressed these ideas. Thus
link as well since statement C mentions ‘grocery the correct sequence is CAEDB.
stores’ and ‘they’ in statement E refers to these
23. The main point of discussion is in statement B.
same stores. Statement A is best suited as a con-
Thus, it should start the sequence. ‘Recommended
cluding sentence, as it mentions that other types
introduction’ in statement B takes us to statement
of retail stores have begun to adapt grocery store
D (‘but before we introduce exchange traded in-
ideas (mentioned in the previous three sentences).
terest rates’). Thus, the BD link. ‘Understanding
Thus the correct sequence DBCEA.
ground realities’ in statement D is naturally to
know ‘the basic issue’(mentioned in statement

85
CATapult
VERBAL REASONING

E). The final sequence should be CA as statement F are elaborated upon in statement C. The other
C mentions what a regulatory regime should not link is CD, ‘discipline’ being the common factor
do and statement A states how it should ideally between the two sentences. EA is a better link
function. Thus the correct sequence is BDECA. as statement E compares taste to character in
the aesthetic and moral life respectively followed
24. The BD link is fairly clear: statement B mentions
by statement A which states a new topic about
that Wal-Mart has had to support its merchandiz-
the universal standard of taste. Thus the correct
ing performance, and statement D shows how it
sequence is BFCDEA.
has done so. The AC link is another obvious one:
statement A talks about a business strategy and 29. The only stand-alone statement is A, as it intro-
statement C mentions the case of Wal-Mart as duces the subject of discussion. The ‘record’ in
an example. Statement E talks about small town statement A is further elaborated in statement E.
customers traditionally being the focus of Wal- The next statement is B. Statements D, C and F
Mart while statement B mentions how Wal-Mart give the logical sequence of the chain of reactions
has come closer to urban customers in the search that followed the assault on Mr. Prescott. Thus the
of growth. So the EB link is also clear. Thus the correct sequence is AEBDCF.
correct sequence is ACEBD.
30. Statement E is the first sentence of the paragraph
25. Statement E is the first sentence of the paragraph as it mentions temperance as the virtue of moder-
as it mentions the name of the film which is the ation and self-control in anything. ‘But especially
topic of the paragraph. This should be followed by in pleasures’ in statement C links it to statement
statement C because of the phrase ‘this Academy E. The next link is AB, since statement A mentions
Award-winning film’. Statement D starts narrating Greek virtue while statement B mentions Plato’s (a
the plot of the film as follows the EC link. The Greek) views on the same. The other link here is
last two sentences should be BA, as statement the BD link. Statement B talks about Plato’s ideas,
B states that the characters engage in practical which are elaborated in statement D. Statement
jokes, and statement A gives examples of some F follows statement D, as the difference between
such jokes. Thus the correct sequence is ECDBA. ‘higher and lower parts’ is further elaborated in
statement F. Thus, the BDF link becomes vital.
26. Only statement B is possible as an opening sentence.
Thus the correct sequence is ECABDF.
Logically, it should be followed by statement E as
‘medicine’ is mentioned first, followed by statement
D (‘Psychology’) and statement C (‘Criminology’).
Thus the correct sequence is BEDCA. PRACTICE EXERCISE 4
27. Statement D is the opening statement, as it is
the only stand-alone statement that introduces 1. Statement B is the opening sentence as it intro-
the topic of ‘radical feminism’. The DC link is duces the idea of the systematic methodology of
prominent, as ‘this form’ in statement C refers mass killings in the Holocaust. The AD link is very
to ‘radical feminism’ mentioned in statement D. strong, as statement A talks about ‘detailed lists’
Statement A presents a contrast to the idea in and statement D talks about ‘meticulous records’.
statement C, which states that radical feminism Statement E follows next, as it continues the idea
is not popular today. The EB link carries forward of the methodology mentioned in the preceding
the idea in statement A that feminism is not only statements. Statement C is the last sentence, as
about radical feminism. Statement E states that the phrase ‘in addition’ points out. It brings in
oppression cannot be generic and statement B gives a new topic - new methods of killing. Thus the
an example of that. Thus the correct sequence is correct sequence is BADEC.
DCAEB.
2. Since the entire paragraph is based on radiation
28. ‘Taste’ with its historic significance in statement B and colour, statement C is the first sentence of
should start the sequence. ‘It’ in statement F refers the paragraph as it mentions electromagnetic ra-
to ‘taste’ and the aesthetic experience spoken of diation to be a mixture of different wavelengths
in statement B. So, we get the BF link. The ‘two and intensities. There is an EBD link as statement
distinguishable elements’ mentioned in statement E mentions which radiation is called light while

86
Chapter VA
JUMBLED PARAGRAPHS AND ODD SENTENCE 1.6

the phrase ‘the light’s spectrum’ in statement B 7. Statement E must be the beginning of the paragraph
links it to statement E and ‘the full spectrum’ in or the end as it fits in as an introductory line as
statement D links it to statement B. Since statement well as the conclusion of the passage. BA is a pair
A talks about many more spectra, besides colour because both mention two programs introduced
sensations, it should conclude the paragraph. Thus for helping out borrowers. Statement C describes
the correct sequence is CEBDA. the failure of these programs. Therefore, it should
come after BA. The phrase ‘in fact’ shows clearly
3. Statement B seems like a much better opening
that statement D should come right after C. Thus
sentence than statement A, as it is more general
the correct sequence is EBACD.
in nature. There is a clear EA link as statement
E mentions how tango was not performed in this 8. EC is a strong link as statement C talks about
way earlier and how new forms of the dance the ‘offer’ that is mentioned in statement E. Also,
continue to develop while statement A states how statement E is the most appropriate statement to
tango was performed in its formative days. ‘And’ start, as it states an incident which is the trigger
in statement C implies that women didn’t always for the actions described in the entire paragraph.
give up control within the dance and was another The reply of the Maoist leader can be received only
problem faced by tango dancers. Statement D after the offer has been made to him. Therefore,
follows statement C, as it gives an example of statement B should follow statement C. Statement
how women started taking control in tango. Thus D adds to the rejection of the offer by stating
the correct sequence is BEACD. that the Maoists refused to give up arms too.
Therefore, statement D comes after statement B.
4. Statement B has to be the first sentence, as it
The reaction of the state police to the rejection
is the only one that mentions the person being
of the offer logically comes last. Thus the correct
discussed by name (i.e. Matthew), whereas all
sequence is ECBDA.
the other statements use only pronouns. Since
statement A describes how Matthew was a quiet 9. The passage is about the diplomatic dialogue
person and preferred talkative people as long between the United States and India. Statement
as they didn’t expect him to talk. The ‘but’ in E marks the start of the topic, as it is the only
statement E is a contrast to his intrinsic nature standalone sentence. Statement D goes on to ex-
and follows statement A. The other link is ED, plain why the recently concluded dialogue is just
‘girl’ being the common factor between the two another talking shop. Therefore, D should come
sentences. Statement C gives the reason as to next. Now looking at the remaining statements, B
why Matthew thought little girls to be worse than introduces an idea which is opposite to the idea
women. Thus the correct sequence is BAEDC. discussed in the first two statements. Thus the use
of the word ‘but’ in statement B links it strongly
5. The link which helps solve this question is the EA
to statement D. BA is another link as the ‘first’
one. Statement A elaborates on the information
indicates that A is the first key shift of the ‘two
given in statement E. ‘The process’ mentioned in
key shifts’ mentioned in statement B. Statement
statement C is stated in statement A and thus the
C mentions the second of the two key shifts and
AC link is formed. ‘These’ in statement D refers
it should come next. Thus the correct order is
to the ‘poorly embalmed bodies’ in statement B.
EDBAC.
so there is a BD link as well. Thus the correct
sequence is EACBD. 10. Statement B must begin the paragraph as it
mentions India’s track record of producing great
6. There is a strong link between statements C and
writers but not buying books. In statement E ‘that
E, as the latter qualifies the expression ‘apparent
lack of enthusiasm’ refers to statement B, which
lack of purpose or cause’ in the former. Since
mentions that there are not enough buyers of
statement A talks about cause/effect, it comes
books of Indian authors. So there is a BE link.
after CE. There is another link between B and
This should be followed by statement C as it men-
D - the former mentions animals and the latter
tions how only one foreign publishing house was
goes on to give an example about animals. Thus
located in India. Statement D is a contradiction
the correct sequence is CEABD.
to the views presented as it mentions how the
brightening of the economic outlook in the country

87
CATapult
VERBAL REASONING

is helping aspiring authors and publishers. DA is 15. [1] introduces the topic of impressionist paintings
a mandatory pair because statement A provides and how they were different. [4] should follow
the proof of the argument in statement D. Thus [1] as it talks about another point of difference.
the correct sequence is BECDA. [3] and [4] are connected as [4] talks about the
change in the treatment of colour and outline
11. The passage sequence cannot start with A, B, or
while [3] states the change. However, [2] talks
C because of ‘but’, ‘they’ and ‘furthermore’ re-
about impressionist painters (not paintings) and
spectively. Because of the clearly stated DA link,
their lack of interest in philosophy. Hence, [2].
statement E cannot be the opening sentence. Thus
D starts the sequence, followed by A. Statement E 16. [4] mentions Clark’s theoretical arguments. [2]
mentions how copyright regulations were announced introduces the theoretical argument. [1] continues
last year. ‘They state’ in statement B refers to the that line of thought. However, [3] talks about an
regulations mentioned in statement E. Statement alternative to constructivism. Hence, [3].
C follows the EB link as it mentions some other
17. [2] should start the paragraph as it introduces
regulations to the replicated or copied versions
how reading comprehension takes place. [4] and
as pointed out in statement B with the phrase
[1] are connected by the ‘word recognition’ link.
‘similar works of art’. Thus the correct sequence
[4] gives a condition for proficient reading and [1]
is DAEBC.
talks about what happens if that condition is not
12. Statement E follows statement B (which begins the met. [3] talks about analysing text without reading
sequence by introducing a link between Canada it. It does not follow the same line of thought as
and Bollywood) because statement E elaborates the other sentences. Hence, [3].
on the comment made in statement B. Statement
18. [2] mentions the effects of caffeine and attributes
D follows statement E as the ‘projection’ in state-
them to the inhibition of the production of phos-
ment D refers to the figure in statement E. AC is
phodiesterase. ‘Cyclic AMP’ connects [1] and [2].
another prominent link as A.S.Rattan in statement
Both [1] and [3] explain how the effects of caffeine
A is referred to as ‘him’ in statement C. Thus the
come about. [4] weakens the argument put forth
correct sequence is BEDAC.
by [2]-[1]-[3]. Hence, [4].
13. Statement B is the first sentence of the paragraph
19. [4] gives the meaning of war as per the Oxford
as it mentions the topic of the paragraph i.e. chan-
Dictionary. ‘This’ in [2] refers to the definition in
nel conflict. This should be followed by statement
[4]. [2] praises the definition and then [1] goes
E as it mentions how channel management can
on to question an aspect of it [3] talks about the
help to understand a conflict’s source and gravity.
‘philosophy’ of war and not its definition. Hence,
Since statement C elaborates on the destructive
[3].
kind of conflicts, it follows the BE link. There is
a DA link as well since statement D mentions the 20. [1] and [4] are clearly related to each other due
reaction of ‘the best manufacturers’ and statement to the phrases ‘on one hand’ and ‘on the other’.
A that of the ‘less sophisticated players’. Thus the Since these sentences talk about disease and
correct sequence is BECDA. evolution, [2], which talks about the problems
of climate change and antimicrobial resistance
14. Statement A is the opening sentence because it
should precede them. Though [3] also talks about
opens the topic of the lyre bird’s behaviour. On
protecting the planet, it doesn’t fit with the other
close observation, the AD link is clear as ‘reports’
three sentences at all. Hence, [3].
in statement D is in direct relation to the observa-
tions made by the ornithologists in A. Statement 21. [2] is the opening sentence followed by [4] (‘such
B flows from statement D, as that there is no naturalness’ refers to Merkel’s actions in [2]) and
‘mistaken identity’ issue is further confirmed in [1]. [3] only connects with [2] but none of the
statement B. The other link is BE, ‘the bower other sentences. Hence, [3].
and the lyre’ being the common factor between 22. [2] is the only possible opening sentence. [1] and
the two sentences. The CF link is also clear as [3] are connected as the hints about job cuts would
statement C mentions the sweet things whispered have led to speculation. [4] doesn’t connect as
to the female and statement E giving an example well to [1] as [3] does because [1] talks about a
of the sweet things whispered. Thus the correct past event while [4] talks about future possibilities.
sequence is ADBECF. Hence, [4].

88
Chapter VA
JUMBLED PARAGRAPHS AND ODD SENTENCE 1.6

23. [2]-[1]-[4] make a better connected paragraph than two commentaries mentioned. [2] follows [4] as it
any other combination. [2] talks about how lyrics elaborates on how explanations have been written
are important to music, [1] gives the example of in these commentaries. This is followed by [3]
the singer and [4] mentions what comes out of which further expounds on the rules explained
in the commentaries. It can be deduced that [2]
his mouth i.e., the lyrics. Hence, [3].
and [3] are elaborating on [4], not [1], because
24. [4] is the opening sentence as it talks about the of the ‘rule and procedure’ link. Hence, [1].
two kinds of emotions related to music. This is 6. [2] introduces the author’s perception about human
followed by [1] as it elaborates on ‘perceived’ rights not providing a fully comprehensive account
and ‘felt’ emotions. [3] follows [1] as it further of morality. [1] exemplifies [2]. [4] talks about what
differentiates between the two kinds of emotions. human rights do instead. [3] talks about human
[2], which talks about noise levels and processing rights being one of many moral perspectives, which
is a tangential point. Hence, [3].
difficulty, does not connect to any sentence. Hence,
[2]. 7. [3] starts the paragraph as it talks about the cam-
paign of the restaurant workers. [2] is a response
to this campaign while [4] gives a reason for the
response. Though [1] also talks about food stamps,
PRACTICE EXERCISE 5 it talks about ‘poor Americans’ in general while the
rest of the sentences are specific to restaurant
workers. Hence, [1].
1. The paragraph starts with [2] which explains that
when Music and the Brain was published 20 years 8. [3] introduces the crucial element of sympathy.
ago, neuroscience was not advanced enough to [1] and [2] follow; they are connected as they
explain the phenomena mentioned in [2]. This talk about beneficial and harmful experiences and
is followed by [4] as it explains how the last 20 their effects on sympathetic people. [4] is in slight
years have seen advances in this field. [4] also contrast to the remaining sentences as ‘sympathy’
asks why music has so much power over us. [3] requires us to understand what another person is
mentions how relevant that question is. Thus, the experiencing. Hence, [4].
correct sequence is [2]-[4]-[3]. Though [1] also 9. [3] mentions the tendency of Indian scholars to
talks about music, it does not contextually fit with exaggerate the achievements of a particular math-
any of the other sentences. Hence, [1]. ematician, ignoring his contemporaries. [1] reveals
2. [2] is the opening sentence as it talks about how the result of this tendency. This is followed by [2]
therapists help clients identify the problematic as it talks about the tendency of Western historians
beliefs. [4] follows [2] as it reveals what this to ignore the mathematical achievements of India.
stage is called and how it helps the patients. [3] ‘Again, even more unfortunately’ connects [2] to
further elaborates on functional analysis. [1] talks [3] and [1]. The tone of [4] contrasts with the tone
about negative thinking patterns, which are not of the rest of the sentences as [4] undermines
mentioned in any of the other sentences. Hence, India’s contribution to mathematics. Hence, [4].
[1]. 10. [1] is the opening sentence as it asks the question
3. [4] puts forward a conditional argument. [1] and about being altruistic. This is followed by [4] as it
[3] support it with results. [2] talks about the explains why biological altruism does not answer
‘hardness’ of scientific research and is not related the question. ‘This’ in [3] refers to the information
to any other sentence. Hence, [2]. given about biological altruism in [4]. [2] gives
a different perspective on evolutionary theories
4. [2] states that humans have certain common needs by stating their usefulness while the intention of
and values. Based on this statement, [1] states the rest of the sentences is to reveal their lack
that it is possible to develop a scientific system of usefulness. Also, [2] seems redundant as [4]
of ethics in the future. [3] follows [1] as it states gives the same information. Hence, [2].
what will continue to happen till such a system
is developed. [4] also talks about motivations and 11. Statement B defines the ancient form of Origami.
values but attributes these to biology and early The conjunction ‘but’ in statement D is significant
experiences. These two factors are different from as it states how Origami is assuming a new role
what the remaining sentences discuss: needs, these days. So one gets the BD link. Statement
drives, interests, etc. Hence, [4]. A should precede statement C, as Martin Kruskal
is introduced in statement A. Thus the correct
5. [4] is the opening sentence of the paragraph as sequence is BDAC.
it introduces the most important feature of the

89
CATapult
VERBAL REASONING

12. Note the AB link. ‘Since then’ in statement B fol- formation given in statement B. Statement A lists
lows the time ‘when the oil crisis hit the world’. the diseases that statement D talks about. Thus
But ‘at that time’ in statement D refers to the the correct sequence is CBDA.
time before the oil crisis hit the world. So state-
18. Statement A is the opening sentence of the
ment D must follow 1. The C6 link is prominent,
paragraph as it mentions how over the years,
as statement C gives an example of the kind of
Florida’s real estate values have become beyond
cars being created by Chevrolet and 6 talks about
the reach of many families. Statement D can only
the possibility of launching this car in the Indian
relate to statement A since it mentions how long
market. Thus, the correct sequence is DABC.
the dysfunctional property-tax system has been
13. ‘Those rates’ in statement D refers to the rate haunting Florida. There is a BC link as statement
mentioned in statement A, so statement D has to B states how in the 1990s a homestead measure
follow statement A. Statement A seems to make was introduced resulting in a cap on assessed
more sense if it follows directly from statement property value while statement C mentions how
C. Since statement B mentions another crises, it base assessment values are different when houses
is the concluding sentence of the paragraph. Thus are sold. Statement E is the concluding sentence
the correct sequence is CADB. as it mentions specifically to property levies in
only Southern Florida while the rest of the sen-
14. The passage discusses the types of environmental
tences are for Florida in general. Thus the correct
standards. This is introduced in statement A with
sequence is ADBCE.
reference to the U.S. The three types of environ-
mental standards follow in the order in which they 19. After the first statement, there seem to be two
are mentioned, so statements C, E and B follow. possible options for the next sentence: statements
However, we find that statement D qualifies what A and B. However, if statement A is chosen, there
statement E has to say, so the order is C-E-D-B. is no way in which statement D could fit in later,
Statement F is an example of what is stated in as it is an argument which supports the ‘race to
statement B, so it follows statement B. Thus the bottom’ view. Thus, the second sentence is state-
correct sequence is ACEDBF. ment D, and the third sentence is statement A.
Statement C mentions what the companies would
15. The passage should start with C because it is
actually look to do, and statement B describes
the only stand-alone sentence. Statements E and
what effect this outlook may have on the economy.
B follow statement C, as they both explain how
Hence, the correct sequence is DACB.
Indonesia and Vietnam respectively ‘come out
differently’, as mentioned in statement C. ‘Such
are the two vagaries’ in statement D refers to the
examples in statements E and B, and so follows
them. Statement A concludes the sequence. Thus
the correct sequence is CEBDA.
16. The paragraph must start with either statement B
or E, as all the other statements refer to O’Brien
as ‘he’. Statement B is a better opening statement,
as it states O’Brien’s origins. Then comes state-
ment D which talks about his job. The word ‘later’
shows that statement A goes on to elaborate on
his work. ‘Towards the end of his career’ marks
the end of the paragraph. Therefore, statement C
is the last sentence. Thus the correct sequence
is BDAEC.
17. Statement C is the opening sentence as it intro-
duces the topic of how cures for certain diseases
are being found. The CB link is established, as
‘these diseases’ in statement B refers to the set
of diseases mentioned in statement C and not the
set mentioned in statement A. Also, there is no
logical link between statement C and any of the
other sentences. Thus it is the opening sentence
of the paragraph. The BD link can be established,
as statement D makes a comparison with the in-

90

You might also like